Board review questions





Chapter 1 : History of pediatric critical care medicine




  • 1.

    Treatment for which of the following disease entities was not an important trigger in the early development of distinct, full-time, multidisciplinary pediatric intensive care units:



    • A.

      Measles


    • B.

      Poliomyelitis


    • C.

      Reye syndrome


    • D.

      Tetanus




Preferred response: A


Rationale


In Europe, pediatric intensive care followed shortly after the poliomyelitis epidemic in Denmark in 1952. In 1955, Dr. Goran Haglund, a pediatric anesthesiologist, established the first medical-surgical pediatric intensive care unit (PICU) for infants and children at the Children’s Hospital in Göteborg in Sweden.


In France, in 1963, a newborn presented with tetanus and was admitted to l’Hôpital des Enfants Malades of Paris. Shortly afterward, Dr. Gilbert Huault and J.B. Joly, both neonatologists, opened the first multidisciplinary PICU in France at Saint Vincent de Paul Children’s Hospital. This unit was the first pediatrician-directed PICU in Europe; it soon became a major influence on the development of PICUs.


In the mid-to-late 1970s, as pediatric cardiovascular surgery for more complex lesions in infants was developing, nurses provided postoperative care in designated units. Children with Reye syndrome suddenly appeared, requiring complex multisystem care. In addition, in the 1980s, emergency medical services (EMS) systems began transporting severely injured children to hospitals, where they required rapid assessment and intervention by nurses and physicians and initiation of cardiorespiratory and neurologic support.



  • 2.

    Of the following, which most profoundly influenced the development of early distinct, geographically separate, multidisciplinary pediatric intensive care units?



    • A.

      Advanced forms of mechanical ventilation


    • B.

      Federal government finance programs


    • C.

      New therapeutic interventions for oncology patients


    • D.

      Nursing




Preferred response: D


Rationale


Pediatric critical care medicine (PCCM) developed initially through the efforts of pediatric anesthesiologists, as well as pediatric general and cardiac surgeons, and neonatologists. In fact, most of the original PICUs were founded by pediatric anesthesiologists. Before discrete, geographically separate, intensive care units evolved, critically ill children often received close monitoring, intensive nursing care, and pulmonary support in the postanesthetic recovery room.


Chapter 2 : High-reliability pediatric intensive care unit: role of intensivist and team in obtaining optimal outcomes




  • 1.

    The three dimensions of healthcare quality stated by Donabedian are:



    • A.

      Reliability, operations, resilience


    • B.

      Safety, efficiency, outcome


    • C.

      Structure, process, outcome


    • D.

      Structure, system, efficiency




Preferred response: C


Rationale


Avedis Donabedian, an early “systems thinker” in healthcare, stated that healthcare quality should be based upon three dimensions: structure-process-outcomes. Structure is the setting in which care is delivered. Process refers specifically to how care is provided, including incorporation of high reliability principles into daily activities. Outcomes refer to endpoints of care, including commonly used quality and safety measures, and other key outcomes such as length of stay, patient/family experience, and cost/value.



  • 2.

    A care bundle is defined as:



    • A.

      A flow-chart to guide provider decision making for a disease process


    • B.

      A list of standardized, best practice interventions for a patient population or disease


    • C.

      A way of designing an intensive care unit to cohort patients according to their disease process


    • D.

      Collaboration between multiple hospitals to provide the best possible patient care




Preferred response: B


Rationale


A care bundle is a relatively short list of standardized, generally evidence-based or best practice interventions for a patient population or disease that, when implemented consistently, leads to improved outcomes. It is the combination of elements performed consistently and in aggregate that drive improvement. A clinical pathway is a flow chart to guide provider decision making.



  • 3.

    The PICU leadership team at your hospital is discussing the results of a recent analysis of a quality and safety benchmark report showing a drop in performance measures over the previous three quarters. The PICU nursing director states that several PICUs have improved quality by learning from so-called high-reliability organizations. Which of the following industries are commonly cited as high-reliability organizations?



    • A.

      Coal mining


    • B.

      Commercial aviation


    • C.

      Healthcare


    • D.

      High-speed passenger railroad




Preferred response: B


Rationale


The most commonly cited examples of high-reliability organizations include U.S. Navy aircraft carrier flight deck operations, commercial aviation, nuclear power, and wilderness/forest firefighting. Whereas many healthcare organizations certainly aspire to become high-reliability organizations, there are few examples (if any) of high-reliability organizations in healthcare today.



  • 4.

    High-reliability organizations are characterized by five core principles. Which of the following is a core principle of all high-reliability organizations?



    • A.

      Commitment to resilience


    • B.

      Deference to hierarchy (“command and control”)


    • C.

      Lack of transparency


    • D.

      Preoccupation with success




Preferred response: A


Rationale


The five core principles of high-reliability organizations include deference to expertise, reluctance to simplify, sensitivity to operations (which often means greater transparency), preoccupation with failure (and learning from failures), and commitment to resilience.


Chapter 3 : Critical communication in the pediatric intensive care unit




  • 1.

    Which is an important factor in the successful implementation of huddles?



    • A.

      Designating a leader


    • B.

      Encouraging the elective participation of team members


    • C.

      Holding the huddle in a remote location


    • D.

      Performing on an ad hoc basis




Preferred response: A


Rationale


All huddles should be led by a designated leader to keep the discussions brief and focused. Mandatory participation of all team members is required for a successful huddle. Huddles should be held in a central location to encourage all team members to attend. Huddles should be incorporated in an institution’s standard work. The duration of huddles should be 10 minutes or less, whenever possible.



  • 2.

    What is the focus of team training programs?



    • A.

      Avoiding errors during training to prevent confusion


    • B.

      Developing communication strategies that promote hierarchy among team members


    • C.

      Emphasizing individual tasks, duties, and responsibilities


    • D.

      Facilitated debriefings




Preferred response: D


Rationale


Team training programs focus on facilitated debriefings to transform experiences into retained knowledge. Members are encouraged to learn all team tasks, duties, and responsibilities. Incorporating errors during training allows for the creation of contingency plans. The focus of communication strategies is to flatten hierarchy and encourage assertiveness. Closed-loop communication maintains a shared mental model among all team members.


Chapter 4 : Professionalism in pediatric critical care




  • 1.

    Which of the following best describes the Charter on Medical Professionalism?



    • A.

      It depicts the good and virtuous doctor.


    • B.

      It focuses on putting the patient first.


    • C.

      It portrays professionalism as a social contract.


    • D.

      It recognizes the patient’s right to access all medical resources.




Preferred response: C


Rationale


The charter sets forth a social contract by which professionals justify the privileges of their profession by performing as professionals. It lays out principles that underlie 10 commitments of physicians to patients and society. It is not a description of the good and virtuous doctor. All recognize that doctor by his or her behaviors toward patients and others. It views the social contract in the context of the “medical marketplace” and social and financial pressure. Although it emphasizes altruism and putting the patient first, it goes well beyond that in defining broad commitments to society. The charter recognizes the patient’s right to make informed choices among the available options, but it also lays out commitments under which the physician distributes finite resources equitably and informs patients honestly and as completely as possible so that they can choose wisely.



  • 2.

    Which is true of the Physician Charter?



    • A.

      It describes professional responsibilities.


    • B.

      It expresses ethical values.


    • C.

      It mandates professional behaviors.


    • D.

      It sets guidelines for regulatory agencies.




Preferred response: A


Rationale


The charter describes the responsibilities that fulfill the physician’s professional contract with society. It does not have the force of law and has no power to mandate professional behavior. It does not reduce physician responsibilities to guidelines for actions. It is not so much a statement of ethical values as it is a commitment to 10 practical physician responsibilities.


Chapter 5 : Leading and managing change in the pediatric intensive care unit


The following questions are based on this vignette:


You are the medical director of a “closed” PICU in which surgical patients are co-managed by the surgical and critical care services with all other medical admissions admitted to the critical care service, and consultant services do not enter patient orders without express discussion with the critical care team. One common exception to this delineation of medical team hierarchy involves the anesthesia pain service which has maintained direct management of all patient controlled anesthesia (PCA) infusions even in the PICU. In the past 3 months, (1) two new orthopedic surgeons joined the practice and have contributed to a doubling of the usual admission volume for postoperative spinal fusion admissions, (2) a new delivery system was deployed for all PCA pumps that requires a code to be entered to reprogram the PCA settings, (3) a new hospital policy now restricts prescription of additional opioid medications to only pain service physicians and nurse practitioners for all patients who have PCA orders, and (4) the pain service has lost personnel going from 2 physicians and 4 nurse practitioners providing 24/7 coverage to 1 physician and 1 nurse practitioner that provide daytime in-person coverage with nights and weekend coverage provided by the on-call anesthesia team. At the same time, patient satisfaction scores have declined for these patients, anonymous family surveys report feeling like their child’s pain was ignored in the postoperative period, and multiple adverse event reports have been submitted detailing challenging conversations and delays in patient care. Specifically, families have complained, and the bedside nursing team has reported that acute pain episodes not resolved with PCA parameters are left unaddressed for too long while the critical care team calls the pain service to adjust analgesia and add adjunctive therapies in response to breakthrough pain. Also, the PICU bedside team reports frustration that the pain service does not take into account nursing pain assessments when making decisions.



  • 1.

    You have been tasked with implementing a new care guideline that addresses the perception that there is a delay in responding immediately to patient reported pain symptoms. As you consider how to address this patient care issue, the initial approach MOST LIKELY to achieve rapid and sustained improvement in management of postoperative pain in this patient population with the LEAST amount of disruption of established practice and work flow among involved parties is:



    • A.

      Meet with the medical director of anesthesia pain service and explain that for all other consulting services on PICU patients, the PICU has direct management of all patient care. Therefore, the PICU will now assume primary responsibility of all pain management including PCA prescriptions. The pain service would be consulted only if the patient’s pain is not adequately controlled by the PICU team’s approach and at time of transfer from the PICU, at which time the pain service will assume primary responsibility for pain management as currently occurs in all other surgical inpatient admissions.


    • B.

      Request a comprehensive report from the hospital Process Improvement Committee of the past year’s posterior spinal fusion post-op PICU admissions that details patient data from the first 24 hours of PICU admission related to pain scores, total patient opioid exposure, frequency of prn doses of rescue medications for increased patient pain, and pain service documentation.


    • C.

      Meet with the PICU RN leading Process Improvement initiatives in the PICU to discuss the situation and identify multi-disciplinary and multi-professional representatives to investigate the scope of the concerns, to understand the current work flow and impacted team members, to list key drivers impacting this aspect of care, and to identify current “best practice.”


    • D.

      Develop a comprehensive re-education process targeting new PICU RNs on the physiologic and psychologic aspects of pediatric pain and nonpharmacologic interventions that have been proven to reduce or eliminate need for pharmacologic intervention.


    • E.

      Empower bedside PICU RNs to advocate for their patient and re-emphasize the role of the PICU charge RN to support and guide individual RNs through difficult situations.




Preferred response: C


Rationale


While all five answers are reasonable components of developing a new patient care guideline, only answer C describes an initial planning process that maps out a plan for gaining understanding of the scope of the challenge and for identifying needed participants in the development and implementation of the new guideline. Depending on this preliminary planning work, any of the other four approaches could be needed components of the overall strategy. However, beginning with any of the other four answers could create significant resistance to change (answer A), create significant time-intensive data collection that does not answer targeted questions (answer B), repeat training that does not address the necessary gaps in knowledge (answer D), or fail to recognize the role of other PICU team members in addressing the situation (answer E).



  • 2.

    You and your team have developed a new patient care guideline for post-op pain management for these patients. The MOST EFFECTIVE and MOST RESPONSIVE means of evaluating the initial impact of these new guidelines is to:



    • A.

      Develop data collection tools that populate quarterly reports for review.


    • B.

      Track monthly patient satisfaction scores and surveys and review the results with PICU staff.


    • C.

      Perform rapid, real-time data collection for each patient impacted by the new analgesia and solicit verbal feedback on whether or not the patient’s pain has been adequately managed.


    • D.

      Perform rapid, real-time data collection of pre-determined objective and subjective data elements and solicit input from bedside PICU team members for ways to improve the new guideline that can be immediately incorporated and evaluated after ad hoc review by a core team of identified project leaders.


    • E.

      Hold monthly meetings during which the current bedside team caring for any patients undergoing posterior spinal fusions can provide their input on how the guidelines are working.




Preferred response: D


Rationale


Rapid Plan-Do-Study-Act (PDSA) cycles allow immediate assessment of the impact of change as well as timely and responsive adjustments to new initiatives that address unanticipated consequences of the new guidelines. PDSA cycles evaluate pre-determined outcomes and promote transparency about the effort for all involved PICU team members. In order to avoid premature changes based on single instances, a core leadership team should evaluate proposed changes and adjust planned performance metrics accordingly.



  • 3.

    Since deployment 6 months ago, these postoperative analgesia management guidelines have been very successful and are being used as a model for interprofessional pain management and team communication among other surgical specialty patients. A senior anesthesiologist who started the pain service and was away on extended sabbatical over the past year returns to clinical service. When first informed, he was skeptical about the guidelines, citing 30 years of personal experience in postoperative pain management as being superior to the approach used in the guidelines. He has cancelled two previously informally scheduled meetings with you to review the new guidelines and discuss his specific concerns. In the past 2 weeks, he has become more vocal in his objections and has voiced his disagreement in front of families in the middle of morning rounds, leading to multiple complaints by PICU team member and family members. You have firsthand experience working with this physician on other successfully implemented patient care initiatives and have always found his brusque comments and opinions to be insightful and well-meaning. You believe that you have a productive, honest, and friendly relationship with him based on your prior shared successes. The BEST initial method for addressing this physician’s disruptive behavior is:



    • A.

      Meet with the hospital Chief Medical Officer and the department chair of anesthesiology to discuss this individual’s behavior and request that he be removed from participation in clinical care for these patients until he has been counseled and completed education and training on these guidelines.


    • B.

      Meet him in his office with two cups of coffee to begin an impromptu conversation to better understand the basis of his disagreements with the guidelines.


    • C.

      Hold an intervention meeting with the anesthesiologist, yourself, and one or two members of your core guideline development leadership team to acknowledge that the anesthesiologists recent behavior has been disruptive and counterproductive to your shared goals for delivering optimal patient care, to describe the ways in which the guidelines have improved patient care, acknowledge the instances in which the guidelines could still be optimized, and to ask for specific thoughts on how the guidelines can be modified to address his specific concerns and misgivings.


    • D.

      Collect statements from witnesses of his disruptive behavior to share with him and his department chair as examples of the negative impact of his actions, demand that he apologize to the PICU team, and set the expectation that he comply with the analgesia guidelines.


    • E.

      Refer the situation to the hospital professional standards committee.




Preferred response: C


Rationale


Addressing disruptive behavior directly can be uncomfortable and challenging but is essential in maintaining morale and momentum when leading and implementing change initiatives. Depending on the situation, the specific approach can be preventative and informal (answer B), accusatory (answer D), or punitive (answers A and E). However, in a situation in which the disruptive individual has cancelled previously scheduled meetings, has not commented on the strengths and weaknesses of the guideline, and is interfering with constructive interprofessional team communication and collaboration, a direct confrontation that emphasizes a shared common goal (improving patient care) and outlines unacceptable behavior that still leaves room for education (review of the guideline development process and observed outcomes) and dialogue (specific solicitation of guideline critique) can preserve relationships and avenues for partnership on future projects.


Chapter 7 : Fostering a learning healthcare environment in the pediatric intensive care unit




  • 1.

    Which of the following elements provides a foundation for a learning healthcare environment?



    • A.

      Best practice


    • B.

      Clinical research


    • C.

      Professionalism


    • D.

      Shared educational model




Preferred response: C


Rationale


The elements of professionalism, namely accountability, respect, and teamwork, need to be consistently present in order for best practice, clinical research, and shared education to occur.



  • 2.

    Which of the following accurately characterizes gender disparity in the medical workspace?



    • A.

      About 10% of female physicians have experienced sexual harassment.


    • B.

      A significant gender pay gap persists in medicine.


    • C.

      Male and female physicians are both likely to be introduced as “doctor.”


    • D.

      Women receive similar research start-up funding when compared to men.




Preferred response: B


Rationale


30% of female physicians have experienced sexual harassment. Women receive significantly less research start-up funding when compared to men. Male physicians are more likely to be introduced as “doctor,” compared to female physicians. In addition, women are less likely to be first authors in top tier journals; women are less likely to be included on expert guideline consensus panels; there are fewer women in leadership positions, even in pediatrics; and there are fewer women full professors as compared to men.



  • 3.

    Which of the following characteristics of clinical standard work is correct?



    • A.

      Amplifies occurrence of nuisance variables


    • B.

      Complicates communication among providers


    • C.

      Establishes a baseline for continuous improvement


    • D.

      Increases waste transiently with implementation




Preferred response: C


Rationale


Standardization utilizing clinical standard work facilitates identifying and eliminating waste, communicating between providers, establishing a baseline for continuous improvement, and minimizing noise/controlling for nuisance variables. Standardization represents the foundation for iterative improvement, and without standardization, measurements of improvement are not possible.



  • 4.

    Which of the following outcomes has been reported in a dose-response fashion as a function of proportional compliance with the ICU Liberation ABCDEF bundle elements?



    • A.

      Decreased incidence of anemia requiring transfusion


    • B.

      Decreased ventilator-associated lung injury


    • C.

      Improved ICU and hospital survival


    • D.

      Increased proportion of rapid eye-movement sleep




Preferred response: C


Rationale


Two independent cohort analyses demonstrated that proportional compliance with ABCDEF bundle elements resulted in significant and dose-related improvements in patient-centered, clinically meaningful outcomes such as survival, duration of mechanical ventilation, neurological organ dysfunction (i.e., delirium and coma), use of physical restraints, ICU readmission rates, and discharge disposition of ICU survivors. Anemia, ventilator-associated lung injury, and sleep quality data have not been published.



  • 5.

    Which aspect of simulation in an interdisciplinary teaching model is particularly effective?



    • A.

      Real time debriefing


    • B.

      Role playing


    • C.

      Systems thinking


    • D.

      Teamwork practicing




Preferred response: A


Rationale


All activities related to simulation-based education are important, but real-time team debriefing around critical events (doing in context) represents a particularly effective interdisciplinary (simulation) teaching modality.



  • 6.

    In addition to identification of best available evidence to support best practice, what is the other benefit of a learning healthcare environment?



    • A.

      Encouraging empiric treatment modalities


    • B.

      Facilitating craftsman care attitude


    • C.

      Promoting wellness and resiliency


    • D.

      Training basic researchers




Preferred response: C


Rationale


The second, sometimes less obvious benefit of a learning healthcare environment is promoting wellness and resiliency among critical care providers. Participation of the interdisciplinary team in shared education and research/quality improvement activities provides opportunities for critical care providers to debrief and reflect, to provide mutual support, and to reinvigorate a sense of purpose.


Chapter 8 : Challenges of pediatric critical care in resource-poor settings




  • 1.

    Which of the following is the leading cause of childhood deaths under the age of 5 years in low-middle income countries?



    • A.

      Diarrheal illnesses


    • B.

      Lower respiratory illnesses


    • C.

      Malaria


    • D.

      Neonatal problems


    • E.

      Road traffic injuries




Preferred response: D


Rationale


The majority of childhood deaths under the age of 5 years in low-and middle income countries are related to neonatal problems (34%), followed by lower respiratory (16%) and diarrheal illnesses (10%), as well as malaria (7%). Road traffic injuries play an important role in children 5 to 14 years old.



  • 2.

    You are the attending consultant in a tertiary care center in a low-middle income country. The unit is equipped with facilities for mechanical ventilation (1 available ventilator), intermittent arterial blood gases, vasopressors, intermittent blood products. You are on call, and there are two patient calls:




    • The first call is from the casualty senior resident for an 11-year-old child with chronic liver disease with pneumonia of 5 days duration with pediatric acute respiratory distress syndrome (PARDS) requiring mechanical ventilation and monitoring.



    • The second call is for a child admitted in the pediatric ward. A colleague of yours is requesting shifting this 10-year-old child with a rare genetic disorder, who he followed since birth to the ICU with septic shock, PARDS, and multiorgan dysfunction. The child is also being referred from the ministry of health.



    • Which of the following policies regarding admission to PICU are to be considered while deciding admissions in units in low resource settings?



      • A.

        Chronological order in which PICU requests are made do not matter.


      • B.

        Optimization of overall benefit that could result from use of PICU resources


      • C.

        Preference is to be given for requests made by the central and state health authorities or hospital administration.


      • D.

        The trainee can decide “who to admit” or “who not to admit’” without discussion with the attending physician.





Preferred response: B


Rationale


In resource limited countries, many children do not have access to ICUs. With a growing demand for intensive care and availability of limited resources, having policies for ICU admission therefore is of significant importance in such settings. Guidelines available for admission and discharge policies from high income countries may not be applicable to low- and middle-income countries with high burden of mortality from acute illnesses. However, the broad principles that could be adopted based on the patient population and resource availability have been discussed by Argent et al. At Red Cross War Memorial Children’s in South Africa, for example, general principles for PICU admission are the following:




  • Children will be considered for admission in terms of the chronological order in which that request is made



  • Preference will not be given to children from the Red Cross War Memorial Children’s Hospital. In fact, where possible, preference will be given to children who are referred from other areas (where they have very limited access to alternative intensive care)



  • No child will be refused admission to the PICU without a decision by the ICU attending on call at the time



  • No trainee will be asked to make a final decision about the appropriateness of PICU admission. This MUST be a consultant decision.



  • 3.

    Which of the following statements is false regarding performance of PCCM research in low resource settings (LRS)?



    • A.

      Investigators in LRS would benefit from participation in networks that include other investigators within LRS and collaborations with investigators outside of LRS.


    • B.

      Research programs based on high-income countries should lead efforts to solve problems in LRS as they have better resources.


    • C.

      Research led by investigators in LRS have informed important changes in clinical guidelines for fluid resuscitation for shock and for cost-effective delivery of noninvasive respiratory support.


    • D.

      The research agenda for LRS should be informed by research priorities identified in LRS.




Preferred response: B


Rationale


Research priorities are ideally based on indicators such as the leading causes of child morbidity and mortality and thus vary by region. Clinicians and researchers in low-resource regions should lead prioritization of the research agenda, even though there are less research resources available including training opportunities and funding. Collaboration, research network, and training opportunities in concert with investigators and programs in high-resourced regions may serve to benefit overall strengthening of research infrastructure in low-resourced regions.


Chapter 9 : Public health emergencies and emergency mass critical care




  • 1.

    Your hospital receives news from the World Health Organization and the Centers for Disease Control that there is an emerging infectious disease rapidly spreading internationally. Estimates indicate your area will start seeing cases within the week. What actions should the ICU team take to prepare?



    • A.

      Activate crisis standards of care


    • B.

      Review the incident command system with ICU leaders


    • C.

      Teach staff to provide manual ventilation


    • D.

      Utilize personal protective equipment for all patients




Preferred response: B


Rationale


Preparing for a potential pandemic should prompt each unit to review their own internal emergency plans and ensure all key staff are aware of how decisions will be made and communicated during the public health emergency (PHE). The primary way decisions are centralized and made with the best available information is the Incident Command System (ICS). ICU leaders will need to know what information they are expected to provide the incident command center and what decisions the incident commander will make versus what the local leaders will make. As the current resources are adequate to provide for the patients there is not a need to utilize crisis standards of care yet; hopefully conservation and contingency strategies will help avoid the need to utilize crisis standards of care. Personal protective equipment (PPE) should not be used for all patients as this will exhaust supplies of critical equipment. Manual ventilation should only be used as part of emergency mass critical care (EMCC) and when mechanical ventilators are not available; as such, this technique should not be a high priority for education and instead educational efforts should focus on reviewing the hospital and unit disasters plan for infectious outbreaks.



  • 2.

    The emergency department calls to alert the ICU that a local shopping center building has collapsed on a Saturday with many people inside, and they are expecting many casualties to arrive within minutes. What is the correct order of steps to take?



    • A.

      Communicate with the incident commander, send a triage team to the ED, rapid transfer/discharge of any non-ICU level patients


    • B.

      Rapid assessment of surge capacity, communicate with the incident commander, send a triage team to the ED


    • C.

      Rapid transfer/discharge any non-ICU level patients, communicate with the ICS, assemble a triage team


    • D.

      Send a triage team to the ED, communicate with the incident commander, rapid assessment of surge capacity




Preferred response: B


Rationale


In any potential surge knowing the available hospital surge capacity is a critical first step. Upon notice of an incident each unit performs a rapid assessment of which patients can be safely transferred or discharged and communicates this information succinctly to the incident commander. Units should not proactively transfer or discharge patients without word from the ICS as this could result in uncoordinated, unneeded, or inappropriate allocation of resources. After alerting the incident commander, members of the ICU designated for the triage team should proceed to the ED to aid in helping with identifying patients appropriate for rapid transfer to the ICU.



  • 3.

    A prolonged surge event due to severe influenza has caused the hospital to activate their incident command center (ICS) and utilize strategies to meet surge needs. The incident commander sends word that the ICU should begin converting OR spaces to ICU spaces, that personal protective equipment goggles should be cleaned and reused whenever possible, and that ICU staff are being asked to work overtime. What level of surge response is this?



    • A.

      Conventional


    • B.

      Contingency


    • C.

      Crisis


    • D.

      Emergency mass critical care




Preferred response: B


Rationale


Converting reasonably equipped hospital spaces to ICU space, selective reuse of supplies, and extension of existing ICU staffing pool are all contingency strategies. Conventional strategies lie in conserving resources, utilizing available staff without significantly extending staffing, substituting only where equivalent, and making use of supply caches. Crisis strategies include converting poorly equipped or non-hospital areas to ICU spaces, using non-ICU staff for ICU care, and reallocating (rationing) scarce resources. EMC refers to the global category of needing to provide ICU care to a much larger than usual number of patients.



  • 4.

    Another hospital in the area calls to request that some of their ICU level patients be moved to your hospital’s ICU. They were the hospital closest to a school bus crash and received 15 critically ill pediatric victims from the crash. Their usual ICU capacity is 16 patients, and they were at capacity before the event. Your ICU’s usual capacity is 80 patients and it is usually >90% occupied. What size of surge did the 15 victims create at the outside hospital and what size surge would your ICU have if you accepted all 15?



    • A.

      Major surge at outside hospital, minor surge for your ICU


    • B.

      Minor surge for outside hospital, usual volume for your ICU


    • C.

      Moderate surge for both


    • D.

      Moderate surge for outside hospital, minor surge for your ICU




Preferred response: D


Rationale


For a 16 bed ICU an additional 15 patients is a 94% increase, which meets the range of a moderate surge (20–100% increase from usual capacity). For an 80 bed ICU, an additional 15 patients is an 18% increase which is a minor surge (up to 20% above usual capacity). This scenario would not constitute a major surge (>100%) for either unit.


Chapter 10 : Lifelong learning in pediatric critical care




  • 1.

    You are preparing to teach a group of intensive care fellows about how to conduct an effective family meeting. Interpersonal and communication skills are an example of which of the following:



    • A.

      Competency domain


    • B.

      Easily testable skill


    • C.

      Entrustable professional activity


    • D.

      Milestone


    • E.

      Stage of skill acquisition




Preferred response: A


Rationale


The six core competency domains established by the ACGME are:




  • Practice-Based Learning and Improvement



  • Patient Care and Procedural Skills



  • Systems-Based Practice



  • Medical Knowledge



  • Interpersonal and Communication Skills



  • Professionalism




    • These six domains should be used to guide and coordinate evaluation of all residents or fellows in their development.




  • 2.

    A critical care fellow is preparing a simulation scenario to practice communication skills during cardiopulmonary resuscitation with the bedside team. This type of simulation is an example of which of the following?



    • A.

      Crew Resource Management


    • B.

      Procedural Training


    • C.

      Resuscitation Manikin Teaching


    • D.

      Standardized Role Play


    • E.

      Task Training




Preferred response: A


Rationale


Crew resource management was initiated in the aviation industry to improve safety in critical environments by focusing on interpersonal communication, leadership, and decision making.



  • 3.

    You are preparing a curriculum to teach residents about pediatric acute respiratory distress syndrome. Which of the following is an example of a method you might use that embraces adult learning principles?



    • A.

      Behavior management strategy


    • B.

      Didactic lecture using PowerPoint


    • C.

      See one, do one, teach one


    • D.

      Small group learning


    • E.

      Videotaped lecture with question and answer period




Preferred response: D


Rationale


Adults are self-directed and autonomous. They learn best when they are active participants in the learning process and are allowed to practice newly acquired skills and concepts. As a consequence, education is typically most effective when programs facilitate self-learning with specific goals of acquiring practical information. Efforts to be inclusive of curricular methods that support adult learning principles are occurring in undergraduate, graduate, and continuing medical education. Problem-based and small group learning, flipped classrooms, and simulation exercises allow many venues for reaching learners in different ways.



  • 4.

    You are preparing to take the American Board of Pediatrics Critical Care Subspecialty examination. Which of the following is not required to take the examination?



    • A.

      A job as a pediatric intensivist


    • B.

      Evidence of meaningful scholarly project during pediatric critical care fellowship


    • C.

      Initial certification in General Pediatrics


    • D.

      Successful completion of a Critical Care Medicine fellowship program


    • E.

      Valid unrestricted license to practice




Preferred response: A


Rationale


To qualify for the ABP subspecialty examination, applicants are required to have a valid unrestricted allopathic and/or osteopathic license to practice, initial certification in general pediatrics, and successful completion of critical care medicine training in a program accredited by the ACGME. The applicant must also provide evidence of meaningful scholarship during training which can include research, an educational project, or a quality project.



  • 5.

    You are planning to teach a group of critical care fellows how to manage a patient with acute respiratory distress syndrome. Which of the following instructional techniques takes best advantage of the qualities of the adult learner?



    • A.

      Attending a webinar about ventilation strategies followed by a short quiz


    • B.

      Engaging in a detailed review of real patient cases followed by discussion of ventilation strategies


    • C.

      Attending a lecture with a PowerPoint presentation followed by a quiz


    • D.

      Reading a research article on ARDS and then discussing the merits of the study




Preferred response: B


Rationale


Adult learners thrive with active, engaged learning environments. Although all options have the potential for active, engaged learning, response B, review of real patient cases followed by discussion of care strategy, is an example of problem-based learning and requires active engagement by the learners. It is also clearly related to their experience.



  • 6.

    Which of the following is an example of an entrustable professional activity (EPA)?



    • A.

      Identifies subtle or unusual physical exam findings


    • B.

      Recognizes a patient requiring urgent or emergent care and initiates evaluation and management


    • C.

      Routinely provides vital signs and other laboratory data on rounds


    • D.

      Shows self-awareness of one’s own knowledge, skill, and emotional limitations that leads to appropriate help-seeking behaviors




Preferred response: B


Rationale


Domains of competence refer to the six groups of physician subcompetencies defined by ACGME, including medical knowledge, patient care, practice-based learning and improvement, interpersonal and communication skills, professionalism, and systems-based practice. Milestones are the developmental levels of knowledge, skills, and attitudes for each subcompetency and describe the progression of a physician’s ability. Entrustable professional activities (EPAs) are tasks or responsibilities that trainees are entrusted to perform unsupervised once they have attained sufficient competency. EPAs have been developed with patient safety in mind, but they also provide a framework for curricular development and lifelong progression of learning. Response B, “recognizes a patient requiring urgent or emergent care and initiates evaluation and management,” is a discrete, concrete skill that can be observed and entrusted.


Chapter 11 : Essential concepts in clinical trial design and statistical analysis




  • 1.

    In a randomized controlled trial to evaluate the effects of prone positioning compared to supine positioning in pediatric acute respiratory distress syndrome (PARDS), bedside nurses and treating physicians are asked to complete daily questionnaires classifying whether the patient has had clinical improvement, stability, or deterioration. This study is at risk of what kind of bias?



    • A.

      Ascertainment bias


    • B.

      Bias due to loss of data


    • C.

      Lead time bias


    • D.

      Selection bias




Preferred response: A


Rationale


Ascertainment bias exists when those responsible for determining outcomes are aware of treatment group assignment. In this study, it would not be possible to blind bedside nurses and treating physicians to study group assignment, and knowledge of the treatment group assignment might influence their responses regarding illness trajectory. Bias due to loss of data occurs when substantial patient outcome data is missing, or when data is missing related to treatment arm assignment. Lead time bias is a risk in screening intervention studies, where time to an outcome may increase only because an illness was identified earlier than it would have naturally presented, but the screening intervention did not actually impact the occurrence of the outcome. Finally, selection bias occurs when certain patients are preferentially enrolled in the treatment or control arm of a study, creating imbalance between the study arms.



  • 2.

    A randomized controlled trial of a biologic immunomodulating drug in patients with severe sepsis decreased mortality to 7% among patients receiving treatment, compared to 21% among patients in the control arm. Among patients with similar baseline risk of mortality, what is the number needed to treat (NNT) with this medication to save one life?



    • A.

      3


    • B.

      3.5


    • C.

      7


    • D.

      14


    • E.

      Not enough information to answer




Preferred response: C


Rationale


NNT is the inverse of the absolute risk reduction (ARR) × 100, or 1/(risk untreated – risk treated ) × 100. In this case, ARR = 21% − 7% = 14%, and 100/14 ∼7. The NNT can be adjusted for baseline risk; among septic patients with twice the baseline risk of mortality (∼42%), the NNT would be halved, at 3.5.



  • 3.

    An interventional study for patients with traumatic brain injury randomizes half to receive an early physical therapy and mobilization bundle, compared to standard of care. What is the most appropriate statistical test to compare the hospital length of stay between the two groups?



    • A.

      ANOVA


    • B.

      Chi square test


    • C.

      T-test


    • D.

      Wilcoxon rank-sum test




Preferred response: D


Rationale


Hospital length of stay is a good example of a continuous variable with a skewed distribution, as most patients have short lengths of stay, but a handful of patients have very long lengths of stay. A nonparametric test, like the Wilcoxon rank-sum test , is most appropriate for comparing the median of this kind of outcome measure between two groups. A t-test is appropriate for comparing the mean of a normally distributed outcome measure between two groups (e.g., low-density lipoprotein cholesterol.) ANOVA is used to compare means of a normally distributed outcome measure between multiple groups. Finally, a chi square test is used to compare frequencies in a 2 × 2 table, typically used for binary outcomes.



  • 4.

    An investigator wishes to conduct a randomized trial examining continuous renal replacement therapy (CRRT) for fluid overload in patients with severe acute respiratory distress syndrome (ARDS). Which of the following is true for this study?



    • A.

      If the study results demonstrate a relative risk of mortality of 0.8 in the treatment group, with a 95% CI of 0.55-1.05 and a p value of 0.12, then we can conclude that CRRT is not effective in improving outcomes for patients with severe ARDS.


    • B.

      Patients who are randomized to CRRT but do not receive it because their hemodialysis catheter malfunctioned should be analyzed along with patients who are in the control group.


    • C.

      The outcomes should be compared based on original treatment group assignment, regardless of what therapy the patient actually received.


    • D.

      The outcomes should be compared based on what therapy the patient actually received, regardless of original treatment group.


    • E.

      This study cannot be conducted because it is impossible to blind patients and providers to the treatment group assignment.




Preferred response: C


Rationale


Intention to treat analysis preserves the benefits of randomization, and patients who are randomized to a treatment group should be analyzed along with that group. It is legitimate to conduct studies in which blinding is not possible; however, study staff who ascertain subjective outcomes should be blinded to treatment group assignment. These study results suggest that there may be a treatment effect—the majority of the 95% confidence interval lies below 1.0—but, there is a reasonable probability (12%) that this estimate of effect was found by chance. It may be that there is no effect, but it may also be that this study was underpowered to identify this 20% decrease in mortality.


Chapter 12 : Prediction tools for short-term outcomes following critical illness in children




  • 1.

    A 4-year-old previously healthy boy is admitted to the intensive care unit with septic shock. He is intubated on arrival to the unit, started on broad spectrum antibiotics, and fluid resuscitated. He has ongoing hypotension and multiple vasoactive infusions are initiated. His laboratory studies demonstrate evidence of disseminated intravascular coagulation (DIC) and his most recent arterial blood gas analysis demonstrates a severe metabolic acidosis with a lactate of 8. The resident on call tells you that his Pediatric Risk of Mortality (PRISM) score two hours after admission is 19. The most correct interpretation of the PRISM score in this patient is:



    • A.

      His risk of mortality during this admission is 19%.


    • B.

      His risk of mortality in the next two hours is 19%.


    • C.

      It cannot be interpreted because he has not yet been fully resuscitated.


    • D.

      It cannot be interpreted because he has not yet been admitted long enough.


    • E.

      It cannot be interpreted because PRISM does not apply to individual patients.




Preferred response: E


Rationale


PRISM scores are not designed to be interpreted for an individual patient, but instead provide a mechanism by which case-mix can be measured over a large group of patients. PRISM scores are calculated during the first 4 hours of a patient’s admission to the ICU and include physiologic and laboratory data for 2 hours prior to and 4 hours after admission. This time limitation is intended to more accurately represent a patient’s presenting physiologic status, as opposed to the provision of intensive care.



  • 2.

    As part of a quality improvement initiative, a colleague has created a model to examine patients at risk of developing pressure injuries while in the intensive care unit. She reports in a divisional research meeting that the discrimination of the model is 0.92. Which of the following is the most correct interpretation of this value?



    • A.

      The model’s overall sensitivity is 92%.


    • B.

      The model’s overall specificity is 92%.


    • C.

      The positive predictive value of the test is 92%.


    • D.

      92% of variation in a patient’s likelihood of developing a pressure injury is explained by the model.


    • E.

      92% of patients with pressure injuries will meet all the components of the model.




Preferred response: B


Rationale


Discrimination is a model’s ability to correctly differentiate patients with a specified outcome from those without the outcome. This is in contrast to calibration, which is the model’s ability to predict overall event rates. The model’s discrimination is often reported as the C statistic, which represents the average sensitivity of the model over the range of possible specificity values.



  • 3.

    A fellow is planning a research project to study patient risk factors for mortality in patients with traumatic brain injury. She is hoping to include patients from multiple centers and would like your advice about how to control for variation in risk between hospitals. Which of the following is most useful for the research team to assess?



    • A.

      The sample size from each hospital.


    • B.

      The standardized mortality ratio for each hospital.


    • C.

      The hospital length of stay for each included patient.


    • D.

      The yearly number of traumatic brain injury patients admitted to each hospital.


    • E.

      The presence or absence of neurosurgical services at each hospital.




Preferred response: B


Rationale


Standardized mortality ratios represent the ratio of observed to expected mortalities and provide a mechanism by which researchers can control for case mix variation between institutions. This can help to isolate patient characteristics, which are of interest to the fellow, from systems-level characteristics. While the other choices represent important considerations, the standardized mortality ratio is most important to allow for control.



  • 4.

    A 4-month-old former 35-week gestational age girl is admitted to the intensive care unit for hypoxic respiratory failure requiring mechanical ventilation. She was intubated at an outside hospital and was started on broad spectrum antibiotics for concern for pneumonia with an elevated white blood cell count (WBC) and infiltrate on chest radiograph. As part of her admission process, the bedside nurse completes a worksheet with information for the Pediatric Risk of Mortality (PRISM) score as well as the Paediatric Index of Mortality-3 (PIM3) score. Which of the following pieces of information will be included in the PIM3 but not PRISM?



    • A.

      The pH from an arterial blood gas obtained one hour after admission.


    • B.

      The measured systolic blood pressure at admission.


    • C.

      The results of pupillary reflex testing.


    • D.

      The fact that the child is undergoing mechanical ventilation.


    • E.

      The measured WBC obtained at the outside hospital one hour prior to admission.




Preferred response: D


Rationale


Both PIM3 and PRISM include evaluation of physiologic and clinical data for a patient but the methods differ slightly in the amount of included data, the observation period, and inclusion of nonphysiologic data. Specifically, PIM3 includes mechanical ventilation within an hour of admission, while PRISM includes more laboratory values. Both scoring systems include evaluation of systolic blood pressure and pupillary reflexes.



  • 5.

    A researcher develops a model to predict the risk of patients admitted to the PICU of developing a deep venous thrombosis based on a number of demographic and clinical factors. The model is developed on 1000 patients with an area under the receiver operating characteristic curve (AUC) of 0.8. The model is subsequently validated on a separate dataset of 1000 patients with an AUC = 0.7. What term best describes the model’s performance as quantified by the AUC?



    • A.

      Calibration


    • B.

      External validity


    • C.

      Discrimination


    • D.

      Internal validity




Preferred response: C


Rationale


Discrimination is the accuracy of a model in differentiating outcomes groups and is most often assessed by the AUC. External validity is the extent to which the results of a study can be generalized to other situations and other people. External validity is typically assessed through the process of study replication. Calibration refers to the ability of a model to assign the correct probability of outcome to patients over the entire range of risk prediction. The most accepted method for measuring calibration is the Hosmer-Lemeshow goodness-of-fit test. Internal validity relates to the extent to which a causal conclusion based on study findings is warranted and is assessed by the degree to which the study design minimized systematic error or bias. The precision of a measurement system is the degree to which repeated measurements under unchanged conditions show the same results.



  • 6.

    A previously healthy 7-year-old male is admitted to the PICU of a tertiary care children’s hospital in septic shock secondary to lobar pneumonia. He is tachycardic (heart rate, 130 beats per minute) and tachypneic (respiratory rate, 55 breaths per minute) and undergoes intubation of the trachea within 30 minutes of PICU admission for respiratory failure. Arterial blood gas reveals pH = 7.2, Paco 2 = 40 mm Hg, Pao 2 = 80 mm Hg, and HCO 3 = 12 mEq/L on F io 2 of 0.6. Coagulation studies reveal an elevated INR = 1.8. Which scoring system is most appropriate to assess this patient’s physiologic status on the initial day of PICU admission?



    • A.

      PELOD


    • B.

      PRISM III


    • C.

      SNAPPE-II


    • D.

      STS-EACTS score




Preferred response: B


Rationale


CRIB II and SNAPPE-II are scoring systems for neonates. STS-EACTS score was developed for patients with congenital heart disease. PELOD is a scoring system designed to quantify the degree of organ dysfunction in PICU patients and correlates well with mortality. PELOD, however, is calculated based on the greatest degree of physiologic dysfunction to occur at any point during the entire PICU admission, not just the initial day of admission. PRISM III is a physiology-based acuity score that is calculated in the interval from the 2 hours preceding PICU admission to 4 hours following PICU admission.



  • 7.

    You are asked to accept the transfer of a 2-year old child with respiratory failure secondary to viral pneumonitis from a rural emergency department (ED). The child was seen and discharged from that ED 2 days ago. This morning, her parents awoke to her loud breathing and called emergency medical services. The emergency medical technicians assessed the child, determined that she had respiratory failure, stabilized her airway, provided bag-valve-mask ventilation with oxygen, and provided respiratory treatments during transport to the local ED. They notified the ED of the child’s condition. The ED is performing additional steps for stabilization, along with some diagnostic tests, including a chest radiograph and basic laboratory studies. The ED has explained to the family that their child will need to be transported to the local children’s hospital, and someone from the ED is calling you for a helicopter transport. This scenario demonstrates appropriate system design for the critically ill child around which of the following issues?



    • A.

      Access


    • B.

      Cost


    • C.

      Outcome


    • D.

      Process




Preferred response: A


Rationale


Systems are designed to facilitate easy access to services for those who need them. Access (response A) describes organized and integrated care for critically ill children and their families who are in need of tertiary pediatric services. Cost (response B) is an important element of healthcare delivery that focuses on the monetary resources devoted to patient care. However, economic terms are not discussed in this scenario. Research is an important activity in generating new knowledge, but this scenario is concerned with the delivery of care to an individual patient. Although the child ultimately may be enrolled in a research project or database evaluating outcomes, research is not the primary focus of her care. Process measures (response D) are those that occur between patients and providers or between various providers. Although a number of clinical processes have occurred in the scenario, the scenario also includes a number of interactions between other structural components, such as institutions and emergency services, which establish an organized hierarchy of interactions for the delivery of care. The final outcomes of care from the ICU perspective (response C) are not discussed in the scenario. Although the child was effectively rescued and delivered to the rural ED and a number of intermediate outcomes may be described, the final outcomes in terms of vitality, economics, or morbidity are yet to be determined. Therefore, the most appropriate answer is A.



  • 8.

    After the first ICU day, the child’s PRISM III score is 8. What should you do?



    • A.

      Ignore the score for today.


    • B.

      Remove the child from mechanical ventilation and provide comfort care.


    • C.

      Rescore the patient using the Pediatric Index of Mortality (PIM).


    • D.

      Recognize that this score is correlated with a population risk of mortality.




Preferred response: D


Rationale


The PRISM score is a physiologically based severity of illness measure calculated from variables determined within the first 24 hours of care. It is helpful for understanding the risks for mortality and length of stay for populations of patients with a given severity of illness. However, the score cannot be applied for prognostication at the level of individual patients; hence it should not be used to guide clinical decisions as suggested in response B. Repetitive scoring using PRISM during hospitalization (response A) has not been validated. Using the PIM (response C), which captures care at the point of admission rather than at 24 hours, provides a different measure of severity that also can be compared across populations. Importantly, a bias exists in relation to the inclusion of mechanical ventilation as a predictor variable in PIM. Therefore the most appropriate answer is D.


Chapter 13 : Pediatric critical care transport




  • 1.

    What is the most common mode of inter-facility transport for pediatric patients?



    • A.

      Ferry


    • B.

      Fixed-wing


    • C.

      Ground


    • D.

      Rotor-wing




Preferred response: C


Rationale


The advantages of ground transport include easier and direct access to referring and receiving locations, lower cost, and ability to respond in most weather conditions. Circumventing traffic rules via use of lights and sirens does not improve the outcomes of pediatric transports and significantly increase the risk of an accident. Rotor-wing (helicopter) transport may be faster than ground transport for patients located 45–60 miles away from admitting institution, particularly in geographically challenging areas such as waterways, and can help to minimize out-of-hospital time. Fixed-wing transports are typically reserved for long-distance flights and have the ability for cabin pressurization. Disadvantages of both rotor-wing and fixed-wing transports are limited space and high cost.



  • 2.

    A pediatric patient is intubated for acute respiratory failure secondary to pneumonia at a critical access hospital and now requires rotor-wing transport to the nearest pediatric intensive care unit for ongoing care. The referring facility is located at an altitude of 718 ft above sea level where the atmospheric pressure is 741 mm Hg. The flight path will take the patient over a mountain pass with an unpressurized cabin altitude equivalent to 5100 ft above sea level and an atmospheric pressure of 630 mm Hg. If the endotracheal tube cuff is inflated with 2 mL of air prior to departure from the referral facility, what is the expected volume of air in the cuff when at the highest altitude?



    • A.

      1.5 mL


    • B.

      2.4 mL


    • C.

      3.0 mL


    • D.

      3.2 mL




Preferred response: B


Rationale


Boyle’s law (P 1 V 1 = P 2 V 2 ) describes the relationship between pressure and volume of a gas in an enclosed space at a constant temperature. Given that this patient will be rising in altitude, atmospheric pressure is expected to decrease with increasing height. Any gases trapped in enclosed spaces are expected to expand proportionately to any decreases in atmospheric pressure. Substituting the known pressures and volumes into the equation allows for calculation of the unknown cuff volume at the highest altitude:


(741×2=630×V2)V2=2.4mL
(741×2=630×V2)V2=2.4mL


  • 3.

    According to the Emergency Medical Treatment and Labor Act (EMTALA) established by the COBRA legislation, an appropriate transfer criterion includes which of the following?



    • A.

      The referring hospital must provide care and stabilization within its ability.


    • B.

      The referring physician certifies that the medical benefits expected from the transfer outweigh the risks.


    • C.

      The referring physician consents to transfer after being informed of the risks of transfer.


    • D.

      The receiving facility must have available space and qualified personnel and agree to accept the transfer.




Preferred response: B


Rationale


EMTALA delineates rules for inter-facility transfer. Appropriate transfers must meet the following criteria: (1) the transferring hospital must provide care and stabilization within its ability; (2) the referring physician certifies that the medical benefits expected from the transfer outweigh the risks; (3) the patient consents to transfer after being informed of the risks of transfer; (4) the receiving facility must have available space and qualified personnel and agree to accept the transfer; (5) copies of medical records and imaging studies should accompany the patient; and (6) the inter-facility transport must be made by qualified personnel with the necessary equipment.



  • 4.

    Which of the following characterizes a typical skilled inter-facility pediatric transport team in comparison to care providers at a referring hospital?



    • A.

      Administration of drugs not typically available at non-tertiary centers


    • B.

      Aggressive, early institution of simple therapies


    • C.

      Availability of superior airway and breathing support modalities


    • D.

      Superior diagnostic capabilities




Preferred response: B


Rationale


For most pediatric critical illnesses, definitive care, ideally beginning with a skilled inter-facility transport team, does not involve miracle drugs or technologies but rather the early, aggressive administration of simple therapies. Therapy that includes timely initiation of resuscitation fluids, early administration of inotropes (frequently via peripheral intravenous or intraosseous catheters), and early antibiotic therapy can improve outcomes.


Chapter 14 : Pediatric vascular access and centeses




  • 1.

    Which of the following is a recommended practice to reduce catheter-related blood stream infections (CRBSI) from central venous catheters (CVC)?



    • A.

      Full barrier precautions at the time of insertion


    • B.

      Giving all intravenous antibiotics through the catheter


    • C.

      Povidone-iodine ointment applied to insertion site daily


    • D.

      Vancomycin containing heparin flushes




Preferred response: A


Rationale


CRBSI is the most common complication related to CVCs. In children, the location of the insertion site is not related to infection risk. The risk of infection is decreased by the use of a bundle of practices during insertion and ongoing maintenance of the CVC. The insertion bundle includes strict maximal sterile barrier precautions and aseptic technique. Dressing changes with chlorhexidine skin prep, minimizing catheter access, and daily assessment of the need of the catheter are all recommended as a part of CVC maintenance. Antimicrobial-impregnated catheters may decrease the risk of catheter-related infection, but more pediatric studies are needed.



  • 2.

    Which of the following is the most common complication of intraosseous (IO) infusion?



    • A.

      Fat embolus requiring mechanical ventilation


    • B.

      Fracture requiring internal fixation


    • C.

      Hypercalcemia from bone demineralization


    • D.

      Infiltration of fluids into the surrounding tissues




Preferred response: D


Rationale


Significant complications of IO insertion and infusion are rare. The most common complication is extravasation of fluid. The causes of extravasation include incomplete penetration of the bony cortex, movement of the needle such that the hole is larger than the needle, dislodgment of the needle, penetration of the posterior cortex, and leakage of fluid through another hole in the bone, such as a previous IO site or fracture. Extravasation of a small amount of fluid is usually not problematic, but with larger volumes, compartment syndrome can develop and may require fasciotomy and even amputation. Use of the IO line for prolonged periods or with pressure bags increases the risk for this complication. If extravasation occurs, the needle should be removed, and the extremity diligently observed for signs of compartment syndrome. Experience to date suggests that the complications of the new mechanical insertion devices are similar to manual IO needle use.


Other rare complications include infection and bone fracture. Osteomyelitis, cellulitis, and sepsis have been reported in conjunction with IO infusion. Risk for infection is increased when IO access is prolonged, and these devices are used in patients with bacteremia.



  • 3.

    Ultrasound guidance for needle pericardiocentesis should be standard practice for which of the following indications?



    • A.

      Cardiac tamponade with ongoing cardiopulmonary resuscitation


    • B.

      Elective pericardiocentesis with idiopathic pericarditis


    • C.

      Penetrating trauma of the right ventricle


    • D.

      Second procedure after a failed blind, landmark-based attempt




Preferred response: B


Rationale


Except in life-threatening tamponade, ultrasound imaging should be used to improve success and reduce complications. Drainage of a pericardial effusion due to any cause is absolutely indicated when cardiac tamponade is present. Often drainage is recommended if the effusion is large, even in the absence of tamponade, for diagnosis and fluid removal. For small effusions, pericardiocentesis may be indicated for diagnosis alone. With purulent pericarditis, open surgical drainage may be more effective because of the difficulty in draining purulent exudate. Traumatic pericardial effusions secondary to penetrating trauma often require surgical drainage of the blood, because tamponade is common. Pneumopericardium secondary to pulmonary air leaks in mechanically ventilated patients is usually well tolerated hemodynamically but may require drainage, especially in small infants, because of the development of tamponade.


There is no absolute contraindication to pericardiocentesis in an emergency situation. The presence of aortic dissection or myocardial rupture is considered a major contraindication. The presence of a bleeding diathesis or coagulopathy is another contraindication. Open drainage is preferred over closed drainage when the patient has traumatic tamponade and is in cardiac arrest. When the effusion is loculated in a location not easily reached via the subxiphoid approach, needle pericardiocentesis is contraindicated because the risk of complications increases, while the possibility of successful drainage is low.



  • 4.

    Which of the following is most consistent with an exudative pleural effusion?



    • A.

      Pleural to serum lactate dehydrogenase (LDH) ratio < 0.4


    • B.

      Pleural fluid LDH < the upper limit of normal serum LDH value


    • C.

      Pleural to serum protein ratio > 0.5


    • D.

      Pleural fluid WBC count <5% of peripheral WBC count




Preferred response: C


Rationale


Analysis of pleural fluid is separated into two basic diagnostic categories: exudates and transudates. Transudates arise from imbalances of hydrostatic or oncotic pressures, such as seen in congestive heart failure or nephrotic syndrome. Exudates can be caused by a variety of mechanisms, most commonly from pleural and lung inflammation or impaired lymphatic drainage. The criteria used to distinguish between the two have evolved, but are rooted in Light’s Criteria Rule (see below). The updated combination of two or more of these criteria increases the diagnostic sensitivity for the rule. More recent diagnostic rules, including the “two-test rule” and “three-test rule,” include pleural fluid cholesterol level greater than 45 mg/dL and do not require concomitant serum levels to be obtained.


Additional pleural fluid studies should be sent to aid in diagnosis, especially fluid for culture, cell count with differential, and cytology. Low pleural glucose (<60 mg/dL) and pleural pH (<7.3) with very elevated nucleated cell counts (>50,000/mL) are highly suggestive of empyema. Elevated pleural triglyceride levels (>110 mg/dL) and lymphocyte predominance suggest chylothorax, while triglycerides <50 mg/dL effectively rule it out. Elevated amylase suggests pancreatitis or esophageal rupture. Advances in polymerase chain reaction (PCR) technology allow for rapid and accurate diagnosis of viruses and bacteria in pleural fluid.


Light’s criteria rule for diagnosing exudative effusions are as follows: If at least one of the following present, effusion defined as exudate:




  • Pleural to serum protein ratio >0.5



  • Pleural to serum lactate dehydrogenase (LDH) ratio >0.6



  • Pleural fluid LDH more than twice the upper limit of normal serum LDH value





  • 5.

    Which of the following is suggestive of spontaneous bacterial peritonitis when evaluating fluid obtained from a paracentesis?



    • A.

      Ascitic fluid glucose 22 mg/dL with serum glucose 110 mg/dL


    • B.

      Multiple organisms in the culture


    • C.

      Neutrophils ≥250/mm 3


    • D.

      Total protein 2.2 g/dL




Preferred response: C


Rationale















Condition Clinical Characteristics Laboratory Findings
Spontaneous bacterial peritonitis Cloudy or turbid
Gram stain positive <10%, cultures may be negative, single organism
Neutrophils >250/mm 3
Total protein <1 g/dL
LDH and glucose similar to serum




  • 6.

    Initial management of limb ischemia after arterial catheterization should include which of the following?



    • A.

      Aspirin and local application of nitroglycerin paste


    • B.

      Catheter removal and systemic heparin anticoagulation


    • C.

      Local infusion of alteplase


    • D.

      Thrombectomy within 4 hours




Preferred response: B


Rationale


Limb ischemia that develops after arterial catheterization first requires immediate catheter removal. If ischemia does not rapidly resolve and no contraindications to anticoagulation exist, heparinization should be considered. In larger vessels, thrombectomy or local infusion of thrombolytic agents such as alteplase may be considered in consultation with vascular surgeons and interventional radiologists.



  • 7.

    During an ultrasound-guided pericardiocentesis, saline microbubble contrast (saline solution in a syringe that has been agitated) injected through the introducer needle demonstrates the appearance of contrast in the left ventricle. The most appropriate next step is:



    • A.

      Alerting a cardiovascular operative team for emergency open pericardiotomy


    • B.

      Catheter placement, followed by high-resolution computed tomography coronary arteriography


    • C.

      Insertion of the guidewire


    • D.

      Removal of the needle




Preferred response: D


Rationale


Several techniques are helpful to determine if blood obtained during pericardiocentesis is of cardiac or pericardial origin. One technique involves injection of small amounts of saline microbubble contrast (saline solution in a syringe that has been agitated) through the introducer needle while imaging with echocardiography. If contrast bubbles are seen in the heart, then the tip of the needle is intracardiac and should be removed. If bubbles appear in the pericardial sac, then the needle is appropriately placed in the pericardium.


Chapter 15 : Ultrasonography in the pediatric intensive care unit




  • 1.

    Which of the following statements is true regarding ultrasound guidance by intensivists for vascular access in children?



    • A.

      It is not useful for the placement of umbilical arterial or venous catheters.


    • B.

      It should be performed using active (dynamic) ultrasound guidance, which is superior to physically marking the vessel location preprocedure (static guidance).


    • C.

      It should be performed with a transverse visualization of the vessel, as this is demonstrably superior to needle insertion in the longitudinal plane.


    • D.

      It sufficiently increases safety in multiple studies of femoral and subclavian central venous catheter insertion to be considered the standard of care.




Preferred response: B


Rationale


Placement of central venous catheters using active ultrasound guidance has been demonstrated to be superior to marking position prior to the procedure. Generalizable studies that demonstrate a significant safety benefit for subclavian vein central venous cannulation under ultrasound in children remain lacking. Advantages of ultrasound use for arterial catheter and umbilical vein catheter placement have been described. There have not been sufficient studies to determine whether transverse or longitudinal guidance approaches for vascular access are advantageous.



  • 2.

    Which of the following statements is true regarding cardiac ultrasonography by intensivists in children?



    • A.

      It demonstrates good concordance with studies performed and interpreted by expert echocardiographers.


    • B.

      It is useful in cardiac arrest to determine the likelihood of failed return of spontaneous circulation (ROSC) if the heart is not moving.


    • C.

      It is not affected by high intraabdominal pressure.


    • D.

      When used for determining intravascular volume status, identical criteria for IVC measurement should be used for mechanically ventilated and spontaneously breathing patients.




Preferred response: A


Rationale


Cardiac ultrasonography by pediatric intensivists and emergency medicine physicians has demonstrated good concordance in terms of both interpretation and image acquisition with ICU ultrasound equipment. Though adult cardiac arrest studies have demonstrated cardiac standstill may predict failure to achieve ROSC, recovery of cardiac function in children on ECMO initiated during CPR has also been described. Increased intrathoracic pressure with mechanical ventilation affects the measurement of the inferior vena cava (IVC) for volume status assessment as well. Accordingly, heart position is also affected by elevated intrathoracic or intraabdominal pressures due to position of the diaphragm and lungs in these clinical scenarios.



  • 3.

    What is true regarding pulmonary ultrasound in critically ill children?



    • A.

      It demonstrates reduced visibility of lung parenchyma and pulmonary vascular structures as consolidation progresses.


    • B.

      It is affected by changes in body composition, development, and size throughout the growth of a child.


    • C.

      It is prone to artifactual findings, which should be ignored during interpretation of the image.


    • D.

      It should be performed with a phased array transducer.




Preferred response: B


Rationale


The appearance of the lung changes throughout pediatric development as infant body water decreases and bones ossify. Early in life, intrathoracic structures may be more visible because of less impedance from lung and bony structures. Through late childhood into adolescence, ongoing growth affects available windows and the depth of visualization in patients. A phased array transducer can visualize areas within the thorax and the pleural line; however, this is not optimal for visualizing pleural sliding because the field of view with this type of sector transducer is narrow near the skin. Artifactual pulmonary ultrasound findings, such as B- and Z-lines, have been described as useful for assessing pediatric pulmonary pathology and should be incorporated into the assessment. Consolidation of lung parenchyma increases visualization of structures due to less interference from air as it is excluded from the tissue. B-lines are not apparent in pneumothorax due to separation of the visceral pleura from the chest wall.


Chapter 16 : Patient- and family-centered care in the pediatric intensive care unit




  • 1.

    High quality and collaborative communication includes:



    • A.

      Avoidance of conflict


    • B.

      Decreased used of empathetic statements


    • C.

      Explicit support of family decision making


    • D.

      Guarantee of positive outcome


    • E.

      Higher proportion of physician speech




Preferred response: C


Rationale


Explicit support of family decision-making, higher proportion of family speech, increased use of empathetic statements and expressing nonabandonment are all elements of high quality communication that is associated with increased patient survival, family and patient satisfaction, and is protective against physician and nursing burnout. Conflict is abundant in clinical practice. It may be unavoidable. Rather than side-stepping conflict, collaborative communication helps physicians identify conflict as an opportunity to develop a more complete understanding of the family’s differing perspectives.



  • 2.

    Parental presence during invasive procedures and cardiopulmonary resuscitation has been shown to:



    • A.

      Complicate the ability of clinicians to teach trainees procedural skills


    • B.

      Increase the likelihood of adverse events


    • C.

      Increase parental emotional lability


    • D.

      Negatively impact physicians’ technical performance


    • E.

      Reduce parental anxiety surrounding their child’s care




Preferred response: E


Rationale


Parental presence during invasive procedures and cardiopulmonary resuscitation is associated with improved satisfaction, better understanding, reduced anxiety, better coping, more emotional stability, and improved adjustment to a child’s death. Parental presence during procedures or resuscitation has not been associated with negative impacts on technical performance, ability to teach, adverse events, or clinical decision making.



  • 3.

    In general, what is the primary cause of stress in parents related to the admission of their child to an ICU?



    • A.

      They are concerned about the cost of care.


    • B.

      They are afraid their child will die.


    • C.

      They are stressed about being separated from their child.


    • D.

      They are concerned their child will not be the same after leaving the hospital.




Preferred response: C


Rationale


The admission process can be frightening for the parents and child, especially when the admission is emergent or unplanned. Every effort should be extended to help the parents acclimate to the new environment; they should be treated with compassion and courtesy, and time should be taken to meet their needs. Parents report a loss of control, which can be unbearable when they are separated from their ill child. To support the child and the parents, caregivers should invite the parents to be part of the admission process and enable them to remain with their child to the extent possible.


Chapter 17 : Pediatric critical care ethics




  • 1.

    The use of substituted judgment is most appropriate for decision-making on behalf of which of the following patients?



    • A.

      18-month-old male with severe ARDS on ECLS


    • B.

      7-year-old female with an accidental ingestion


    • C.

      13-year-old male with 60% total body surface area burns who has been sedated and intubated since his accident


    • D.

      16-year-old female with cystic fibrosis end-stage lung disease who is ventilated and sedated




Preferred response: D


Rationale


The substituted judgment standard applies to situations in which the surrogate decision makers know what the patient would have wanted. Very young children are unable to communicate what they want. Even older children often have not have developed the ability to consider and verbalize their preferences about complex medical decisions prior to becoming incapacitated. Therefore, utilization of the best interest standard (as opposed to substituted judgement) as the basis of surrogate decision-making is most common in pediatrics. In situations where an older adolescent (commonly living with chronic medical conditions) has considered and conveyed their wishes to adults prior to the losing the ability to participate in decision-making, substituted judgment is appropriate to utilize in making surrogate medical decisions. In all situations where the wishes of the patient are unknown, the best-interest standard is the most appropriate decision-making tool.



  • 2.

    Which of the following statements regarding adolescent decision-making is true?



    • A.

      A 15-year-old patient can seek medical care for reproductive and mental health needs without parental consent.


    • B.

      Adolescents are typically not developmentally ready to participate in discussions about their medical care.


    • C.

      Adolescents are legally required to provide assent as part of the informed consent process.


    • D.

      Pediatric patients designated as mature minors are still required to have adult caregiver consent for significant medical decisions.




Preferred response: A


Rationale


In pediatrics, parents or legal guardians must provide informed consent for non-adult children. If children are 7 years of age or older, their informed assent is desired but not legally required. However, there are exceptions. Adolescents who qualify as mature minors can provide consent for themselves. Similarly, all states allow adolescents to provide consent for specific healthcare needs like reproductive health and mental health services after a certain age (usually 14 or 15). Regardless of legal decision-making authority, clinicians should strongly consider including adolescents in medical decisions and should work with adolescents and their families to achieve agreement whenever possible.



  • 3.

    During an influenza outbreak, the PICU is running out of ventilators. The MOST appropriate allocation strategy is:



    • A.

      Adhere to your institution’s predetermined standardized triage criteria


    • B.

      Perform a lottery


    • C.

      Prioritize neonates over older children since they have a longer life expectancy


    • D.

      Withdraw the ventilators from patients least likely to survive




Preferred response: A


Rationale


Allocation decisions for scarce resources are best made in advance, before a period of surge. This prevents triage decisions from being made at individual bedsides and eliminates potential injustices. Every institution should have a disaster plan in place to deal with surges—natural or manmade.



  • 4.

    A 3-year-old girl suffered a cardiac arrest following cardiac surgery. She has had minimal neurologic recovery. She has been in the CICU for 5 months with chronic critical illness. Her parents adamantly demand that everything medically possible be done for her despite ongoing communication with the medical team. Many members of the medical team are distressed at caring for this patient and advocate for withdrawal of life support. The most appropriate response from the medical team is:



    • A.

      Allow distressed staff to refuse to care for the child


    • B.

      Continue to tell the family daily that their child is suffering and that life support should be withdrawn


    • C.

      Continue to support the family and the medical team, and engage the ethics consult service to apply a fair process-based approach to evaluate if continuing medical therapies is potentially inappropriate


    • D.

      The medical team should tell the family that life support will be withdrawn against their wishes




Preferred response: C


Rationale


Clinicians cannot simply refuse to care for a child based on a claim of moral distress, or unilaterally withdraw life-support against a family’s wishes without a fair process. The medical team should continue to support the family and staff through these challenging times, working diligently to maintain open lines of communication and trust with the family.



  • 5.

    Which of the following is a true statement regarding the doctrine of informed consent?



    • A.

      Decisions must be voluntary and not subject to coercion, manipulation, or undue influence.


    • B.

      It does not apply to decisions involving research.


    • C.

      The patient needs to understand all the minutia of the treatment being discussed.


    • D.

      The decision making should be shared with a competent medical provider.




Preferred response: A


Rationale


The doctrine of informed consent applies to both medical decisions and research. Informed consent must satisfy four requirements that apply when surrogates provide permission as well as when consent is obtained directly from patients. With disclosure, the clinician should supply the patient with sufficient information that a “reasonable person” would desire to be able to make an informed medical decision. With understanding, the clinician should assess the patient’s understanding of the proposed course of action, the risks and benefits of that course of action, and any available alternatives along with the risks and benefits associated with those actions. Understanding may be particularly impaired in the critical care setting where the high stakes and time pressures can impact the ability to achieve optimal understanding. With capacity, the patient must meet legal requirements for competency, be able to understand the medical decision, form a reasonable judgment based on the consequences of the decision, and be able to communicate that decision to others. Legally, children under the age of 18 are not considered competent for medical decision making with the exception of emancipated and mature minors. Emancipated minors are considered competent based on characteristics that are defined by state law, but may include pregnancy, parenthood, or establishing financial independence. Mature minors represent another category that is defined by state law whereby a minor, usually above a certain age, can be judged competent to make certain medical decisions. Most states require a judge to make these determinations, and the judge may restrict the determination to the medical decisions at hand. With voluntariness, decisions must be voluntary and not subject to coercion, manipulation, or undue influence. Importantly, physicians should not withhold or deemphasize information in an effort to manipulate patients.



  • 6.

    Which of the following is an important consideration that may justify seeking legal intervention to override a parent’s decision to refuse a medical therapy?



    • A.

      Mediation and negotiation efforts are initiated and conducted by the state courts.


    • B.

      Parental authority is absolute and cannot be challenged legally by medical professionals.


    • C.

      The intervention refused by the parent is one that is not commonly performed.


    • D.

      The parent’s decision places the child at a significant risk of serious harm.




Preferred response: D


Rationale


In most situations, parents are granted wide latitude in the decisions they make on behalf of their children, and the law has respected those decisions except when they place the child’s health, well-being, or life in jeopardy. Parental authority is not absolute, however, and when a parent or guardian fails to adequately guard the interests of a child, the decision may be challenged, and the state may intervene. A clinician’s authority to interfere with parental decision making is limited. Except in emergency situations where a child’s life is threatened imminently or a delay would result in significant suffering or risk to the child, the physician cannot do anything to a child without the permission of the child’s parent or guardian. Touching (physical examination, diagnostic testing, or administering a medication) without consent is generally considered battery under the law. The clinician’s options include either tolerating the parents’ decision (while continuing to try to convince them to act otherwise) or involving a state agency to assume medical decision-making authority on behalf of the child.


Only the state can order a parent to comply with medical recommendations. This can take different forms, but most frequently either includes involvement of child protective services (under a claim of medical neglect) or a court order. Both of these options represent a serious challenge to parental authority, and parents will generally perceive them as disrespectful and adversarial. Such action interferes with family autonomy, can adversely affect the family’s future interactions with medical professionals, and can negatively impact the emotional well-being of the child. Neither should be undertaken without serious consideration. Before initiating the involvement of state agencies to limit parental authority and override parental refusal, the clinician must establish that (1) the recommended course of action is likely to benefit the child in an important way, (2) the treatment is of proved efficacy with a reasonable likelihood of success, (3) the parent or surrogate’s decision to refuse intervention places the child at significant risk of serious harm in comparison to the recommendations of the healthcare team (applying the harm principle), and (4) all attempts at mediation and negotiation to find a mutually acceptable solution have been exhausted.


Chapter 18 : Ethical issues around death and dying




  • 1.

    A 10-year old patient with recurrent acute myelogenous leukemia (AML) is now 10 days post-stem cell transplant and is admitted to the Pediatric Intensive Care Unit in critical condition from septic shock. Despite maximal support, the patient continues to deteriorate from multiple organ failure, including acute respiratory distress syndrome (ARDS), ventricular dysfunction, hepatic failure and diffuse coagulopathy with bleeding from every orifice. The patient’s parents now demand a trial of extracorporeal membrane oxygenation (ECMO). The most appropriate response to this request is:



    • A.

      Conduct a family meeting to discuss prognosis


    • B.

      Continue with current therapy


    • C.

      Consult hospital legal office


    • D.

      Provide a trial of ECMO


    • E.

      Write a unilateral DNR order




Preferred response: A


Rationale


The parents have a long established legal and ethical right to make decisions for their child, but they may not demand therapy that the physician deems inappropriate. However, before escalating the situation further, it is most appropriate to invest all necessary time and energy into listening to their concerns and fostering respectful communication by holding a family meeting as the next best step.



  • 2.

    The parents and the attending physician have made the decision to withdraw life-sustaining treatment from the 10-year-old patient with recurrent AML, now one month post-stem cell transplant with progressive multiple organ failure. Which of the following options reflects current thinking in medical ethics:



    • A.

      Neither withholding life-sustaining treatments nor withdrawing life-sustaining treatments is allowed without the assent of the 10-year old child in addition to the permission of the parents.


    • B.

      There is no ethical distinction between withholding life-sustaining treatments and withdrawing life-sustaining treatments


    • C.

      Withdrawing life-sustaining treatments is more ethical than withholding life-sustaining treatments


    • D.

      Withholding life-sustaining treatments is more ethical than withdrawing life-sustaining treatments.




Preferred response: B


Rationale


There is no morally relevant or logically valid distinction between withholding and withdrawing life-sustaining treatments in mainstream bioethics or in the law in the United States. They are considered to be the same.



  • 3.

    The decision is made to withdraw mechanical ventilation on a 10-year-old patient with recurrent AML, now one month post-stem cell transplant with progressive multiple organ failure and allow her to die. Which of the following options reflects current thinking in medical ethics on the goal of administering sedatives and analgesics in this context:



    • A.

      To hasten the patient’s death


    • B.

      To relieve the parent’s anxiety


    • C.

      To relieve the team’s anxiety


    • D.

      To treat patient discomfort




Preferred response: D


Rationale


The goal of administering these therapies is only to treat patient discomfort and not to hasten the dying process or treat concerns of family members or the team with medication to the patient.



  • 4.

    The decision is made to withdraw mechanical ventilation on a 10-year-old patient with recurrent AML, now one month post-stem cell transplant with progressive multiple organ failure and allow her to die. Which of the following might be indicated in this context?



    • A.

      Antisialagogue


    • B.

      Caffeine


    • C.

      Neuromuscular blocking agent


    • D.

      Potassium




Preferred response: A


Rationale


Medications that have no comfort relieving properties, but will hasten the death of the patient, such as neuromuscular blocking agents, potassium, and calcium are contraindicated in this context. Similarly, caffeine would work at odds with sedatives and analgesics in a patient in this context. An antisialagogue might be indicated in this context to reduce the secretion burden in this patient being extubated from mechanical ventilation with the goal of allowing her to die.


Chapter 19 : Palliative care in the pediatric intensive care unit




  • 1.

    12-year-old boy with a history of glioblastoma is in the ICU with respiratory failure and mental status changes. An MRI of his brain is obtained and identifies extensive leptomeningeal spread of his tumor. The oncology team has met with his parents to let them know there are no further cancer-directed therapies to offer. The family and medical team decide that it is time to discontinue invasive interventions and focus on comfort, and begin to plan for a compassionate extubation. Which of the following medications would be inappropriate to give as the ventilator is discontinued?



    • A.

      Hydromorphone


    • B.

      Ketamine


    • C.

      Lorazepam


    • D.

      Propofol


    • E.

      Vecuronium




Preferred response: E


Rationale


There is long-standing legal and moral precedent that pain and suffering at the end of life can and should be treated, and fears of legal repercussions should not prevent physicians from treating patients appropriately. It is also generally accepted, and supported by a series of court cases, that any medical technology can be discontinued if it is no longer providing a benefit to the patient, no longer desired by the competent adult patient, or merely prolonging imminent death. For children, parents are assumed to be the legal decision-makers except in unusual circumstances. Typically, there is little controversy when parents and the medical team agree on goals of care, the withdrawal of technology that is no longer helpful, and the management of suffering. The Doctrine of Double Effect is often used as an ethical justification for providing medications to treat pain and suffering at the end of life, since there is also a possibility that they will hasten death. The components that make the unintended consequence justifiable include:



  • 1.

    The act itself (treating suffering) must be inherently good


  • 2.

    The agent intends the good effect (treating suffering) rather than the bad effect (hastening death)


  • 3.

    The good effect must outweigh the bad effect (e.g., hastening death by many years for mild pain would be unacceptable)



Many add that the bad effect must not be a means to the good effect—meaning death is not used as the means to end suffering. Practically, these arguments can shed light on what medications may or may not be acceptable to use at the end of life. Medications that treat pain or other symptoms should be used in reasonable doses based on the patient’s prior exposure and expected tolerance and may be escalated rapidly as needed for untreated symptoms. Medications that are unacceptable are those that would merely hasten the dying process without treating suffering (e.g., neuromuscular blockade, potassium chloride).


In the vignette, it would be appropriate to give any of the sedative or opioid agents listed with an intent to treat pain or dyspnea, even if the respiratory drive might be suppressed or the intracranial pressure affected. Vecuronium, however, would be inappropriate as the ventilator is being withdrawn since neuromuscular blockade would hasten death without treating suffering (and in fact would potentially make it difficult to detect distress). Agents that are considered to provide deep sedation (e.g., ketamine or propofol) are typically considered only when other agents have proven ineffective or when the patient is known to be very tolerant to less sedating agents, but their use can still be justified.



  • 2.

    You are caring for an 8-year-old boy with end-stage metastatic osteosarcoma, with extensive bony and lung metastases. He was admitted to the PICU from home due to a pain crisis that could not be adequately controlled by his hospice team. He is currently on a hydromorphone PCA (patient-controlled anesthesia), with both a basal rate and a demand dose. You are escalating the dose aggressively, and he continues to complain of refractory pain, particularly in his right humerus and right scapula. Which of the following adjunctive pain treatments would be LEAST appropriate in this setting?



    • A.

      Bisphosphonates


    • B.

      Epidural analgesia


    • C.

      Ketamine


    • D.

      Lidocaine patch


    • E.

      Steroids




Preferred response: B


Rationale


Steroids and bisphosphonates are important adjuvant therapies for bone pain that is refractory to opiates. Ketamine would be a reasonable addition to the hydromorphone PCA, given its distinct mechanism of action (NMDA-antagonist) and its potential to help alleviate any component of neuropathic pain. Lidocaine patches can be useful adjuncts, particularly when the pain is highly localized to one or two areas. Epidural analgesia can be an invaluable adjunctive therapy for refractory pain in end-stage cancer, as long as the site of pain is at the chest or below. This patient’s site of pain (shoulder and scapula) would be inappropriately high to place an epidural catheter.



  • 3.

    You are caring for a 2-year-old boy, formerly a 26-week premature infant, with chronic respiratory failure due to bronchopulmonary dysplasia, dependent on mechanical ventilation via a tracheostomy tube. He also has global developmental delay and a seizure disorder. He is admitted to the PICU with severe pneumonia, sepsis, and multiorgan dysfunction syndrome. What is the best strategy to begin a discussion of limitation of interventions with the family?



    • A.

      Ask the family to share their understanding of their son’s current medical condition, as well as their values, hopes, and goals with regard to their son and his care.


    • B.

      Impress upon the family that the severity of the child’s illness and poor overall prognosis do not justify further aggressive life support given the child’s underlying developmental delay.


    • C.

      Inform the family that the patient is not going to live through the night.


    • D.

      Present a list of interventions and ask whether or not the family would like them performed.




Preferred response: A


Rationale


Discussions of this nature should always begin by allowing the family to articulate their understanding of the child’s current condition, as well as their hopes, values, and goals for their child overall. After the clinician has heard this information, recommendations for or against various interventions can be made as they pertain to the family’s stated goals. Although it is important to ensure that the family understands the severity of illness and poor prognosis, it is not appropriate to make a recommendation against continuing life support based only on the presence of developmental delay. Physicians are poor at predicting the exact time of death for patients, so statements of certainty around death should be avoided. Physicians should avoid presenting families with a “menu” of options for care; rather they should elicit the family’s goals and make recommendations for pursuing or limiting certain interventions based on their stated goals. Physicians must avoid phrases such as “do everything,” as they are nonspecific and imply that “doing” is always the best course of action.



  • 4.

    Which of the following is true of methadone use for symptom management?



    • A.

      It has a toxic metabolite that can accumulate and lead to seizures.


    • B.

      It has a long and variable half-life, which can lead to accumulation over time.


    • C.

      It is renally excreted and therefore should be avoided in patients with renal failure.


    • D.

      It shortens the QT interval.




Preferred response: B


Rationale


The half-life of methadone varies from less than 10 hours to greater than 75 hours, depending on a variety of host factors. It can easily accumulate as it is reaching steady state and lead to oversedation and even obtundation. Therefore dosing and titration must be performed carefully. Morphine is renally excreted and should be avoided in renal failure. Meperidine has a toxic metabolite that can accumulate and lead to seizures. Fentanyl is the only opiate available as a transdermal patch. Methadone prolongs the QT interval, and a screening electrocardiogram should be considered prior to its initiation.


Chapter 20 : Organ donation process and management of the organ donor




  • 1.

    You are caring for a 4-week-old infant who had a prolonged cardiac arrest after being placed face down in the crib. The child has sustained significant anoxic brain injury. On the third hospital day, the infant has a Glasgow Coma Score of 3 with fixed and dilated pupils. The infant is mechanically ventilated and receiving a dopamine infusion of 4 μg/kg/min. Which statement is most correct about ongoing medical care for this infant?



    • A.

      Authorization for donation after circulatory death (DCD) should be obtained from the family during discussions and decisions about end-of-life care.


    • B.

      Early referral for organ donation to the organ procurement organization (OPO) enhances recovery of organs for transplantation.


    • C.

      Organs cannot be recovered from neonatal donors.


    • D.

      The role of the pediatric intensivist ends after providing support for the family during end-of-life care and declaring death because of potential conflict of interest with organ donation.




Preferred response: B


Rationale


Early referral for organ donation to the organ procurement organization (OPO) is considered a best practice and enhances recovery of organs for transplantation. Donor management should be viewed as a continuum of care provided by the pediatric intensivist and the critical care team from the time of admission to recovery of organs for transplantation. Involvement of intensivists results in better donor management and recovery of more organs with better graft function following transplantation. The discussions and decision to authorize organ donation after cardiac death (DCD) should only occur after an independent decision to withdraw medical support has been decided upon by the parents or legal guardian. This firewall avoids the perceived ethical conflict that the patient is being allowed to die to recover organs. Authorization must occur from parents or legal guardians.



  • 2.

    A 10-month-old unrestrained child who is a victim of a motor vehicle crash is admitted to your PICU. The child has sustained significant intracranial injury with subdural blood noted on the initial computed axial tomography scan of the head. The neurosurgical team has indicated that no surgical intervention is required at this time. This child has a Glasgow Coma Score of 3. No sedation or analgesia has been administered for the past 8 hours. The child is endotracheally intubated and mechanically ventilated with fraction of inspired oxygen (F io 2 ), 0.8; peak inspiratory pressure, 30 cm H 2 O; positive end expiratory pressure, 10 cm H 2 O; ventilator rate, 22 breaths per minute. The chest radiographs show bilateral pulmonary contusions. Agonal respirations noted 24 hours ago have ceased. Blood pressure is being supported with an epinephrine infusion of 0.7 μg/kg/min. Urine output has remained acceptable since admission 2 days ago. The bedside nurse notifies you that the child has fixed and dilated pupils. Which of the following statements is most correct?



    • A.

      Ancillary studies are not required to make the diagnosis of neurologic death in infants younger than one year of age.


    • B.

      An observation period of 24 hours between neurologic examinations is recommended to determine neurologic death in this patient.


    • C.

      The partial pressure of carbon dioxide (Paco 2 ) of 59 mm Hg during the apnea exam is sufficient to confirm brain death.


    • D.

      Two separate examinations and a single apnea test are required to determine neurologic death in infants and children.




Preferred response: A


Rationale


Determination of neurologic death is a clinical diagnosis that relies on the neurologic examination and an apnea test. Neurologic death can be determined in term infant 37 weeks estimated gestational age (EGA) to 30 days of age to 18 years of age. It can be more difficult to make a determination of neurologic death in the neonate and younger child; therefore, serial examinations are essential to ensure the clinical examination remains consistent throughout the observation and testing period. The recommended observation period for infants 37 weeks EGA to 30 days of age is 24 hours. A 12-hour observation period is recommended for children older than 30 days of age. Apnea testing must be performed with each examination, and the Paco 2 must rise to >60 mm Hg and 20 mm Hg above the baseline Paco 2 for the apnea test to be valid by current national guidelines. Ancillary studies are not required to make a determination of neurologic death in any patient of any age unless the clinical examination and apnea test cannot be completed, making choice A the correct response.



  • 3.

    You have been asked by the organ procurement organization to assist with management of a 4-year-old child. This patient has been declared dead by neurologic criteria, and the parents have authorized donation. This patient has required extensive fluid resuscitation for blood pressure support. The child continues to receive vasopressor support with an epinephrine infusion of 0.6 μg/kg/min and dopamine of 12 μg/kg/min. Mechanical ventilation is instituted as follows: fraction of inspired oxygen (F io 2 ), 0.65; peak inspiratory pressure, 28 cm H 2 O; positive end expiratory pressure, 12 cm H 2 O; ventilator rate, 22 breaths per minute. Laboratory evaluation reveals: hemoglobin, 7.6 g/dL; hematocrit, 22.3%; serum sodium, 162 mEq/L; potassium, 3.1 mEq/L; chloride, 119 mEq/L; blood urea nitrogen (BUN), 19 mg/dL; creatinine, 1.2 mg/dL. Which of the following statements is correct?



    • A.

      Corticosteroids can stabilize lung function and reduce free water accumulation in a donor


    • B.

      Desmopressin for treatment of diabetes insipidus can improve blood pressure and potentially reduce vasopressor requirements in a donor


    • C.

      Intranasal vasopressin is the preferred treatment for diabetes insipidus in a donor


    • D.

      Thyroid hormone prevents the anaerobic to aerobic cellular metabolic shift in a donor




Preferred response: A


Rationale


Hormonal replacement therapy (HRT) restores aerobic metabolism, replaces hormone derived from the hypothalamus and pituitary, augments blood volume, and minimizes the use of inotropic support while optimizing cardiac output. Common agents used for HRT include thyroid hormone, corticosteroids, and vasopressin or desmopressin for treatment of diabetes insipidus.


Corticosteroids such as hydrocortisone are another pharmacologic agent routinely used by many centers for HRT to assist with hemodynamic support. Treatment of the donor with high doses of corticosteroids reduces inflammation associated with neurologic death and modulates immune function. The potential benefit of hydrocortisone and other steroids may lie in their ability to alter adrenergic receptors and regulate vascular tone by increasing sensitivity to catecholamines. Steroids have also been shown to stabilize pulmonary function, reduce lung water accumulation, and increase lung recovery from donors, making A the correct response.


The effects of thyroid hormone on myocardial contractility can be immediate or delayed. Thyroid hormone is commonly used in the hemodynamically unstable donors. The acute inotropic properties of thyroid hormone may occur as a result of beta-adrenoreceptor sensitization. Additionally, thyroid hormone administration may play an important role in maintaining aerobic metabolism at the tissue level after neurologic death has occurred. Levothyroxine (Synthroid) and triiodothyronine (T3) are the two intravenous thyroid agents available for administration.


Vasopressin is commonly used to treat diabetes insipidus (DI) in the donor. This agent is not as potent as desmopressin on a weight per weight basis. Use of vasopressin for treatment of DI can reduce the need for vasopressor support. Vasopressin and desmopressin should be administered by the intravenous route. Intranasal administration is not recommended because of erratic or no absorption in the brain-dead donor.



  • 4.

    You are caring for a critically ill 15-month-old patient who was a victim of abusive head trauma admitted to your PICU 5 days ago. This child is comatose with a Glasgow Coma Score of 3, fixed and dilated pupils, and no response to noxious stimulation. The child is mechanically ventilated with the following settings: fraction of inspired oxygen (F io 2 ), 1.0; peak inspiratory pressure, 32 cm H 2 O; positive end expiratory pressure, 8 cm H 2 O. Blood pressure is being supported with an epinephrine infusion of 0.9 μg/kg/min, and 15 mg/kg/min of dopamine. The child has a markedly distended abdomen and has minimal urine output.




    • A chest radiograph revealed posterior and lateral rib fractures and bilateral pleural effusions with a right upper lobe consolidation.



    • Computed tomography scan of the head revealed bilateral subdural and subarachnoid hemorrhages, diffuse cerebral edema, and effacement of the lateral ventricles.



    • Serum laboratory studies revealed white blood cell count, 10.6 × 103/µL; hemoglobin 7.8 g/dL; hematocrit, 24.2%; sodium, 157 mEq/L; potassium, 4.1 mEq/L; chloride, 116 mEq/L; bicarbonate, 16 mEq/L, blood urea nitrogen, 24 mg/dL; creatinine, 1.92 mg/dL; glucose, 136 mmol/L; lactate, 4.3 mg/dL; alanine aminotransferase, 93 U/L; aspartate aminotransferase,126 U/L; albumin, 2.6 g/dL; protein, 4.1 g/dL; prothrombin time (PT), 15 seconds; international normalized ration (INR), 1.6; activated partial thromboplastin time (aPTT), 62 seconds.



    • Arterial blood gas revealed pH, 7.392; partial pressure of carbon dioxide (Paco 2 ), 30 mm Hg; partial pressure of oxygen (Pao 2 ), 83 mm Hg; bicarbonate, 18 mEq/L; base excess, −5 mmol/L.



    • Which of the following statements is correct?


    • A.

      Early referral of the organ procurement organization after neurologic death has been declared can enhance chances for organ recovery.


    • B.

      Maintenance of euvolemia and correction of metabolic derangements will have little effect on graft function and viability of organs for transplantation.


    • C.

      Notifying the medical examiner or coroner’s office prior to death may expedite and facilitate organ donation.


    • D.

      Organ donation is not possible because of multisystem organ dysfunction.




Preferred response: C


Rationale


Involvement of the pediatric intensivist and critical care team in the management of critically ill and injured children, especially in pediatric donation, where there is a limited and decreasing number of donors improves the quality and number of organs recovered.


Preconceptions about eligibility for donation by the critical care team may not be current or accurate. Donor organ suitability for transplantation is best assessed by the organ procurement organization (OPO). Thresholds for acceptable organ dysfunction can vary according to time of evaluation, transplant program comfort levels, and recipient urgency. In many instances management of the potential organ donor can improve organ function and increase chances of successful organ recovery. Serial echocardiograms may demonstrate donor response to effective medical therapy enabling cardiac recovery for transplantation; a positive blood culture or bacterial meningitis may not preclude organ donation if antibiotic therapy has been administered. Organs from HIV positive donors can be transplanted into HIV-positive recipients, and organs from hepatitis positive donors are now being transplanted with good success.


Successful recovery of organs and the prosecution of the perpetrator may still occur in most cases of child homicide with close cooperation between forensic investigators, treating physicians, the transplant team, and OPO. Early involvement of the medical examiner or coroner prior to determination of death and protocols to facilitate organ recovery in homicide cases may reduce denials for organ donation, making answer C the correct choice. Efforts to reduce the number of medical examiner denials for donation are supported in the position statement by the National Association of Medical Examiners.


Early involvement and timely notification of the OPO prior to determination of death allows a greater amount of time for collaboration to coordinate the donation process. Notification of the OPO after determination of death is considered a late referral and may not allow adequate time for the OPO and critical care team to fully discuss donation opportunities. The OPO can also work closely with the medical examiner or coroner to facilitate donation in cases of homicide. Ensuring OPO engagement early in the course of caring for a critically ill patient allows the intensive care team to better understand the entire donation process and eliminate confusion that may disrupt end-of-life care and the process of donation. Early referral improves authorization rates and can assist families with understanding and coping with end-of-life care issues.



  • 5.

    Which of the following substances is markedly elevated immediately following brain death:



    • A.

      Antidiuretic hormone


    • B.

      Catecholamines


    • C.

      Cortisol


    • D.

      Insulin


    • E.

      Thyroid hormone




Preferred response: B


Rationale


Neurologic death resulting from cerebral ischemia increases circulating cytokines, reduces cortisol production, and precipitates massive catecholamine release. Hemodynamic deterioration associated with neurologic death is initiated by a massive release of catecholamines, commonly referred to as sympathetic, catecholamine, or autonomic storm. The sympathetic storm results in intense but transient hypertension.


The tremendous physiologic derangements associated with neuroendocrine dysfunction require specific interventions to restore normal physiology. Agents such as thyroid hormone, corticosteroids, vasopressin, and insulin are commonly employed during donor management. Hormonal replacement therapy can reduce circulatory instability associated with thyroid and cortisol depletion, especially in situations where significant inotropic support is required.



  • 6.

    Which of the following metabolic derangements is common following neurologic death?



    • A.

      Hypercalcemia


    • B.

      Hyperglycemia


    • C.

      Hypermagnesemia


    • D.

      Hypochloremia


    • E.

      Hyponatremia




Preferred response: B


Rationale


The critical care team should actively manage the potential donor and correct existing physiologic and metabolic derangements that follow neurologic death to preserve the option of organ donation for the family. Metabolic derangements such as iatrogenic hypernatremia from hyperosmolar therapy and hyperglycemia associated with catecholamine release and reduced cerebral metabolism should be corrected. Volume loss from osmotic diuresis associated with hyperglycemia and diabetes insipidus (DI) following neurologic death must be anticipated and addressed to prevent cardiovascular collapse.


Without substrate consumption by the brain, glucose needs are reduced, and the patient is prone to hyperglycemia. As neurologic death occurs, cerebral metabolism is further decreased and co 2 production falls resulting in a reduction in Paco 2 . Hypothermia should be anticipated as a result of hypothalamic failure and loss of thermoregulation. Additionally, impaired adrenergic stimulation results in loss of vascular tone with systemic vasodilation and amplified heat losses.


Hypocalcemia occurs commonly secondary to large volume replacement with colloids such as albumin, massive blood transfusions that result in large amounts of citrate reducing free calcium concentrations, and sepsis. Calcium is necessary for myocardial contraction and hypocalcemia can depress cardiac output, affect SVR, and organ perfusion. The use of calcium supplementation should be guided by ionized calcium levels.



  • 7.

    Evaluation of a 3-year-old child for brain death is being considered. To determine brain death in this child, which of the following is most accurate?



    • A.

      A longer period of observation in this young child is required because of legal issues.


    • B.

      An ancillary study is required to make the diagnosis of brain death in children.


    • C.

      Ancillary studies such as electroencephalogram, radionuclide cerebral blood flow study, and computed tomography angiography are used in children.


    • D.

      A thorough neurologic examination and apnea test are required to make the diagnosis of brain death.




Preferred response: D


Rationale


In a child older than 1 year, brain death is a clinical diagnosis and does not require an ancillary study unless the examination and apnea test cannot be completed. A thorough neurologic examination and apnea test that is repeated after a specified observation period is required to determine brain death based on clinical criteria. A longer observation period is currently recommended in infants younger than 30 days and for children younger than 1 year. For children older than 1 year, an observation period of 12 hours is recommended. Ancillary studies such as an electroencephalogram and cerebral blood flow study can be used to assist with the diagnosis of brain death. Computed tomography angiography has not been validated in children and cannot be relied on as a diagnostic ancillary test to determine brain death in children. The apnea test must achieve a Pco 2 of ≥60 mm Hg to be consistent with brain death.



  • 8.

    Which statement about neurologic death in infants and children is true?



    • A.

      Ancillary studies are required to make the diagnosis of neurologic death in infants younger than 1 year of age.


    • B.

      A Paco 2 of 59 mm Hg in a child with acute lung injury who desaturates 3 minutes into the apnea exam is consistent with brain death.


    • C.

      Neurologic death can be determined in term newborns at 37 weeks’ estimated gestational age.


    • D.

      The revised pediatric brain death guidelines require two separate examinations and a single apnea test to determine neurologic death in infants and children.



    Preferred response: C



Rationale


Determination of neurologic death can occur in a term infant at 37 weeks’ estimated gestational age (EGA) to 30 days of age. It can be more difficult to make a determination of neurologic death in the neonate; therefore serial examinations are essential to ensure that the clinical examination remains consistent throughout the observation and testing period. The recommended observation period for infants at 37 weeks’ EGA to 30 days of age is 24 hours. Apnea testing must be performed with each examination, and the Paco 2 must rise to >60 mm Hg and 20 mm Hg above the baseline Paco 2 to document apnea.


Two neurologic examinations and apnea tests separated by an observation period are required to establish the diagnosis of neurologic death in the United States. The duration of observation between examinations is based on age. Ancillary studies are not mandatory to make a determination of neurologic death. The physician caring for the child will determine the need for ancillary studies based on history and the ability to complete the clinical examination and apnea testing. If clinical examination and apnea testing cannot be safely completed, an ancillary study should be used to assist with determination of death.


Chapter 21 : Long-term outcomes following critical illness in children




  • 1.

    How does health-related quality of life (HRQL) differ from quality of life?



    • A.

      Consideration of entirety of past medical history for HRQL


    • B.

      Dimension of personal judgment over one’s health and disease


    • C.

      Inclusion of functional status during assessment of HRQL


    • D.

      Utilization of prospective as opposed to retrospective evaluation




Preferred response: B


Rationale


Quality of life is defined as an individual’s perception of his or her position in life in relation to the individual’s goals, expectations, standards, and concerns. HRQL is defined as quality of life in which a dimension of personal judgment over one’s health and disease is added, and encompasses the impact of health status on physical, mental, emotional, and social functioning.



  • 2.

    A group of investigators is designing an interventional trial and planning to utilize health-related quality of life (HRQL) as the primary endpoint. Which of the following protocol elements would optimize assessment for this outcome measure?



    • A.

      Ascertain baseline medical complexity for chronic comorbid conditions


    • B.

      Conduct paired baseline and follow-up HRQL surveys


    • C.

      Control for duration of intensive care unit stay


    • D.

      Undertake illness severity measures (e.g., PRISM, PIM) of all patient participants




Preferred response: B


Rationale


Conducting paired HRQL assessments controls for each subject’s baseline status and permits change from baseline (paired) analyses. Semi-quantification of baseline chronic comorbid conditions represents an alternative but less specific approach.



  • 3.

    Which of the following represents a risk factor for prolonged deterioration of functional status from baseline following pediatric critical illness?



    • A.

      Duration of stay >28 days


    • B.

      Elective PICU admission


    • C.

      Nononcologic diagnoses


    • D.

      Younger age at PICU admission




Preferred response: A


Rationale


Longer duration of PICU stay, oncologic diagnoses, older age, and emergent PICU admission are all risk factors for prolonged deterioration of functional status assessed by Pediatric Overall Performance Category.



  • 4.

    Which of the following adverse events was most commonly documented among a cohort of children surviving critical illness at a median follow-up time of 5 months after PICU discharge?



    • A.

      Fatigue disorder


    • B.

      Post-traumatic stress disorder


    • C.

      Psychiatric disorders


    • D.

      Sleep disorder




Preferred response: D


Rationale


One study found that at a median of 5 months after discharge, 20% of PICU survivors were at risk for psychiatric disorders, 34% were at risk for PTSD, 38% were at risk for fatigue disorder, and 80% were at risk for a sleep disorder.


Chapter 22 : Burnout and resiliency




  • 1.

    Which of the following is a common characteristic of compassion fatigue?



    • A.

      Alcohol use


    • B.

      Low energy in the workplace


    • C.

      Reduced capacity for empathy


    • D.

      Unrelieved stress and tension




Preferred response: C


Rationale


Compassion fatigue is broadly defined as reduced capacity and interest in being empathetic for those who are suffering. Although often used interchangeably, burnout consists of three dimensions including depersonalization, emotional exhaustion, and diminished feelings of personal accomplishment. While compassion fatigue may result in burnout, they are not synonymous. Low energy in the workplace, unrelieved stress and tension, and alcohol use may all contribute to or be signs or symptoms of burnout; none is specific to define compassion fatigue.



  • 2.

    Which of the following factors has been found to be significant in the development of burnout among pediatric critical care physicians?



    • A.

      Female gender


    • B.

      Older age of a practitioner


    • C.

      Presence of a palliative care consult team


    • D.

      Years in pediatric critical care practice




Preferred response: A


Rationale


A recent national cross-sectional online survey exploring burnout and psychological distress among over 250 pediatric critical care physicians in the United States found that the risk of any burnout was about two times more in women physicians (odds ratio, 1.97; 95% CI, 1.2–3.4). Association between other personal or practice characteristics and burnout was not evident in the study, while regular physical exercise appeared to be protective.



  • 3.

    Which of the following measures is considered a successful organizational strategy for mitigating burnout in critical care providers?



    • A.

      Administering annual staff satisfaction surveys


    • B.

      Increasing helpful pop-up alerts within the EMR


    • C.

      Limiting family visitation hours


    • D.

      Using ethics and palliative care consultations in the ICU




Preferred response: D


Rationale


The Critical Care Societies Collaborative’s Call to Action to address burnout identified that ethics and palliative care consultations is a successful strategy to mitigate burnout by divesting some of the burden of end-of-life discussions to clinical experts. Additionally, providing healthy food options for clinicians, providing an on-site gymnasium, and offering stress-reduction courses are all considered to be useful for mitigating burnout in critical care providers.



  • 4.

    What factor most specifically impacts the development of burnout in pediatric critical care providers?



    • A.

      Dealing with dying children


    • B.

      ICU shift length


    • C.

      Open visitation in the pediatric ICU


    • D.

      Pediatric resident and fellow rotation schedules




Preferred response: A


Rationale


Dealing with the death of a critically ill pediatric patient has been demonstrated to be a specific factor impacting the development of burnout in pediatric critical care providers compared to general ICU related factors.


Chapter 23 : Structure and function of the heart




  • 1.

    When examining a cardiac pathology specimen, atrial anatomy is defined by:



    • A.

      The venous return to the heart (typically the inferior vena cava or IVC, superior vena cava or SVC, and coronary sinus to the right atrium and pulmonary veins to the left atrium), the sinus node as a right atrial structure, and the atrial relationship to the atrioventricular (tricuspid or mitral) valves.


    • B.

      The constant defining features of a right versus left atrium are the atrial appendage and its extent of pectinate muscles and the venous return to the heart (IVC, SVC, and coronary sinus to the right atrium and pulmonary veins to the left atrium).


    • C.

      The constant defining features of a right versus left atrium are the atrial appendage and its extent of pectinate muscles and typically the venous return to the heart (IVC, SVC, and coronary sinus to the right atrium and pulmonary veins to the left atrium).


    • D.

      Pulmonary venous return, a broad based atrial appendage, and sinus node as constant features of a right atrium.




Preferred response: C


Rationale


Ventricular morphology is defined by the atrioventricular (AV) valve it contains; however, the atria are not always related to the same sided AV valve (i.e., AV discordance), making answer A incorrect. Systemic venous return is typically to the right atrium and pulmonary venous return to the left atrium, but not always, making answer B incorrect. Answer D is also incorrect, since pulmonary venous return is typically a feature of the left atrium.



  • 2.

    Which statement is true regarding the transition from fetal to postnatal circulation?



    • A.

      Compared to an infant at 6 weeks of age, the infant at birth relies mostly on stroke volume in order to have adequate cardiac output.


    • B.

      The ductus arteriosus is kept open in fetal circulation by a balance between prostaglandin E2 (PGE2) and endothelin-1 (ET-1). After birth, the vasoconstrictive effects of PGE2 become more dominant and the ductus constricts, leading to closure of the ductus within 24 hours to 3 weeks


    • C.

      There is a significant decrease in total body oxygen consumption and cardiac output after birth. Over time, with increased distensibility of the ventricular myocardium, the infant relies less on heart rate to augment cardiac output.


    • D.

      With cord clamping, the baby experiences a rise in systemic vascular resistance (SVR) and a reduction in pulmonary vascular resistance (PVR).




Preferred response: D


Rationale


During transition to postnatal circulation, there is a rise in SVR and a reduction in PVR, and there is a significant increase in total body oxygen consumption and cardiac output, which initially is heavily reliant on heart rate to increase output until ventricular distensibility improves. Cardiac output in the fetus is determined mainly by heart rate because of a limited capacity to increase stroke volume that results mainly from decreased diastolic distensibility. Consequently, fetal bradycardia is detrimental to blood flow and oxygen delivery. In addition, because at birth approximately 80% of the infant’s hemoglobin is in the form of fetal hemoglobin, the reduced ability of this hemoglobin to unload oxygen at the tissue level compels the infant to have a higher cardiac output than the infant will have 4 to 6 weeks later. Therefore the neonate has limited cardiac output reserve, and the heart has near-maximal contractility. These features make the neonate unusually susceptible to diseases that impair cardiac function.



  • 3.

    When assessing a child with cardiomyopathy who has left ventricular dilation and moderate mitral regurgitation, the preferred method to evaluate and monitor systolic function via echocardiography is:



    • A.

      Change in ventricular pressure during isovolemic contraction before the aortic valve opens (dP/dT), taken from a continuous wave Doppler of the mitral regurgitation jet.


    • B.

      M-mode derived TAPSE (tricuspid annular plane systolic excursion).


    • C.

      M-mode generated ejection fraction.


    • D.

      No reliable echocardiographic method is available, and this patient should undergo cardiac MRI for functional assessments.




Preferred response: A


Rationale


TAPSE (tricuspid annular plane systolic excursion) is used to assess right ventricular systolic function (answer B is incorrect). Although an M-mode generated ejection fraction is a popular and common method used to assess systolic function in children, it is less accurate in the context of mitral regurgitation and left ventricular (LV) dilation (answer C is incorrect). Although cardiac MRI is useful for obtaining accurate chamber volumes and measures of systolic function, echocardiography is more practical and remains an accurate method for noninvasive regular surveillance of ventricular systolic function (answer D is incorrect). In the context of LV dilation and mitral valve regurgitation (MR), dP/dT (rate of pressure change over time during isovolumic contraction) is an accurate method of monitoring ongoing changes in contractility and is relatively unaffected by preload changes. However, the clinician must keep in mind that dP/dT is affected by changes in afterload. Therefore, in the context of heart failure therapy with afterload reducing agents, changes in dP/dT measurements of systolic function will need to be considered in this clinical setting.



  • 4.

    Which statement is true in the context of reduced left ventricular contractility?



    • A.

      Due to the steep slope of the ventricular elastance line, small changes in afterload result in large increases in stroke volume and decreases in end diastolic pressure and volume.


    • B.

      In order to eject a normal stroke volume, even in the face of a normal afterload, the ventricle compensates by increasing end diastolic volume.


    • C.

      In order to maintain stroke volume and cardiac output, afterload is increased.


    • D.

      In order to keep ejecting the same stroke volume, preload reserve is maintained during periods of increased afterload.




Preferred response: B


Rationale


See Figure 136.1 . With decreased LV contractility, the ventricle compensates by increasing end diastolic volume (EDV) in order to maintain a normal stroke volume (answer B is correct). A higher afterload causes even further increases in EDV and EDP, which eventually leads to pulmonary edema (answer C is incorrect). In this setting, the normal preload reserve has already been used to maintain stroke volume (answer D is incorrect). Afterload reduction in this context is useful because a small decrease in afterload results in a large increase in stroke volume and large decrease in end-diastolic volume (EDV) and end-diastolic pressure (EDP). There is a relatively flat slope of the maximal ventricular elastance line, NOT a steep slope. Answer A is incorrect.



  • 5.

    The main difference in the fetal/neonatal myocardium compared with mature adult myocardium is:



    • A.

      Complete coupling of β adrenoreceptors to G-proteins


    • B.

      More β adrenoreceptors


    • C.

      More contractile components


    • D.

      Sparse, disorganized, and immature T-tubule and sarcoplasmic reticular system





• Fig. 136.1


Preferred response: D


Rationale


The fetal/neonatal myocardium contains less β adrenoreceptors and less contractile components when compared to the adult myocardium. There is incomplete uncoupling of β adrenoreceptors to G-proteins in the fetal/neonatal myocardium as well. The T-tubule and sarcoplasmic reticular system of the fetal/neonatal myocardium are sparse and disorganized.



  • 6.

    Except in young infants, the preferential source of energy for myocardial function comes from:



    • A.

      Glucose


    • B.

      Glycogen


    • C.

      Ketones


    • D.

      Long-chain fatty acids




Preferred response: C


Rationale


In all except young infants, the preferential source of energy for myocardial function comes from the β-oxidation of long-chain fatty acids. After fatty acids enter the cell, they are activated to fatty acid (or acyl) coenzyme A (CoA) compounds by palmitoyl-CoA synthetase, then linked by carnitine palmitoyl transferase I to carnitine to form acylcarnitines, thus releasing CoA. The acylcarnitines cross the mitochondrial membrane, and at the inner surface of the membrane another enzyme, carnitine palmitoyl transferase II, transfers the fatty acids back to CoA. The fatty acids can now undergo β-oxidation with the production of ATP. These enzymes also help transport acylcarnitine esters of CoA out of the mitochondria. These esters are toxic in high concentrations. Fetuses and neonates have decreased activity of carnitine palmitoyl transferase and palmitoyl-CoA synthetase, so glucose, lactate, and short-chain fatty acids are the preferred myocardial energy substrates at this age.



  • 7.

    The true statement concerning endothelins is:



    • A.

      Bosentan is an endothelin agonist.


    • B.

      Endothelins act on ET-A receptor and cause vasodilation.


    • C.

      Endothelins act on ET-B receptor and cause vasoconstriction.


    • D.

      Endothelins cause either vasoconstriction or vasodilation.




Preferred response: D


Rationale


The vascular endothelium elaborates the endothelins (ET-1, ET-2, ET-3), a family of compounds that are vasoactive, structurally related peptides. ET-1 is the most potent vasoconstrictor known. It also promotes mitogenesis and stimulates the renin-angiotensin-aldosterone system and the release of vasopressin and atrial natriuretic peptide. These peptides act on one of two receptor subtypes: ET(A) and ET(B). ET(A) is located mainly on vascular smooth muscle cells and is responsible for mediating vasoconstriction and cell proliferation. ET(B) is present predominantly on endothelial cells and mediates vasorelaxation, as well as ET-1 clearance. Endothelins cause local vasoconstriction or vasodilation, depending on dose and location in the circulation. Individual endothelins occur in low levels in the plasma, generally below their vasoactive thresholds. This finding suggests they are primarily effective at the local site of release. Even at these levels, however, they may potentiate the effects of other vasoconstrictors such as norepinephrine and serotonin. Endothelin antagonists, such as bosentan, now are being used specifically in the setting of pulmonary arterial hypertension.



  • 8.

    Regarding the regulation of vasomotor tone:



    • A.

      Adenosine causes vasoconstriction.


    • B.

      Hypokalemia causes vasodilation.


    • C.

      Hyperkalemia causes vasodilation.


    • D.

      Neuropeptide Y causes vasodilation.




Preferred response: C


Rationale


Local metabolic regulation of vasomotor tone provides an ideal homeostatic mechanism whereby metabolic demand can directly influence perfusion. Adenosine, which accumulates locally when tissue metabolism is high and tissue oxygenation is marginal, causes pronounced vasodilation in the coronary, striated muscle, splanchnic, and cerebral circulations.


Potassium is released from muscle in response to increased work, ischemia, and hypoxia. Hypokalemia causes vasoconstriction. Hyperkalemia, within the physiological range, causes vasodilation by stimulating KIr channels.


In all organs, sensory and efferent nerve endings contain nonadrenergic, noncholinergic (NANC) peptides, for example, neuropeptide Y, VIP, calcitonin gene-related peptide (CGRP), and substance P. Neuropeptide Y is colocalized and released with norepinephrine, and VIP is colocalized with acetylcholine and released upon stimulation of vagal nerve endings. Most of these peptides except neuropeptide Y are vasodilatory, and they help modulate blood pressure and regional flows.



  • 9.

    What does the segment “A” denote in the Pressure-Volume loop depicted in Figure 136.2 below?



    • A.

      Ejection fraction


    • B.

      End diastolic pressure


    • C.

      Ejection time


    • D.

      Stroke volume




    • Fig. 136.2



Preferred response: D


Rationale


The figure depicts the pressure-volume relationship for a single cardiac cycle. During diastolic filling, volume increases and diastolic pressure rises slightly because of the increase in passive tension. At the end of diastole, isovolumic systole occurs, and ventricular pressure rises with no change in volume. When ventricular pressure exceeds aortic pressure, the aortic valve opens, blood is ejected, and ventricular volume decreases. Ejection ends, and pressure falls to diastolic levels as isovolumic relaxation occurs. The pressure and volume reached at the end of systole are those that would have been attained by the isolated ventricle at that same end-systolic volume. In other words, at a given volume, no higher pressure can be generated. The decrease in volume during ejection is the stroke volume which, divided by the end-diastolic volume, gives the ejection fraction; normally, ejection fraction is greater than 65%.



  • 10.

    In what situation would the left ventricular (LV) pressure-volume relationship in a previously healthy 12-year-old be expected to change from loop abcd to loop a′b′c′d′ in Figure 136.3 , below?



    • A.

      Blood transfusion


    • B.

      Hemorrhage


    • C.

      Myocardial contusion


    • D.

      Use of low-dose epinephrine




    • Fig. 136.3



Preferred response: C


Rationale


Loop abcd represents the pressure-volume relationship in an intact heart. Contraction begins at the end-diastolic pressure and volume of point a. Line ab represents isovolumetric contraction. At point b, the aortic valve opens, since left ventricular (LV) pressure exceeds that in the aorta. Ejection begins at point b and ends at point c. At this point, the aortic pressure is equal to the maximal force produced by the ventricular wall for that specific end-systolic fiber length. Isovolumetric relaxation starts at point c and the aortic valve closes. The ventricular pressure drops (line cd). The mitral valve will open when LV pressure falls below left atrial pressure and blood will flow into the LV.


The difference between lines ab and cd is the stroke volume. Point a represents preload and point b represents afterload.


Loop a′b′c′d′ reflects a decrease in contractility, which could be the result of myocardial contusion. There is a decrease in stroke volume despite a larger end-diastolic volume at a similar level of LV pressure.


Afterload reduction and increase in inotropy (low-dose epinephrine or milrinone) would shift the PV curve to the left and upward.


A blood transfusion increases preload, resulting in an increase in stroke volume (see Figure 136.4 below, loop 2). The contractility has not changed (both end-systolic points are on the same line).



  • 11.

    At this same level of contractility, what strategies would restore the stroke volume for the patient with myocardial contusion and decreased contractility to his baseline?



    • A.

      Increase in preload and inotropic support


    • B.

      Increase in preload and vasoconstrictor therapy


    • C.

      Increase in preload and vasodilator therapy


    • D.

      Use of diuretics and inotropic support





• Fig. 136.4


Preferred response: C


Rationale


In patients with decreased myocardial contractility, stroke volume can be restored with either an increase in preload (loop 4) or a decrease in afterload (loop 3) (see Figure 136.5 , below).



  • 12.

    Loop 1 in Figure 136.6 , below, represents the pressure-volume loop in an intact heart. The correct statement regarding the other loops (2 and 3) is:



    • A.

      Loop 2 is reflective of a decrease in preload, as would occur with diuretics.


    • B.

      Loop 3 is reflective of a decrease in preload, as would occur with diuretics.


    • C.

      Loop 2 demonstrates the effect of increasing afterload.


    • D.

      Loop 2 demonstrates the effect of decreasing afterload.




    • Fig. 136.6




• Fig. 136.5


Preferred response: C


Rationale


The figure above represents the response of a normal heart to an increase in afterload. Loop 1 represents the normal physiologic pressure-volume loop. Stroke volume is diminished in loop 2 due to an increase in afterload. Loop 3 represents a compensatory response to the increase in afterload. Contraction begins at a higher end-diastolic volume. The heart ejects the same stroke volume at a higher afterload. Even though the stroke volumes for loops 1 and 3 are the same, the ejection fraction is lower for loop 3, since the end-diastolic volume is increased.


Chapter 24 : Regional and peripheral circulation




  • 1.

    The difference between autoregulated and maximal coronary flow is termed coronary flow reserve. What does the coronary flow reserve indicate?



    • A.

      Maximal coronary artery vasoconstriction to meet increased demands for oxygen


    • B.

      Maximal cardiac oxygen consumption


    • C.

      Maximal ventricular contraction at any given coronary flow


    • D.

      The amount of myocardial blood flow that can increase at any given pressure in order to meet increased oxygen demands




Preferred response: D


Rationale


At any given pressure, the difference between autoregulated and maximal flows is termed coronary flow reserve . Coronary flow reserve indicates how much extra flow the myocardium can get at a given pressure to meet increased demands for oxygen; if reserve is much reduced, then flow cannot increase sufficiently to meet demands and myocardial ischemia will occur. Coronary flow reserve is normally lower in the subendocardium than in the subepicardium, and decreases in coronary flow reserve are always more profound in the subendocardium than in the subepicardium.


If autoregulated flow is normal but maximal flow is decreased, then coronary flow reserve will be reduced. Coronary flow reserve also can be reduced if maximal flows are normal but autoregulated flows increase.



  • 2.

    Nitric oxide (NO) is a labile humoral factor produced by nitric oxide synthase from l-arginine in the vascular endothelial cell. Which of the following is true of nitric oxide?



    • A.

      Decreases significantly in the pulmonary vasculature immediately after birth


    • B.

      Is important in basal vascular tone, but less so for dynamic changes in vascular tone


    • C.

      Is increased in response to increases in shear/flow across endothelial cells


    • D.

      Produces vascular smooth muscle cell contraction by increasing the concentration of cGMP via the activation of soluble guanylate cyclase




Preferred response: C


Rationale


NO is a labile humoral factor produced by NO synthase from L-arginine in the vascular endothelial cell. NO diffuses into the smooth muscle cell and produces vascular relaxation by increasing concentrations of cGMP via the activation of soluble guanylate cyclase. NO is released in response to a variety of factors, including shear stress (flow) and the binding of certain endothelium-dependent vasodilators (such as acetylcholine, adenosine triphosphate [ATP], and bradykinin) to receptors on the endothelial cell. Basal NO release is an important mediator of both resting pulmonary and systemic vascular tone in the fetus, newborn, and adult, as well as a mediator of the fall in pulmonary vascular resistance normally occurring at the time of birth. Dynamic changes in NO release are fundamental to the regulation of all vascular beds.



  • 3.

    Of the following agents, which has the most potent vasoconstricting properties?



    • A.

      Angiotensin II


    • B.

      Aldosterone


    • C.

      Endothelin-1


    • D.

      Thromboxane A 2




Preferred response: C


Rationale


Endothelin-1 (ET-1) is produced by vascular endothelial cells. It has complex vasoactive properties, the most striking of which is its sustained hypertensive action. ET-1 is the most potent vasoconstricting agent discovered, with a potency 10 times that of angiotensin II.


Humoral regulators of vascular tone include angiotensin, arginine vasopressin, bradykinin, histamine, and serotonin. Of less importance are thyroxine, aldosterone, and antinatriuretic peptide. Angiotensin plays a special role in the homeostasis of blood pressure and is produced in persons with hemorrhagic or hypovolemic shock. It causes generalized vasoconstriction in both systemic and pulmonary circulations, but locally it stimulates the release of vasodilating prostaglandins in lung and kidney.


Bradykinin is a potent pulmonary and systemic vasodilator released locally by the action of the proteolytic enzymes on kallikrein after tissue injury.


Breakdown of phospholipids within vascular endothelial cells results in production of important byproducts of arachidonic acid, including prostacyclin (PGI 2 ) and thromboxane (TXA 2 ). PGI 2 activates adenylate cyclase, resulting in increased cyclic adenosine monophosphate (cAMP) production and subsequent vasodilation, whereas TXA 2 results in vasoconstriction via phospholipase C signaling.



  • 4.

    Which of the West zones of the lung are effectively perfused in proportion to their height above the left atrium?



    • A.

      Zones I and II


    • B.

      Zone II only


    • C.

      Zones I and III


    • D.

      Zone III only




Preferred response: B


Rationale


The inflow pressure of the pulmonary circulation is low, thus creating a vertical gradation to the distribution of blood flow in the lung. Hydrostatic pressure must be adjusted for vertical height above the left atrium, both at the inflow and at the outflow of every alveolar capillary unit. For example, given a pulmonary artery mean pressure of 20 cm H 2 O (zeroed at the level of the left atrium), an alveolar capillary unit 12 cm above the left atrium faces an inflow pressure of only 8 cm H 2 O. A left atrial pressure of 5 cm H 2 O generates no opposing outflow pressure to an alveolar capillary unit more than 5 cm above the left atrium. Critical closing pressure of postcapillary vessels therefore sets outflow pressure for a unit 10 cm above the left atrium. If intrinsic vascular resistance were identical throughout the lung, flow at any vertical height would be determined by hydrostatic driving pressure (inflow-outflow) and would be greatest at the base and least at the apex of the lung. This phenomenon partitions the lung into three vertical regions, named West zones. Zone I vessels are higher above the left atrium than pulmonary artery pressure and are not perfused by the pulmonary artery. Zone II vessels lie above the height defined by the hydrostatic left atrial pressure but below the height of the pulmonary artery pressure. These units are perfused in proportion to the driving pressure across them, which is approximately pulmonary artery pressure less vertical height (or critical closing pressure, whichever is higher). Zone III vessels lie at a vertical height less than outflow pressure. Driving pressure across these units is independent of height because inflow and outflow pressures are comparably influenced by gravity.



  • 5.

    In which of the following vascular beds do neural stimuli exert little effect on basal blood flow under physiologic conditions?



    • A.

      Cerebral


    • B.

      Coronary


    • C.

      Muscular


    • D.

      Pulmonary




Preferred response: A


Rationale


In marked contrast to other vascular beds, neural stimuli have relatively little effect on basal cerebral blood flow (CBF). Cerebral vessels display extensive perivascular innervations, especially the sympathetic nerves arising from the superior cervical sympathetic ganglia, but the brain is well protected from circulating catecholamines by the blood-brain barrier. Thus many of the vasoactive agents used in the critical care setting (α- and β-adrenergic agonists) have minimal effects on resting cerebral vascular tone. Mild to moderate electrical stimulation, as well as surgical resection of both the sympathetic and parasympathetic nervous system, does not alter cerebral vascular tone under resting conditions. However, vigorous sympathetic stimulation, as occurs with strenuous exercise or hypertension, does result in vasoconstriction of large- and medium-size cerebral vessels. Thus a neurogenic mechanism may not mediate cerebral vascular resistance under normal conditions, but it does provide protection at times of stress.



  • 6.

    Which of the following substrates can the brain use during periods of starvation?



    • A.

      Arginine


    • B.

      Glutamine


    • C.

      Glycogen


    • D.

      Ketones




Preferred response: D


Rationale


At rest, cerebral blood flow is approximately 50 mL/100 g tissue/min. Cerebral oxygen consumption is surprisingly high, averaging 3.2 mL/100 g tissue/min. Glucose is the primary energy substrate, although ketones can be utilized during periods of starvation. The brain has no functional capacity to store energy and thus is completely dependent on a steady supply of O 2 because up to 92% of its adenosine triphosphate (ATP) production results from the oxidative metabolism of glucose.



  • 7.

    Which of the following drugs should be used with caution in patients with increased intracranial pressure due to a risk of precipitating herniation?



    • A.

      Esmolol


    • B.

      Lorazepam


    • C.

      Nitroprusside


    • D.

      Pentobarbital




Preferred response: C


Rationale


Nitroprusside and other nitric oxide donor compounds can dilate cerebral vessels. This process greatly complicates the management of hypertension in patients with increased intracranial pressure. In such patients, nitroprusside may reduce arterial pressure but raise cerebral blood flow and blood volume, thereby causing herniation.



  • 8.

    Which of the following mediators has been implicated in the vasospasm following subarachnoid hemorrhage?



    • A.

      Adenosine


    • B.

      Carbon dioxide


    • C.

      Cyclic AMP (cAMP)


    • D.

      Endothelin-1 (ET-1)




Preferred response: D


Rationale


Substance P, acetylcholine, oxytocin, ET-1, adenosine diphosphate, ATP, and prostaglandin cause nitric oxide (NO)-dependent cerebral vasodilation. Impaired NO signaling is important in the pathophysiology of subarachnoid hemorrhage in which endothelial dysfunction has been well documented, leading to the important clinical problem of vasospasm.


ET-1 is an important mediator of cerebrovascular tone. Both ETA and ETB receptors have been identified in the cerebral vasculature. ET-1 given in high concentrations constricts cerebral vessels, probably via ETA receptor activation. However, ET-1 given in low concentrations relaxes cerebral vessels via endothelial cell ETB receptor activation, a response that is NO dependent. ET-1 has been identified as an important mediator of vasospasm following subarachnoid hemorrhage. ET-1 levels are increased following subarachnoid hemorrhage. In association with the increase in ET-1 levels are increases in ETA receptor levels, smooth muscle cell ETB receptor levels (which mediate vasoconstriction), and endothelin-converting enzyme activity.


Adenosine leads to cerebral vasodilation through an increase in cAMP and increases more than fivefold with hypoxia. Adenosine is critical in CBF autoregulation.


Carbon dioxide plays a critical role in regulation of CBF. A linear increase in CBF is seen with increasing Paco 2 , making CO 2 one of the most potent known cerebral vasodilators. Carbon dioxide exerts its effect via reduction in perivascular pH. Arterial H + cannot cross the blood-brain barrier, but CO 2 can easily diffuse into the brain. Perivascular acidosis dilates the cerebral vasculature, whereas alkalosis leads to vasoconstriction.



  • 9.

    In the normal heart, which of the following regions is more prone to ischemia because of low perfusion pressure?



    • A.

      Right ventricle subepicardial regions


    • B.

      Right ventricle subendocardial regions


    • C.

      Left ventricle subepicardial regions


    • D.

      Left ventricle subendocardial regions




Preferred response: D


Rationale


Myocardial perfusion over a cardiac cycle is normally approximately the same per gram of tissue in the outer (subepicardial), mid, and inner (subendocardial) layers of the left ventricle, but the dynamics during the cardiac cycle are complicated. At the end of diastole, when the ventricle is relaxed and tissue pressures are probably less than 10 mm Hg in any layer of the left ventricle, pressures in the intramural arteries are probably similar to each other and to aortic pressure. At the beginning of systole, tissue pressure rises to equal intracavitary pressure in the subendocardium but then falls off linearly across the wall to about 10 mm Hg in the subepicardium. These pressures are for an instant added to those inside the vessels because the vessels’ walls are not rigid, and as a result, intravascular pressures in subendocardial arteries exceed aortic pressures, but aortic pressures are higher than are pressures in subepicardial arteries. These pressure gradients and the greater shortening of subendocardial versus subepicardial muscle fibers during systole compress the subendocardial vessels and squeeze blood out of them both forward into the coronary sinus and backward toward the epicardium. In fact, narrowing of the subendocardial vessels facilitates thickening and shortening of the myocytes. This backflow enters the subepicardial arteries to supply their systolic flow. In systole, some forward flow into the orifices of the coronary arteries does indeed occur, but this forward flow does not perfuse the myocardium; it merely fills the extramyocardial arteries. In fact, there is often reverse flow in the epicardial coronary arteries. In early diastole, blood flows first into the subepicardial vessels that have not been compressed, but it takes longer to refill the narrowed subendocardial vessels. Given enough time and perfusing pressure, all the myocardium will be perfused, but if diastole is too short or perfusion pressure is too low, subendocardial ischemia occurs. Right ventricular myocardium, on the other hand, normally is perfused both in systole and in diastole, because of lower tissue pressures.



  • 10.

    Based on the graph for pressure-flow relations in the left coronary artery during normal flow and maximal vasodilation, which of the following statements is correct?



    • A.

      Coronary vascular reserve is independent of perfusion pressure.


    • B.

      For normal maximal flow, flow is uncoupled from metabolism.


    • C.

      For normal autoregulated flow, flow is uncoupled from metabolism.


    • D.

      For normal autoregulated flow, an increase in heart rate will increase maximal flow at any perfusion pressure.




Preferred response: C


Rationale


Coronary vascular resistance has three components: a basal low resistance in the arrested heart with maximally dilated vessels, an added resistance when vessels have tone, and a phasic resistance added whenever the ventricle contracts. In the beating heart with vessels maximally dilated by a pharmacologic dilator, the second of these resistances is absent. Perfusion of the left ventricular myocardium then produces a steep pressure-flow relation that is linear at higher flows but usually curvilinear at low pressures and flows ( Figure 136.7 ). Because the vessels are maximally dilated, flow is uncoupled from metabolism and depends only on driving pressure and resistance. If heart rate is increased, maximal flow at any perfusion pressure decreases because the heart is in a relaxed state for a smaller proportion of each minute.




• Fig. 136.7


If tone is allowed to return to the coronary vessels, then the pressure-flow relationship can be assessed at different perfusion pressures after cannulating the left coronary artery. It is necessary to do this because when cardiac metabolism and blood flow are coupled, increasing aortic blood pressure will increase coronary flow not only by increasing perfusion pressure but also by increasing myocardial oxygen demand. Under normal conditions coronary blood flow is autoregulated, such that if perfusion pressure is raised or lowered from its normal value, there is a range over which almost no change in flow occurs; a rise in pressure has caused vasoconstriction, and a fall in pressure has caused vasodilation. At perfusion pressures above some upper limit, flow increases, probably because the pressure overcomes the constriction. More important, at pressures below about 40 mm Hg (but varying, as discussed later), flow decreases predominantly in the deep subendocardial muscle ( Figure 136.8 ), indicating that some vessels have reached maximal vasodilation and can no longer decrease resistance to compensate for the decreased perfusion pressure. In these vessels, flow and pressure are directly related. If this pressure dependency occurs, then a further decrease in perfusion pressure decreases local blood flow below the required amount, or if myocardial oxygen demands increase at the same low perfusion pressure (as will occur if the ventricle becomes dilated), the requisite increase in flow will not occur. These two conditions cause subendocardial ischemia.




• Fig. 136.8


At any given pressure, the difference between autoregulated and maximal flows is termed coronary flow reserve. (Coronary flow reserve can be measured in units of mL/min but also can be assessed by a dimensionless flow reserve ratio derived by dividing maximal flow by resting flow.) Flow reserve depends on perfusion pressure because of the steepness of the pressure-flow relation in maximally dilated vessels. Coronary flow reserve indicates how much extra flow the myocardium can get at a given pressure to meet increased demands for oxygen; if reserve is much reduced, then flow cannot increase sufficiently to meet demands and myocardial ischemia will occur. What the figure does not show is that coronary flow reserve is normally lower in the subendocardium than in the subepicardium and that decreases in coronary flow reserve are always more profound in the subendocardium than in the subepicardium.



  • 11.

    Regarding coronary artery perfusion, a shift from normal autoregulated flow to increased autoregulated flow may be seen in which of the following conditions?



    • A.

      Myocardial ischemia


    • B.

      Coronary spasm


    • C.

      Increased blood viscosity


    • D.

      Carbon monoxide poisoning




Preferred response: D


Rationale


Coronary flow reserve also can be reduced if maximal flows are normal but autoregulated flows increase ( Figure 136.9 ). Increased myocardial flows above normal values can occur with exercise, tachycardia, anemia, carbon monoxide poisoning, leftward shift of the hemoglobin oxygen dissociation curve (as in infants with a high proportion of fetal hemoglobin), hypoxemia, thyrotoxicosis, acute ventricular dilation (because of increased wall stress), inotropic stimulation by catecholamines, and acquired ventricular hypertrophy. If autoregulated flow is normal but maximal flow is decreased, as indicated by the decreased slope of the pressure-flow relation during maximal dilation ( Figure 136.10 ), then coronary flow reserve will be reduced. Such a change can occur with marked tachycardia; a decrease in the number of coronary vessels due to small vessel disease, as in some collagen vascular diseases, especially systemic lupus erythematosus; increased resistance to flow in one or more large coronary vessels because of embolism, thrombosis, atheroma, or spasm; impaired myocardial relaxation due to ischemia; myocardial edema; a marked increase in left ventricular diastolic pressure; marked increase in left ventricular systolic pressure if coronary perfusion pressure is not also increased, as in aortic stenosis or incompetence; and an increase in blood viscosity, most commonly seen with hematocrit levels higher than 65%.



  • 12.

    Pulmonary embolism and subsequent right ventricular failure develops in a previously normal 17-year-old patient. Which of the following drugs would help restore right ventricular function to normal?



    • A.

      Dobutamine


    • B.

      Esmolol


    • C.

      Milrinone


    • D.

      Norepinephrine





• Fig. 136.9



• Fig. 136.10


Preferred response: D


Rationale


Right ventricular myocardial blood flow is affected by the low right ventricular systolic pressure and the fact that changes in aortic pressure alter coronary perfusion pressure without altering right ventricular pressure work. If the normal right ventricle is acutely distended—for example, by pulmonary embolism—eventually right ventricular failure occurs. The increased wall stress increases oxygen consumption but the raised systolic pressure reduces coronary flow, so when supply cannot match demand, right ventricular myocardial ischemia occurs. Raising aortic perfusing pressure mechanically or with α-adrenergic agonists increases right ventricular myocardial blood flow, relieves ischemia, and restores right ventricular function to normal. Improved coronary flow is not the only mechanism of this improvement; increased left ventricular afterload moves the ventricular septum toward the right ventricle and improves left ventricular performance.


Chapter 25 : Endothelium and endotheliopathy




  • 1.

    The endothelium plays an important role in controlling the vascular tone. Which of the following endothelium-derived factors is least likely to cause vasodilation?



    • A.

      Acetylcholine


    • B.

      Endothelin


    • C.

      Endothelium-derived hyperpolarizing factor


    • D.

      Nitric oxide (NO)




Preferred response: B


Rationale


Endothelin is an endothelium-derived vasoconstrictor. All other choices are endothelium vasodilators. The vasodilatory action of acetylcholine is derived through NO. Both NO and prostacyclin can be used clinically.



  • 2.

    Binding of thrombin to cell surface thrombomodulin facilitates activation of which of the following?



    • A.

      Antithrombin III


    • B.

      Platelets


    • C.

      Protein C


    • D.

      Protein S




Preferred response: C


Rationale


Thrombin activity is also modulated by endothelial cell synthesis of thrombomodulin. The binding of thrombin to thrombomodulin facilitates the enzyme’s activation of the anticoagulant protein C. Activated protein C (APC) activity is enhanced by cofactor C, also called protein S, which is synthesized by endothelial cells as well as other cells ( Figure 136.11 ). APC inhibits factor Va and factor VIIIa.




• Fig. 136.11


APC has a variety of antiinflammatory activities. It suppresses inflammatory cytokine during sepsis, inhibits leukocyte adhesion, decreases leukocyte chemotaxis, reduces endothelial cell apoptosis, helps maintain endothelial cell barrier function through activation of the sphingosine-1 phosphate receptor, and minimizes the decrease in blood pressure associated with severe sepsis.


Another interesting effect of APC is its plasminogen activator inhibitor 1 (PAI-1) neutralizing effect. PAI-1 is a glycoprotein that acts as an acute-phase protein during acute inflammation. Its primary role in vivo is the inhibition of both tissue- and urokinase-type plasminogen activators. PAI-1 is the most efficient inhibitor of APC and thrombin in the absence of heparin. PAI-1 competes with thrombomodulin for binding with thrombin, which, in combination with its inhibition of APC, makes it a strong local procoagulant by a combined action of displacing and inactivating anticoagulant thrombin from thrombomodulin. In this way PAI-1 has important pathophysiologic effects in acute and chronic diseases.



  • 3.

    Protein C acts by inhibiting the activities of which of the following?



    • A.

      Factors II and V


    • B.

      Factors II and Xa


    • C.

      Factors Va and VIIIa


    • D.

      Factors VIIa and Xa




Preferred response: C


Rationale


See Fig. 136.11 .



  • 4.

    Exposure and binding of endothelial tissue factor to which of the following factors initiates the coagulation cascade, resulting in the formation of fibrin?



    • A.

      Factor II


    • B.

      Factor Va


    • C.

      Factor VIIa


    • D.

      Factor VIII




Preferred response: C


Rationale


The expression and release of tissue factor is the pivotal step in transforming the endothelium from an anticoagulant to a procoagulant surface. Tissue factor accelerates factor VIIa–dependent activation of factors X and IX. The synthesis of tissue factor is induced by a number of agonists, including thrombin, endotoxin, several cytokines, shear stress, hypoxia, oxidized lipoproteins, and other endothelial insults. Once endothelial cells expressing tissue factor are exposed to plasma, prothrombinase activity is generated and fibrin is formed on the surface of the cells. Tissue factor also can be found in plasma as a soluble protein. Its role there is not well understood, but it probably plays a role in the initiation of coagulation.



  • 5.

    Which of the following causes the production of inducible NO synthase (iNOS)?



    • A.

      Bradykinin


    • B.

      Histamine


    • C.

      Insulin


    • D.

      Tumor necrosis factor-α




Preferred response: D


Rationale


NO is generated from the conversion of L-arginine to NO and L-citrulline by the enzyme nitric oxide synthase (NOS). Two general forms of NOS exist: constitutive and inducible. In the unstimulated state, NO is continuously produced by constitutive NOS (cNOS). The activity of cNOS is modulated by calcium that is released from endoplasmic stores in response to the activation of certain receptors. Substances such as acetylcholine, bradykinin, histamine, insulin, and substance P stimulate NO production through this mechanism. Similarly, shearing forces acting on the endothelium are another important mechanism regulating the release of NO. The inducible form of NOS is not calcium dependent but instead is stimulated by the actions of cytokines (e.g., tumor necrosis factor-α and interleukins) or bacterial endotoxins (e.g., lipopolysaccharide). Induction of iNOS occurs over several hours and results in NO production that may be more than 1000-fold greater than that produced by cNOS. This mechanism is important in the pathogenesis of inflammation (see Figure 136.12 ).



  • 6.

    Where is NO produced?



    • A.

      Endothelial cell


    • B.

      Neutrophils


    • C.

      Platelets


    • D.

      Red blood cell





• Fig. 136.12


Preferred response: A


Rationale


Furchgott and Zawadzki first postulated the existence of an endothelial relaxing factor in 1980, when they noticed that the presence of endothelium was essential for rabbit aortic rings to relax in response to acetylcholine. Later, it was determined that the biological effects of endothelial relaxing factor are mediated by NO. NO is generated in the endothelial cell from the conversion of L-arginine to NO and L-citrulline by the enzyme NOS.



  • 7.

    NO acts by which of the following mechanisms?



    • A.

      Binding to cyclic adenosine monophosphate (cAMP)


    • B.

      Binding to cyclic guanosine monophosphate (cGMP)


    • C.

      Binding to adenyl cyclase


    • D.

      Binding to guanylyl cyclase




Preferred response: D


Rationale


Once NO is formed by an endothelial cell, it readily diffuses out of the cell and into adjacent smooth muscle cells, where it binds to and activates the soluble form of guanylyl cyclase, resulting in the production of cGMP from guanosine triphosphate.



  • 8.

    Prostacyclin acts by which of the following mechanisms?



    • A.

      Binding to cAMP


    • B.

      Binding to cGMP


    • C.

      Binding to guanylyl cyclase


    • D.

      Binding to prostacyclin receptors




Preferred response: D


Rationale


Prostacyclin is a potent vasodilator that is active in both the pulmonary and systemic circulations. In addition to its vasodilatory effects, prostacyclin also has antithrombotic and antiplatelet activity. Its release may be stimulated by bradykinin and adenine nucleotides. Like NO, it is chemically unstable with a short half-life. However, unlike NO, prostacyclin activity in arterial beds depends on its ability to bind to specific receptors in vascular smooth muscle. Its vasodilator activity is therefore determined by the expression of such receptors. Prostacyclin receptors are coupled to adenylate cyclase to elevate cAMP levels in vascular smooth muscle. The increase in cAMP results in (1) stimulation of adenosine triphosphate–sensitive K + channels, resulting in hyperpolarization of the cell membrane and inhibition of the development of contraction, and (2) increased efflux of Ca 2+ from the smooth muscle cell and inhibition of the contractile machinery .



  • 9.

    Which of the following statements regarding the interaction between nitric oxide (NO) and prostacyclin is correct?



    • A.

      NO inhibits the action of prostacyclin.


    • B.

      NO and prostacyclin act sequentially to cause vasodilation.


    • C.

      Prostacyclin inhibits the action of NO.


    • D.

      Prostacyclin facilitates the cellular release of NO.




Preferred response: D


Rationale


Prostacyclin facilitates the release of NO by endothelial cells, and the action of prostacyclin in vascular smooth muscle is potentiated by NO . Interestingly, NO also may potentiate the effects of prostacyclin. The NO-mediated increase in cGMP in smooth muscle cells inhibits a phosphodiesterase that breaks down cAMP, therefore indirectly prolonging the half-life of the second messenger of prostacyclin.


Chapter 26 : Principles of invasive cardiovascular monitoring




  • 1.

    A critically ill 4-year-old child has been admitted to the pediatric ICU after endotracheal intubation, central venous catheter placement in the right internal jugular vein, and arterial catheter placement in the left radial artery. Which set of findings below most reliably predict that a fluid bolus will increase cardiac output?



    • A.

      A central venous pressure of 7 mm Hg while in normal sinus rhythm


    • B.

      A pulse pressure variation of 20% during deep sedation and paralysis


    • C.

      A pulmonary artery occlusion pressure of 12 mm Hg during deep sedation and paralysis


    • D.

      An increase of his mean arterial pressure by 10 when his liver is manually compressed




Preferred response: B


Rationale


Due to cardiopulmonary interactions, when a patient receives a positive pressure breath their right ventricular preload will decrease, an effect accentuated by hypovolemia. The result of this temporary decrease in right ventricular preload is a decrease in left ventricular preload which follows a few cardiac cycles later and manifests itself as a decrease in pulse pressure during insufflation of the lungs due to the decreased blood volume ejected.


A is incorrect as only central venous pressures <5 mm Hg have been reliably shown to predict an increase in cardiac output following a fluid bolus.


C is incorrect because a pulmonary artery occlusion pressure of 12 mm Hg during deep sedation and paralysis is higher than normal suggesting that the left atrium or ventricle is stressed, a condition likely to be worsened by increasing its volume load.


D is incorrect as there is insufficient evidence for manual liver palpation as a surrogate or replacement for passive leg raising, particularly without the explicit inclusion of sedation and paralysis. Liver compression may raise a child’s blood pressure due to pain, discomfort, anxiety, or increased alertness.



  • 2.

    In a well calibrated, well-functioning arterial line of appropriate size, cannulation of which site will report the largest pulse pressure?



    • A.

      Axillary artery


    • B.

      Brachial artery


    • C.

      Common femoral artery


    • D.

      Radial artery




Preferred response: D


Rationale


Since impedance increases in the circulatory system the further a measurement is taken from the heart, the blood pressure increases with distance from the heart while the flow velocity decreases. The effect of this will be to raise the systolic peak while accelerating the diastolic downslope of the pressure reading. Mathematically the area under this curve will remain the same, though the pulse pressure will vary depending on where it is measured, with the highest measurements being those taken most distally to the heart ( Figure 136.13 ).



  • 3.

    Which patient is most likely to display pulsus alternans on their arterial line tracing?



    • A.

      An infant newly diagnosed with anomalous left coronary artery from the pulmonary artery (ALCAPA)


    • B.

      A child on noninvasive positive pressure ventilation with community acquired pneumonia


    • C.

      A teenager with primary pulmonary arterial hypertension newly diagnosed with corona virus infection


    • D.

      A child who is postoperative day one from deceased donor renal transplant





• Fig. 136.13


Preferred response: A


Rationale


Pulsus alternans is an arterial line tracing which alternates between weak and strong beats and is associated with left ventricular systolic impairment. The systolic impairment results in an increased end-diastolic volume which leads to an increased stroke volume on the next cardiac cycle. The most likely patient to have left ventricular systolic impairment is the child with ALCAPA due to the deoxygenated blood from the pulmonary artery supplying their left coronary artery.



  • 4.

    Central venous pressure is most frequently used as a measure of which of the following?



    • A.

      Left ventricular preload


    • B.

      Left ventricular afterload


    • C.

      Right ventricular preload


    • D.

      Right ventricular afterload




Preferred response: C


Rationale


The central venous pressure is measured in the right atrium, super vena cava, or inferior vena cava and is the pressure at the end of diastole. Thus it represents the force, or load, on the right ventricle during diastole which is also called preload. Left ventricular preload cannot be measured from the central venous system, nor can the afterload of either ventricle.



  • 5.

    A 7-day-old male with d -transposition of the great arteries presents 8 hours after an arterial switch operation with a heart rate of 180 beats per minute and hypotension (mean arterial pressure of 35 mm Hg). He appears to have A-V dissociation on telemetry. What would be the most likely pattern seen on the central venous pressure (CVP) monitor?



    • A.

      Broad, flat a waves


    • B.

      Enlarged a waves


    • C.

      Fused c and v waves


    • D.

      Steep abrupt x and y descent




Preferred response: B


Rationale


Cannon a waves are enlarged a waves seen when the right atrium is ejecting against a closed tricuspid valve, and they may be seen when atrioventricular discordance occurs (in this case, during an episode of postoperative junctional ectopic tachycardia). They will resolve once the underlying illness is treated. Response C occurs in tricuspid regurgitation, when the backflow of blood out of the right ventricle obliterates the x descent and the c wave becomes accentuated. Response D occurs during pericardial constriction or tamponade physiology. Although the patient is tachycardic and hypotensive, there are no other signs, from the stem, that the patient has this physiology. Response A, or loss of a wave, occurs during atrial fibrillation because loss of atrial contraction results in missing the a wave ( Figure 136.14 ).



  • 6.

    An otherwise healthy infant has a peripheral arterial line placed in his dorsalis pedis for monitoring during an elective surgery. What discrepancy is likely to be found between a cuff blood pressure on the arm and the readings assessed from the arterial line?



    • A.

      The mean arterial pressure (MAP) measurements will be widely different.


    • B.

      The systolic blood pressure (SBP) measurement will be lower on the intraarterial measurement.


    • C.

      The diastolic blood pressure (DBP) measurement will be lower on the intraarterial measurement.


    • D.

      The MAPs will be the same, but the SBP on the cuff pressure will be higher.





• Fig. 136.14


Preferred response: C


Rationale


This is referred to as distal pulse amplification and occurs due to the nature of the vascular tree; the systolic blood pressure increases, and diastolic blood pressure decreases as you move peripherally, with a more exaggerated pulse pressure. The MAPs remain the same between the peripheral and central sites. Pressure waveform measurements from different sites of the arterial tree have varying morphologies depending on the properties of the vascular bed. The changes that occur as you move peripherally have to do with alterations in impedance and harmonic resonance.



  • 7.

    A 10-year-old child has been admitted to the intensive care unit (ICU) in shock. The child is endotracheally intubated, mechanically ventilated, and has a pulmonary artery catheter in place. The following information is available:




    • Heart rate: 140 beats/min



    • Blood pressure: 90/60 mm Hg



    • Central venous pressure (CVP): 10 mm Hg



    • Mean arterial pressure (MAP): 70 mm Hg



    • Pulmonary arterial pressure: 45/25 mm Hg



    • Mean pulmonary arterial pressure: 32 mm Hg



    • Venous oxygen saturation: 65%



    • Pulmonary artery occlusion pressure: 15 mm Hg



    • Carbon monoxide (CO): 3 L/min



    • What is the calculated systemic vascular resistance (SVR)?


    • A.

      1600 dynes sec/cm 5


    • B.

      30 Woods units


    • C.

      1200 dynes sec/cm 5


    • D.

      1200 mm Hg/m 2




Preferred response: A


Rationale


SVR is calculated by (MAP − CVP)/CO = (70 − 10)/3 = 60/3 = 20 in Woods units. Woods units are converted to dynes sec/cm 5 by a multiplicative factor of 80. Thus 20 × 80 = 1600 dynes sec/cm 5 .



  • 8.

    A 3-year-old child is brought to the emergency department with fever and increasing lethargy during the past 24 hours. On initial evaluation the child is minimally responsive and has poor distal perfusion with cool hands and feet. Blood pressure is 70/30 mm Hg, and oxygen saturation is 85% in room air. The child is intubated, central venous access is obtained, and intravenous fluids are administered. An established goal of continued resuscitation for this child is which of the following?



    • A.

      Achieve a pulmonary artery occlusion pressure of greater than 15 mm Hg


    • B.

      Achieve a central venous pressure (CVP) of 15 mm Hg or higher


    • C.

      Achieve a mixed venous saturation of 70% or higher


    • D.

      Achieve a mean arterial pressure (MAP) of 80 mm Hg or higher




Preferred response: C


Rationale


Achieving a mixed venous saturation of 70% or higher is one of the goals of early goal-directed therapy for persons with septic shock. Early goal-directed therapy has been shown to decrease mortality in persons with septic shock.


Chapter 27 : Assessment of cardiovascular function




  • 1.

    Two infants with dextro-transposition of the great arteries have had arterial switch operations earlier in the day. Approximately 6 hours postoperatively, infant A is on a milrinone infusion of 0.3 μg/kg/min with a mean arterial blood pressure of 45 mm Hg and a central venous pressure of 8 cm H 2 O. The last lactate level was 3.2 mmol/L (previously 4.5 mmol/L) and a recent mixed venous O 2 saturation is 75%. Infant B is on a milrinone infusion of 0.3 μg/kg/min with a mean arterial blood pressure of 45 mm Hg and a central venous pressure of 8 cm H 2 O. The last lactate is 3.2 mmol/L (previously 2.1 mmol/L) and a recent mixed venous saturation is 60%. Infant B has received a total of 20 mL/kg of crystalloid in the last 4 hours. Both infants have 100% arterial oxygen saturations and identical mechanical ventilation parameters. Based on the information above, which infant is at higher risk of clinical decompensation in the next 6 hours:



    • A.

      Infant A


    • B.

      Infant B


    • C.

      Both are at equal risk based on the information provided




Preferred response: B


Rationale


While both infants have identical vital signs and are on the same vasoactive infusions, infant B has received a higher “quantity of therapy” (QOT) due to the volume infusions in the last 4 hours to maintain the hemodynamics listed. In addition, infant B has a rising lactate and a lower mixed venous saturation (higher arteriovenous oxygen difference) which may signify inadequate oxygen delivery. Infant B should be carefully monitored, and additional investigations into the cause for the higher QOT should be considered.



  • 2.

    A 6-month-old infant with tetralogy of Fallot is recovering in the intensive care unit after a complete surgical repair. Approximately 6 hours after the operation is completed, the central venous pressure (CVP) is 15 mm Hg. The patient is hypotensive and receives 10 mL/kg of crystalloid. Which of the following is the least likely to be observed in the physiology described?



    • A.

      Fluid accumulation in the thoracic and abdominal cavities


    • B.

      Increased chest wall stiffness


    • C.

      Increased intrathoracic pressures


    • D.

      Increased peripheral and pulmonary edema


    • E.

      Increased venous return




Preferred response: E


Rationale


The patient described in this question with tetralogy of Fallot is likely to have significant right ventricular diastolic dysfunction resulting in high filling pressures (high CVP). Leaky vasculature as a result of inflammation due to cardiopulmonary bypass can result in significant third-spacing of fluid. A fluid bolus for hypotension causes increased tissue edema and fluid accumulation in the thoracic and abdominal cavities which results in increased chest wall stiffness and increased intrathoracic pressure. As a result, there is decreased venous return and therefore decreased preload to the right ventricle which results in further hypotension and diminished cardiac output.



  • 3.

    An infant with hypoplastic left heart syndrome is recovering after a Norwood Stage I procedure with a modified Blalock-Taussig shunt. The patient’s arterial oxygen saturation and mean arterial blood pressure are within normal limits. The infant has a lactate level that has been rising over the last 4 hours and a wide arteriovenous O 2 (AVO 2 ) saturation difference. Which of the following will not increase systemic oxygen delivery in this patient?



    • A.

      Addition of inhaled nitric oxide to improve pulmonary blood flow


    • B.

      Afterload reduction


    • C.

      Blood transfusion


    • D.

      Increased inotropic support with dobutamine


    • E.

      Sedation and paralysis




Preferred response: A


Rationale


This patient with single ventricle physiology has complete mixing of systemic and pulmonary venous blood. The rising lactate, minimal urine output, and widened AVO 2 difference suggested compromised oxygen delivery despite a normal oxygen saturation and mean arterial blood pressure. The addition of a pulmonary vasodilator to improve pulmonary blood flow would likely exacerbate the problem further by increasing pulmonary blood flow at the expense of systemic blood flow. In other words, the Qp:Qs ratio would increase. All of the other options would increase systemic oxygen delivery by improving Qs (options B and D), decreasing systemic oxygen demand (option E), or increasing oxygen carrying capacity and likely mixed venous saturation (option C).



  • 4.

    A 12-year-old girl is diagnosed with viral myocarditis and has an echocardiogram showing severely depressed biventricular systolic function. Her blood pressure as measured by invasive monitoring is normal for age. Which of the following factors is compatible with a relatively low quantity of therapy?



    • A.

      A central venous pressure of 6 mm Hg


    • B.

      A lidocaine infusion to suppress ventricular tachycardia


    • C.

      An epinephrine infusion


    • D.

      Need for mechanical ventilation


    • E.

      Two sodium bicarbonate boluses due to metabolic acidosis




Preferred response: A


Rationale


The patient is presenting with decreased systolic ventricular function in the setting of viral myocarditis. The cardiac output is low; however, the patient’s blood pressure is normal due to the compensatory rise in systemic vascular resistance (SVR). As a result of these factors, one would expect that the patient might require increased filling pressures, a need of inotropic support, and/or mechanical ventilation to provide positive pressure ventilation to decrease oxygen demands and decrease left ventricular afterload. In addition, a patient with myocarditis is at risk of ventricular arrhythmias due to the irritable myocardium. The presence of a metabolic acidosis can also signify poor systemic oxygen delivery. In this particular case, the presence of a low central venous pressure would be reassuring that the patient has not required fluid resuscitation and therefore a significant quantity of therapy. All of the other options would signify a patient that is more critically ill. In this scenario, it would be important to consider advanced therapies such as mechanical circulatory support if there are markers of poor oxygen delivery despite the interventions listed above.



  • 5.

    What is a primary determinant of systemic oxygen delivery (Do 2 )?



    • A.

      Hemoglobin concentration


    • B.

      Intrathoracic pressure


    • C.

      Oxygen consumption


    • D.

      Pulmonary venous oxygen content




Preferred response: A


Rationale


Do 2 = Cardiac output (CO) × arterial content of oxygen (Cao 2 ), where Cao 2 = (Sao 2 × Hgb × 1.36) + (Pao 2 × 0.0031). Therefore because hemoglobin concentration is a key variable that determines systemic arterial O 2 content, it also is a primary determinant of Do 2 .



  • 6.

    A 3-year-old child with hypoplastic left heart syndrome returns from the operating room following a lateral tunnel fenestrated Fontan procedure. The initial postoperative arterial blood gas shows an oxygen saturation of 80% on a F io 2 1.0 via the mechanical ventilator. Postoperative chest x-ray is unremarkable, and the echocardiogram shows good right ventricular systolic function, a patent 3-mm fenestration with right-to-left shunting, trace tricuspid regurgitation, no systemic outflow tract obstruction, and an unobstructed Fontan pathway with good flow to both pulmonary arteries. The patient is mildly hypotensive in sinus rhythm with a hemoglobin level of 15 gm/dL and no postoperative bleeding. What is the most appropriate initial management to improve blood pressure and oxygenation?



    • A.

      Cool the patient to decrease oxygen consumption.


    • B.

      Remove the patient from positive pressure mechanical ventilation.


    • C.

      Start an inotropic agent to improve myocardial contractility.


    • D.

      Transfuse with red blood cells to increase hemoglobin.




Preferred response: B


Rationale


Positive pressure mechanical ventilation decreases systemic venous return and increases pulmonary vascular resistance, both of which are detrimental to Fontan physiology. Spontaneous respiration with negative intrathoracic pressure will provide the optimal conditions for systemic venous return and decreased pulmonary vascular resistance resulting in improved pulmonary blood flow (oxygenation), preload, and cardiac output.



  • 7.

    A 3-year-old girl with history of a large ventriculoseptal defect is admitted to your pediatric intensive care unit after ventriculoseptal defect closure. A chest radiograph shows that the cardiothoracic ratio is 0.4. The cardiac apex is displaced upward. What is the most likely reason for these radiographic changes?



    • A.

      Large pericardial effusion


    • B.

      Left ventricular (LV) enlargement


    • C.

      Patent foramen ovale


    • D.

      Right ventricular (RV) enlargement




Preferred response: D


Rationale


The cardiothoracic ratio gives a quantitative estimate of cardiac size, which is obtained by dividing the transverse measurement of the cardiac shadow in the posteroanterior view by the width of the thoracic cavity. Cardiomegaly is present if this value is greater than 0.5 in adults and 0.6 in infants. Although useful for assessing LV enlargement, the cardiothoracic ratio is not as sensitive to RV enlargement. RV enlargement results in lateral and upward displacement of the cardiac apex on the posteroanterior view and filling of the retrosternal space on the lateral view. It is perhaps more important to know what the chest x-ray may not reveal: significant cardiac problems, such as constrictive pericarditis, acute fulminant myocarditis, and even acute pericardial tamponade are often associated with a normal-sized heart on a chest radiograph.



  • 8.

    A 1-year-old patient has a hemoglobin of 9 mg/dL and arterial oxygen saturation of 86%. The child’s arterial blood gas results reveal the following values: pH, 7.38; pco 2 , 53 mm Hg; and Pao 2 , 56 mm Hg. Cardiac output is estimated to be 3.2 L/minute. The child’s weight is 10 kg. What is the approximate quantity of oxygen delivered to the tissues in 1 minute (Do 2 )?



    • A.

      10 mL O 2 /kg/min


    • B.

      20 mL O 2 /kg/min


    • C.

      30 mL O 2 /kg/min


    • D.

      40 mL O 2 /kg/min




Preferred response: C


Rationale


Tissue oxygenation is directly related to both Do 2 and systemic arterial blood pressure. Do 2 , the quantity of O 2 delivered to the tissues per minute, is the product of systemic blood flow (SBF), which equals cardiac output except in patients with certain cardiac malformations, and arterial O 2 content:


DO2(mLminute)=10×CO(Lminute)×CaO2(mL100mL blood)
DO2(mLminute)=10×CO(Lminute)×CaO2(mL100mL blood)


where CO = cardiac output or SBF in L/min or L/min/m 2 and CaO 2 = quantity of O 2 bound to hemoglobin plus the quantity of O 2 dissolved in the plasma in arterial blood. The O 2 content of arterial blood (mL O 2 /dL blood) equals:


CaO2=(SaO2×Hgb×1.36)+(PaO2×0.003)
CaO2=(SaO2×Hgb×1.36)+(PaO2×0.003)


where Sao 2 = arterial O 2 saturation, Hgb = hemoglobin concentration (g/dL), 1.36 (constant) = amount of O 2 bound per gram of hemoglobin (mL) at 1 atmosphere of pressure, Pao 2 = arterial partial pressure of O 2 , and 0.003 (constant) = amount of O 2 dissolved in plasma at 1 atmosphere. The quantity of dissolved O 2 is generally considered to be negligible in the normal range of Pao 2 .


In this case, the Cao 2 is (9 × 1.36 × 0.86) + (0.0031 × 56) = 10.53 + 0.17 = 10.7 mL O 2 /100 mL. The Do 2 is therefore 10 × 3.2 × 10.7 = 342.4 mL O 2 /min or 34.24 mL O 2 /k.


Chapter 28 : Cardiac failure and ventricular assist devices




  • 1.

    A 5-year-old male has a 4-day history of persistent high fever (>38.5°C) and progressive lethargy. Mother reports that today he stopped eating and became tachypneic resulting in the child being transported to the emergency department. In triage, the child was noted to have the following vital signs: temperature, 38.8°C; heart rate, 140 beats per minute; respiration rate, 40 per minute; blood pressure, 75/60 mm Hg; Spo 2 , 85% in room air. The child appears to be in shock, with pallor, cyanosis, cold extremities with thready pulses, and he has an audible S3/S4 gallop rhythm with bilateral diffuse rales and hepatomegaly. The child was endotracheally intubated in the ED and ketamine and rocuronium were used during the procedure. The ED physician contacts you for additional recommendations and admission to the PICU. Your recommend the following NEXT IMPORTANT treatment for the child:



    • A.

      Begin vasopressin infusion to increase the child’s blood pressure to >100 mm Hg systolic


    • B.

      Begin epinephrine infusion to increase cardiac contractility, increase blood pressure to >85 mm Hg systolic, and improve central perfusion


    • C.

      Give an intravenous bolus of 20 mL/kg of normal saline


    • D.

      Start dexmedetomidine infusion to lower the child’s heart rate to <100 beats/min




Preferred response: B


Rationale


The patient is presenting in cardiogenic shock requiring increase in cardiac output that can be obtained by epinephrine use due to its B1 effects on contractility which should lower the heart rate and increase blood pressure without increasing systemic vascular resistance.



  • 2.

    The above child arrives in the PICU endotracheally intubated with two peripheral large bore IVs infusing the treatment you recommended above, and has the following vital signs: temperature, 38°C (after IV acetaminophen); heart rate, 120 beats per minute; respirations, 25 per minute (intubated on ventilator with F io 2 of 0.50); blood pressure, 95/50 mm Hg; Spo 2 , 98%. You immediately perform a point-of-care ultrasound (POCUS) of the heart from the apical four chamber view which reveals no tamponade but a left ventricular ejection fraction of 25% and normal proximal coronary arteries. An arterial blood gas is obtained revealing: pH 7.20; pco 2 , 35 mm Hg; po 2 , 90 mm Hg; base excess, −9.0 mmol/L; lactate level, 8.2 mmol/L (normal: 1-2); B type natriuretic peptide level, 4675 pg/mL (normal: <100 pg/mL). The complete blood count is normal. The child has a positive nasal rapid antigen test for SARS-CoV-2.




    • Over the next 4 hours the child progresses downward with a worsening metabolic acidosis, increasing lactate level (12.4 mmol/L), lower mixed venous saturation level (<40%); anuria and decreased central and peripheral pulses. A discussion with your pediatric cardiac surgeon and interventional cardiologist leads to the need for mechanical circulatory support for this child. The best device choice for this child is:


    • A.

      Percutaneously inserted Abiomed Impella 2.5 through the right femoral artery


    • B.

      Thoratec Pediamag Right Ventricular Assist Device (RVAD)


    • C.

      Veno-venous extracorporeal life support (VV-ECLS) with a double lumen catheter in the right internal jugular vein


    • D.

      Veno-arterial extracorporeal life support (VA-ECLS) through the right internal jugular vein and carotid artery




Preferred response: D


Rationale


Only VA ECMO will provide increased systemic cardiac output (Qs) in this patient with cardiogenic shock. Theoretically, the Impella heart pump could provide LV assist in older adolescent but not in a 5-year-old.



  • 3.

    The child stabilizes on the mechanical circulatory support you chose above but unfortunately after 10 days of aggressive therapy there is no recovery of cardiac function but all other organ function including neurologic status are intact. Your next BEST OPTION for this child’s long term survival is to:



    • A.

      Continue on the current therapy for an additional 2 weeks hoping for cardiac recovery


    • B.

      Continue on the current therapy but contact the heart transplant team to begin the process of listing the child status 1A


    • C.

      Convert the child over to a paracorporeal Berlin Heart Excor Biventricular Assist Device using two 25-mL pumps as a bridge to cardiac transplant


    • D.

      Speak to the family about the futility in continuing aggressive care and discontinue MCS




Preferred response: C


Rationale


VA ECMO has stabilized the child with preservation of the child’s other vital organs but this therapy is limited to short term use (despite a few outliers). For the child to be listed and receive a heart transplant, continued support would likely run months. Transitioning the child to a long term VAD is the best option for long term survival (to recovery or transplant). At this time ECLS/VAD are not futile or experimental therapies for this 5-year-old child.



  • 4.

    A previously well 8-kg, 6-month-old girl is referred from an outside hospital with frequent episodes of new-onset polymorphic ventricular tachycardia. She has been cardioverted several times and commenced on a lidocaine infusion. The episodes are becoming more frequent and associated with significant hypotension. Echocardiogram reveals extensive left ventricular noncompaction with an ejection fraction of 31%. No intracardiac thrombus is reported. A venous saturation of 51% is obtained from an existing right internal jugular central venous catheter. What would be the appropriate subsequent management?



    • A.

      Cannulate for venoarterial (VA) extracorporeal membrane oxygenation (ECMO) until arrhythmia control is achieved.


    • B.

      Maximize antiarrhythmic medication and initiate anticoagulation and milrinone infusion.


    • C.

      Place a left ventricular assist device (LVAD).


    • D.

      Place on VA ECMO as a bridge to longer-term biventricular assist and heart transplantation.




Preferred response: D


Rationale


Refractory ventricular tachycardia with clinically significant hemodynamic compromise would necessitate biventricular support. The patient has impaired ventricular function with evidence of inadequate tissue oxygen delivery. The underlying disease process is not amenable to medical therapy alone, and the anticipated duration of mechanical support prior to orthotopic heart transplant would be in the order of weeks to months.



  • 5.

    A 7-year-old, 22-kg boy is referred to the cardiology clinic with a 5-month history of increasing fatigue, exercise intolerance, anorexia, and dyspnea. His parents comment that his color is gray on minimal exertion. Echocardiography reveals a thin-walled, severely dilated left ventricle with an ejection fraction of 20%. There is moderate mitral regurgitation with an otherwise structurally normal heart and vasculature. A diagnosis of idiopathic dilated cardiomyopathy is made. He is admitted, a central line is placed, and he is started on a milrinone infusion and diuretics. He subsequently develops intermittent unifocal ventricular ectopy. After 24 hours, his parents comment on the improvement in his color, energy levels, and appetite. Listing for heart transplantation is considered. Venous oxygen saturation from his central line is initially 63%. Options for subsequent management include which of the following?



    • A.

      Discharge to home on milrinone with regular visits with the cardiologist.


    • B.

      Place on left ventricular assist device (LVAD) as a bridge to transplant.


    • C.

      Remain in the PICU with current regimen until a donor heart is available.


    • D.

      Wait for evidence of end-organ dysfunction or worsening symptoms of heart failure before intervening.




Preferred response: A


Rationale


This is a case of a different disease process in an older child where potential clinical improvement can occur in more than 50% of patients. Management will depend on response to initial therapy. Failure to improve clinically in association with evidence of inadequate tissue oxygen delivery will determine the timing of institution of mechanical support. The child’s size and expected duration of support precludes an adult VAD system or implantation of a short-term device.



  • 6.

    You are caring for a newborn who underwent an arterial switch operation for d -transposition of the great vessels. Eight hours after surgery, his vital signs are as follows: temperature, 37.8°C; heart rate, 165 beats per minute (sinus rhythm); left atrial pressure, 12 cm H 2 O; central venous pressure, 15 cm H 2 O; and blood pressure, 54/31 mm Hg. On physical examination, he has slightly cold extremities and pulses are 1+, breath sounds are clear bilaterally, the abdomen is soft, and the liver edge is palpated 3 cm below the right costal margin. Urine output has been 0.49 mL/kg/h for the past 4 hours. Arterial blood gas values show the following: pH, 7.23; pco 2 , 37 mm Hg; Pao 2 , 65 mm Hg; base deficit, 6; and lactic acid, 3.7 mmol/L. Which of the following interventions would potentially be detrimental to this patient?



    • A.

      Dopamine, 10 µg/kg/min


    • B.

      Epinephrine, 0.03 µg/kg/min


    • C.

      Fentanyl, 3 µg/kg/h


    • D.

      Milrinone, 0.5 µg/kg/min




Preferred response: A


Rationale


Low cardiac output state (LCOS) is commonly seen after cardiopulmonary bypass and is manifested by oliguria, followed by decreased perfusion, progressive acidosis, and hemodynamic compromise. The goal in management of LCOS is to provide adequate oxygen delivery (Do 2 ) to the tissues, which can be achieved by decreasing oxygen consumption or increasing Do 2 . Maintaining normothermia or mild hypothermia, providing adequate analgesia and sedation, and sometimes neuromuscular blockade are common interventions that will help decreased oxygen consumption.


Phosphodiesterase III inhibitors like milrinone are indicated in the postoperative management of LCOS. Milrinone favors calcium transport into the cell by increasing intracellular cyclic adenosine monophosphate. The increased intracellular calcium enhances the contractile state of the myocyte. Milrinone decreases systemic and pulmonary vascular resistance and improves diastolic relaxation by increasing the rate of calcium reuptake after systole. Although low-dose catecholamines are useful in improving contractility (via their β 1 -adrenergic effect), high-dose catecholamines (dopamine >5 µg/kg/min, epinephrine >0.05 µg/kg/min) will increase heart rate and increase systemic vascular resistance, leading to increased myocardial oxygen consumption, decreased cardiac output, and decreased Do 2 . If optimized medical management is unsuccessful in reverting LCOS, mechanical support would be indicated.


Chapter 29 : Echocardiographic imaging




  • 1.

    A patient undergoes surgical repair of tetralogy of Fallot with severe pulmonary stenosis. The postoperative transesophageal echocardiogram demonstrates no residual obstruction in the RV outflow track, with severe pulmonary insufficiency, and a hypertrophied right ventricle. On arrival to the ICU, the patient is noted to be cyanotic. Breath sounds are equal with appropriate readings on ventilator monitors. Which of the following would be an ideal early evaluation or intervention for this infant?



    • A.

      Increase the F io 2 on the ventilator to 100% and hope the cyanosis resolves.


    • B.

      Obtain an echocardiogram to evaluate for right-to-left shunting at the atrial level.


    • C.

      Request that the surgeon take the patient back to the operating room immediately.


    • D.

      Request that an interventionalist take the infant to the catheterization laboratory for device occlusion of a presumed right-to-left shunt.




Preferred response: B


Rationale


Echocardiography is useful (and the modality of choice) in the evaluation of the cyanotic patient, can often be quickly and easily obtained, and gives immediate answers on residual shunts in the postoperative patient. Prior to taking the infant back for revision of the operation, or to the catheterization laboratory for intervention, it is useful to understand the etiology of the cyanosis. An atrial level communication is at times left intentionally to allow a “pop-off” for the stiff right ventricle in the immediate postoperative period. A chest x-ray would be obtained prior to repositioning the ETT rather than assuming malposition in the setting of equal breath sounds. Increasing the F io 2 to 100% is not a long-term solution.



  • 2.

    Which of the following statements is true regarding echocardiography?



    • A.

      Echocardiography is limited to available acoustic windows.


    • B.

      Echocardiography is uniquely a noninvasive modality.


    • C.

      Echocardiography provides precise measurements of pulmonary arterial pressures.


    • D.

      Echocardiography can provide histologic evaluation of cardiovascular tissues.




Preferred response: A


Rationale


Echocardiography is limited to the “windows” where the ultrasound beam can penetrate adequately to produce images. It does not penetrate well through air. This can be a major limitation to echocardiography. The remaining statements are not true. Transesophageal (TEE) and intracardiac echo (ICE) are examples of invasive echocardiography. Echocardiography provides estimates of pulmonary arterial pressures, which can be inaccurate for many different reasons.



  • 3.

    What is the earliest gestational age that cardiac echocardiography can be used to assess fetal cardiac anatomy reliably?



    • A.

      8 weeks


    • B.

      14 weeks


    • C.

      20 weeks


    • D.

      28 weeks




Preferred response: C


Rationale


Complete anatomic and physiologic assessment can be obtained in the neonate and in the fetus at 20 weeks of gestation.



  • 4.

    Which of the following is the mainstay of the anatomic diagnosis of congenital heart disease?



    • A.

      Angiography


    • B.

      Cardiac catheterization


    • C.

      Computerized tomographic angiography


    • D.

      Echocardiography




Preferred response: D


Rationale


The technique of echocardiography and the practice of echocardiology have changed the practice of pediatric cardiology by largely replacing the use of cardiac catheterization/angiography for the diagnosis of congenital malformations. Combined with the use of prostaglandin for maintaining the patency of the ductus arteriosus, echocardiography has dramatically reduced the need for emergency cardiac catheterization in neonates. Most patients with congenital heart disease that is detected in the neonatal period can undergo palliative surgery without cardiac catheterization. Most definitive surgical repairs can be performed successfully without the risk of invasive studies. Pulsed, continuous-wave, color, and tissue Doppler have added important capabilities for anatomic and functional assessment. Intraoperative and postoperative management of congenital heart defects has been aided by the addition of transesophageal echocardiography.



  • 5.

    Right ventricular pressure can be estimated with the use of echocardiography by the simplified Bernoulli equation, where v is peak velocity of which insufficiency jet?



    • A.

      Pulmonary insufficiency jet 4 v


    • B.

      Pulmonary insufficiency jet 2 v


    • C.

      Tricuspid insufficiency jet 4 v 2


    • D.

      Tricuspid insufficiency jet 2 v 2




Preferred response: C


Rationale


Estimation of right ventricular systolic pressure by Doppler echocardiography is done with use of the maximum velocity ( v ) of the regurgitant tricuspid jet. The systolic pressure gradient (delta P) between the right ventricle and the right atrium is calculated by using the simplified Bernoulli equation (delta P = 4 v 2 ).



  • 6.

    Segmental analysis of cardiac disease is classically based on which of the following?



    • A.

      Great vessel arrangement, ventricular looping, situs


    • B.

      Situs, great vessel arrangement, ventricular looping


    • C.

      Situs, ventricular looping, great vessel arrangement


    • D.

      Ventricular looping, great vessel arrangement, situs




Preferred response: C


Rationale


Comprehensive analysis of cardiovascular anatomy requires a step-by-step segmental approach. A complete step-by-step approach to cardiac diagnosis includes the diagnosis of atrial situs; identification of the chambers and their interconnections; and systematic assessment of valves, septa, coronaries, systemic and pulmonary veins, and aortic anatomy. Imaging of the thymus and diaphragm is part of the detailed echocardiographic examination.


The segmental approach is based on the principle that all aspects of abnormal cardiovascular morphology can be broken down into discrete, mutually exclusive descriptors, allowing unambiguous delineation of any complex congenital malformation. The schema must include information on the presence, position, and connection of each cardiac segment. Classically, three segments have been recognized: atria, ventricles, and great arteries. By describing the anatomic segments and indicating the normality or abnormality of each, a complete description of the cardiac anatomy is possible. It now is possible to code cardiac anatomic abnormalities by segmental analysis.


Determination of cardiac position and atrial-visceral situs is a standard portion of the echocardiographic assessment of congenital heart disease and is the foundation of the segmental approach. Atrial situs and atrial morphology are diagnosed together, and four possibilities exist: solitus (normal), inversus, and heterotaxy that may be right atrial isomerism or left atrial isomerism.


Description of the connection of the atria and ventricles (i.e., AV connection) requires knowledge of both atrial and ventricular morphology. Four patterns of AV connection exist: concordant (i.e., normal); discordant; univentricular through a single inlet (i.e., tricuspid or mitral atresia), double inlet, or common inlet; and ambiguous (i.e., two ventricles with atrial isomerism). When the morphologic right atrium connects normally to the morphologic right ventricle and the left atrium connects to the left ventricle, AV concordance is present. When this connection is reversed and the morphologic right atrium connects to the morphologic left ventricle, AV connection is discordant and sometimes is referred to as ventricular inversion.


Ventriculoarterial connection is the manner in which the great arteries and semilunar valves connect to the ventricular outflow tracts. Normally, the morphologic right ventricle connects to the pulmonary valve and the morphologic left ventricle connects to the aortic valve. Four possibilities exist: concordant (i.e., normal), discordant (i.e., right ventricle to the aorta and left ventricle to the pulmonary trunk), double outlet (usually the right ventricle), and single outlet (i.e., aortic or pulmonary atresia or truncus arteriosus).



  • 7.

    Ventricular function can be assessed by the shortening fraction. Shortening fraction is calculated by which of the following equations?



    • A.

      End-diastolic dimension minus end-systolic dimension divided by end-diastolic dimension


    • B.

      End-diastolic dimension minus end-systolic dimension divided by end-systolic dimension


    • C.

      End-systolic dimension minus end-diastolic dimension divided by end-systolic dimension


    • D.

      End-systolic dimension minus end-diastolic dimension divided by end-diastolic dimension




Preferred response: A


Rationale


Echocardiography is a tomographic anatomic tool, but it also provides dynamic information about cardiac function and structure. Observations about the cardiac walls, including their movement, thickness, and degrees of shortening and thickening, can be extremely useful in determining segmental and global cardiac function. In general, the shortening fraction of the left ventricle should be at least 28% (end-diastolic minus end-systolic divided by end-diastolic dimension), and the walls of the left ventricle should move inward symmetrically.



  • 8.

    In congenital heart disease, the major application of contrast echocardiography in the postoperative patient is which of the following?



    • A.

      Assessment of ventricular function


    • B.

      Assessment of coronary arterial flow


    • C.

      Detection of residual shunt lesion


    • D.

      Evaluation of mitral regurgitation




Preferred response: C


Rationale


An ultrasonic contrast agent is a substance that stabilizes microbubbles in solution, which are large enough to reflect ultrasound but small enough that they disappear rapidly and are physiologically safe. The agent may be as simple as an injection of saline solution into the circulation during two-dimensional echocardiographic imaging or as complex as precision-engineered microbubbles of polysaccharide that dissolve in the circulation after injection. Advances in bubble technology allow imaging of myocardial capillary perfusion. Contrast also can be useful in defining the identity of an imaged structure. For example, a structure under the aortic arch may be confusing but can be confirmed to be the innominate vein by echocardiographic contrast injection in a left arm vein. In persons with congenital heart disease, the major application of contrast echocardiography is in the postoperative patient with residual shunts or as a means to exclude congenital heart disease. Systemic venous injection of contrast fills the right side of the heart sequentially, and the site of residual right-to-left shunting can be defined.



  • 9.

    The Ross procedure involves which of the following?



    • A.

      Autologous autograft in the pulmonary position and mechanical prosthesis in the aortic position


    • B.

      Autologous autograft in the aortic position and mechanical prosthesis in the pulmonary position


    • C.

      Mechanical valve in the pulmonary position and autograft in the aortic position


    • D.

      Placement of a pulmonary autograft in the aortic position




Preferred response: D


Rationale


The Ross procedure involves the substitution of the diseased aortic valve with the patient’s own pulmonary valve (autograft). A pulmonary homograft is then used to replace the patient’s pulmonary valve. The coronary arteries are translocated to the new aortic valve. Advantages of this procedure include the lack of need for anticoagulation because of the decreased incidence of thromboembolism, as well as growth of the valve as the child grows. One disadvantage is the treatment of one-valve disease with a two-valve replacement.


Chapter 30 : Diagnostic and therapeutic cardiac catheterization




  • 1.

    A 2-week-old full term female with hypoplastic left heart syndrome has delayed convalescence following Norwood procedure with modified Blalock-Taussig shunt and is referred for cardiac catheterization. The patient has baseline hemoglobin of 14 mg/dL and estimated oxygen consumption of 160 mL/minute/m 2 . The following saturations were obtained while intubated at a fraction of inspired oxygen of 21%: descending aorta, 88%; left upper pulmonary vein, 98%; mixed venous saturation obtained in the superior vena cava, 61%. What is the patient’s estimated ratio of pulmonary to systemic blood flow (Qp:Qs)?



    • A.

      0.4


    • B.

      0.7


    • C.

      1


    • D.

      2.7


    • E.

      Cannot calculate with the given data




Preferred response: D


Rationale


The Fick method uses a patient’s oxygen consumption and oxygen carrying capacity across a vascular bed to calculate blood. In patients with single ventricle physiology, the systemic and pulmonary saturations are identical. For this reason, only three saturations are required for calculating Qp:Qs via the Fick method. As per equations in Table 30.1 , the equation to calculate Qp:Qs can be simplified to include only saturations as many of the terms cancel out through division. For this question, Qp:Qs is calculated as: (systemic saturation – mixed venous saturation) ÷ (pulmonary venous saturation – pulmonary artery saturation). Since the systemic and pulmonary arterial saturations are equivalent, (88 − 61) ÷ (98 − 88) = 2.7. Note that a saturation of 88% in a patient with hypoplastic left heart syndrome after stage I palliation suggests significantly increased Qp:Qs as is demonstrated here.



  • 2.

    A 12-year-old male with no prior history of cardiac disease is admitted to the pediatric intensive care unit with concerns for shock. He has a history of febrile illness with malaise several weeks prior. The patient is tachycardic and tachypneic with a pale appearance. The first Korotkoff sound is heard intermittently with respiration at a systolic blood pressure of 85 mm Hg and consistently at 72 mm Hg. On exam, the patient has weak pulses throughout, jugular venous distension, and distant heart sounds. The electrocardiogram demonstrates sinus tachycardia with beat-to-beat variation in the QRS amplitude. An echocardiogram shows a large pericardial effusion with respiratory variation in mitral inflow by pulse wave Doppler of >20%. Given concern for tamponade, the patient is prepared for a pericardiocentesis. For this patient, select the finding that is NOT representative of tamponade physiology:



    • A.

      Hypotension


    • B.

      Jugular venous distension


    • C.

      Large pericardial effusion


    • D.

      Pulsus paradoxus


    • E.

      All of the above are required for tamponade physiology




Preferred response: C


Rationale


If the pericardial effusion accumulates slowly, patients can remain hemodynamically stable and potentially asymptomatic even with a large pericardial effusion. Tamponade is a clinical diagnosis that is characterized by Beck’s triad: distant heart sounds, hypotension, and jugular venous distension. Patients with tamponade also have tachycardia, tachypnea, narrow pulse pressures, and pulsus paradoxus. The ECG and echocardiogram changes may be suggestive of a diagnosis, but must be accompanied by the clinical signs and symptoms to meet a diagnosis.



  • 3.

    A 2-year-old admitted with tachypnea is referred for catheterization after an echocardiogram demonstrated right heart enlargement and partial anomalous pulmonary venous return. The patient’s baseline hemoglobin is 12.5 mg/dL. The patient is sedated and intubated with fraction of inspired oxygen of 21%. Estimated oxygen consumption (Vo 2 ) is 150 mL/min/m 2 and the following saturations are obtained:




















    Innominate Vein High SVC Right Atrium Pulmonary Artery Pulmonary Vein Descending Aorta
    71% 70% 85% 86% 99% 99%



    • Calculate the pulmonary blood flow for this patient.



      • A.

        2.5 L/min/m 2


      • B.

        4.1 L/min/m 2


      • C.

        6.8 L/min/m 2


      • D.

        10 L/min/m 2


      • E.

        Unable to calculate with the given information





Preferred response: C


Rationale


As per Table 30.1 , pulmonary blood flow can be calculated as Vo 2 /10 × [1.36 × Hgb × (pulmonary vein saturation – pulmonary artery saturation)]. For the question, 150 / [10 × 1.36 × 12.5 × (0.99−0.86)] = 150/(13.6 × 12.5 × 0.13) = 6.8 L/min/m 2 .



  • 4.

    A 10-month-old male is admitted for evaluation of persistent desaturations requiring nasal cannula support. He has a large ventricular septal defect. Despite poor outpatient follow-up, his growth has been normal. Infectious work up is unremarkable. He is referred for catheterization to more clearly delineate the source of desaturations and evaluate pulmonary vascular resistance prior to surgical referral. During the catheterization, the pulmonary venous saturation was 95% and pulmonary vascular resistance was 7 Wood units × m 2 at fraction of inspired oxygen (F io 2 ) 0.21. At 1.0 F io 2 and 20 ppm inhaled nitric oxide, the pulmonary vascular resistance decreased to 4 Wood units × m 2 . Per AHA guidelines, what would be the most appropriate step in cardiac management?



    • A.

      Initiate pulmonary vasodilator therapy with sildenafil and repeat catheterization prior to surgical repair.


    • B.

      Initiate pulmonary vasodilator therapy for symptom management; however, he is not a surgical candidate due to pulmonary hypertension.


    • C.

      Manage with diuretics as pulmonary vasodilator therapy is contraindicated.


    • D.

      Refer for surgery, consider fenestrated patch repair and anticipate need for postoperative pulmonary hypertension therapy.




Preferred response: D


Rationale


Per recent AHA guidelines by Abman et al. entitled Pediatric Pulmonary Hypertension, patients younger than 1 year of age with simple shunting lesions and concern for increased pulmonary vascular resistance should undergo catheterization with acute vasodilator testing. For patients with indexed pulmonary vascular resistance greater than 6 Wood units × m 2 , vasodilator testing is recommended. If the pulmonary vascular resistance improves with testing, then it would be reasonable to refer the patient for surgical repair.



  • 5.

    A 6-month-old infant is 4 days into the postoperative period after tetralogy of Fallot repair. There are persisting signs of right heart failure with hepatomegaly and ascites. An echocardiogram has inadequate windows to visualize the branch pulmonary arteries. What is the most appropriate management?



    • A.

      Communicate to the cardiac surgeon and interventional cardiologist and advocate for a diagnostic catheterization and possible pulmonary artery intervention if significant residual stenotic lesions are diagnosed.


    • B.

      Communicate to the cardiac surgeon and interventional cardiologist and advocate for a diagnostic catheterization and possible ASD closure.


    • C.

      Continue to medically manage with the goal of spontaneous right heart recovery.


    • D.

      Transfer the infant to the catheterization laboratory for pulmonary artery angiography.




Preferred response: A


Rationale


Communication is essential. The cardiac surgeon on a recent surgical patient will know the child’s anatomy intimately and also have an awareness of possible problems. The interventional cardiologist can advise on what is technically feasible. As the ICU attending, involving all providers is crucial for maximizing outcomes.


Arranging a catheterization without consultation will result in a poorly planned and inefficient catheterization that may lead to unnecessary angiograms and worse outcomes. Right heart failure can spontaneously improve early after cardiac repair, but if recovery is prolonged 72 hours longer than anticipated, then strong consideration for residual cardiac defects is very important. There are now reports on safe early postoperative interventions, thus a catheterization for merely diagnostic purposes would be incorrect. Closing an ASD in the setting of right heart failure would worsen cardiac output and elevate central venous pressure. In fact if there is an intact atrial septum, one of the interventions to consider is creating an ASD.



  • 6.

    When should cardiac surgical patients requiring venoarterial (VA) extracorporeal membrane oxygenation (ECMO) in the postoperative period be considered for cardiac catheterization?



    • A.

      If there is left ventricle (LV) dysfunction, left atrial (LA) dilation, and pulmonary edema on chest radiograph


    • B.

      Never; cardiac catheterization due to risks of the procedure and transportation is contraindicated when cannulated for VA ECMO


    • C.

      Only as a last resort and should not be considered unless unable to be decannulated after 2 weeks


    • D.

      Should not be considered unless unable to be decannulated after 1 week




Preferred response: A


Rationale


Left atrial decompression can be safely achieved in the catheterization laboratory and is important in the setting of LV dysfunction with distention. Decompression of the LA with interventional techniques (placing a cannula in the LA, creating an ASD) alleviates this problem. The other option is placement of a LA cannula by cardiac surgery, and this may be preferable in a small child with an open chest.


Safe transport is challenging but should not be the deciding factor. Catheterization on VA ECMO requires a high degree of teamwork, particularly with transport to ensure that cannula position remains stable. Safe transportation is a reality in many centers. Procedural risks even with anticoagulation are minimal. It is important to have a high index of suspicion for residual lesions in postoperative cardiac surgery patients who cannot be weaned from ECMO. Unless there is clear evidence for myocardial recovery, consideration for catheterization by the ICU attending and direct consultation with cardiac surgery and interventional cardiology should occur early and within 72 hours of the start of the unexpected course. Interventions can be performed on VA ECMO and would be indicated if any residual lesions are diagnosed.



  • 7.

    For an early postoperative patient who has persistent low cardiac output and is unable to be weaned from the ventilator, cardiac catheterization is characterized by which of the following?



    • A.

      Carries a significantly increased risk of serious adverse events when compared with late postoperative catheterization


    • B.

      May yield important physiologic and anatomic information that often leads to reintervention


    • C.

      Should only be performed on a patient supported with extracorporeal membrane oxygenation (ECMO)


    • D.

      Should be deferred until 6 weeks following surgery




Preferred response: B


Rationale


Cardiac catheterization should always be considered in the early postoperative period when a patient exhibits a persistent low output state or failure to wean from mechanical ventilation. Early postoperative catheterization often yields important physiologic and anatomic information and frequently leads to reintervention. Whereas elective catheter-based interventions are typically deferred until 6 weeks following surgery, the risk-benefit ratio favors early postoperative catheterization for the patient who does not follow a projected course. ECMO is used in postoperative patients who fail to separate from bypass or have progressive low output. These patients often require early postoperative catheterization to assess unfavorable physiology. In addition, ECMO support is often used to safely support the circulation in patients for whom a high-risk catheter intervention is planned. In the hands of an experienced team (i.e., interventional cardiologist, intensivist, anesthesia personnel, and nursing staff), serious adverse event rates are similar for early or late postoperative studies.



  • 8.

    A balloon atrial septostomy in the preoperative patient with d-TGA is characterized by which of the following?



    • A.

      Increases left atrial hypertension


    • B.

      May increase the risk for pulmonary hypertension prior to cardiac surgery


    • C.

      May be performed under fluoroscopic or echocardiographic guidance


    • D.

      Worsens cyanosis by facilitating mixing at the atrial level




Preferred response: C


Rationale


Balloon atrial septostomy should be considered in any newborn with a confirmed diagnosis of d-TGA who demonstrates inadequate mixing at the atrial level. Signs of inadequate mixing include cyanosis, left atrial hypertension, and pulmonary hypertension. Balloon atrial septostomy improves cyanosis by facilitating mixing at the atrial level, reduces left atrial hypertension, and may reduce the risk for pulmonary hypertension prior to cardiac surgery. Balloon atrial septostomy may be performed in the catheterization laboratory (under fluoroscopic guidance) or in the intensive care unit (under echocardiographic guidance) at the discretion of the intensivist and cardiac interventionist.


Chapter 31 : Pharmacology of the cardiovascular system




  • 1.

    A 4-month-old, 4 kg male with Trisomy 21, complete atrioventricular septal defect (AVSD), congestive heart failure, and failure to thrive is admitted to the pediatric intensive care unit (PICU) following surgical repair. The intraoperative course was complicated by a brief cardiopulmonary arrest, attributed to a pulmonary hypertensive crisis on separation from cardiopulmonary bypass. Spontaneous circulation was restored following 2 minutes of cardiopulmonary resuscitation. Milrinone was administered in 2 divided doses of 25 μg, each over 20 minutes, and infusions of epinephrine, 0.1 μg/kg/min, and milrinone, 0.5 μg/kg/min, were initiated. Nitric oxide at 10 ppm was added, and the patient was transferred to the PICU for further management.




    • Vital signs on arrival are remarkable for a heart rate of 156 beats per minute and arterial blood pressure of 70/28 mm Hg. Oxygen saturation is 100% on F io 2 1.0 and full ventilator support. The central venous pressure is 4 mm Hg.



    • On physical examination the infant is intubated, sedated, and medically paralyzed. His skin is warm and pink, with pulses 1+ to 2+ in all 4 extremities and capillary refill time of 3 seconds. A soft diastolic rumble is present on cardiac auscultation. The lungs are clear. The liver edge is palpable 2 cm below the right costal margin. Pupils are 5 mm and sluggishly reactive to light bilaterally. The neurologic exam is otherwise obscured by neuromuscular blockade.



    • The postoperative chest radiograph shows a well-positioned endotracheal tube and internal jugular vascular catheter, mild cardiomegaly and clear, well inflated lungs.



    • Laboratory data reveal pH 7.38; Paco 2 , 36 mm Hg; Pao 2 , 223 mm Hg; HCO 3 , 22 mmol/L, base excess, −3 mmol/L; ionized calcium, 1.6 mmol/L (reference range 1.2–1.3 mmol/L); lactate, 1.9 mmol/L (reference range <2 mmol/L); creatinine 1.6 mg/dL (reference range 0.2–0.6 mg/dL); hemoglobin 12 g/dL (reference range 9.0–13 g/dL)



    • Ninety minutes later, the patient’s arterial blood pressure is 50/20 mm Hg, the heart rate is 182 per minute, and there has been only 1.5 mL of concentrated urine output from the urinary catheter since PICU admission. The bedside monitor shows sinus tachycardia and the physical exam is otherwise unchanged. After re-zeroing the arterial catheter system, the repeat blood pressure is the same.



    • The most appropriate initial intervention is to:


    • A.

      Confirm the blood pressure via sphygmomanometer


    • B.

      Consult Cardiology for recommendations


    • C.

      Increase the epinephrine infusion to 1 μg/kg/min


    • D.

      Infuse 40 mL Ringer’s lactate over 5 minutes


    • E.

      Start a vasopressin infusion 0.0008 U/kg/min




Preferred response: D


Rationale


An infusion of 10 mL/kg of isotonic fluid should be initiated and the response closely monitored. The observation that the patient’s perfusion appeared unchanged is due to exaggerated peripheral vasodilation that can occur with milrinone, along with a generalized sunburned appearance. This can be mistaken for adequate perfusion if consideration is not given to milrinone as the culprit.


Although following blood pressure cuff readings may be helpful in certain scenarios, there is no reason to discount the invasive arterial blood pressure reading once the system has been interrogated and no problems have been identified.


Increasing the epinephrine dose to 1 μg/kg/minute would be very poorly tolerated in this scenario. The patient is already tachycardic, and the risk of dysrhythmias is a known complication following repair of complete AVSD. In addition, the increased afterload to the left ventricle following closure of the ventricular septal defect and correction of mitral insufficiency would only be compounded by the increase in systemic vascular resistance (SVR) from this dose of epinephrine. For the same reason, adding a vasopressin infusion would not be the best choice. Although the oliguria in this scenario is likely due to a decrease in SVR, the resulting “relative” hypovolemia indicates that gentle fluid repletion should be initiated to optimize organ perfusion prior to increasing vascular tone with a vasoconstrictor.



  • 2.

    While alerting the cardiologists to the change in the patient’s status is an important priority, waiting for their input will delay treatment of a problem that requires immediate attention. The effects of cardiopulmonary bypass and the associated inflammatory response, plus the impact of intraoperative events, combined with the major change in circulatory physiology and potential for adverse medication effects must all be considered and quickly addressed in order to ensure optimal outcomes.




    • The most likely reason for the hypotension is:


    • A.

      Anaphylaxis from perioperative, prophylactic cefazolin


    • B.

      Cumulative effect of the milrinone loading dose and infusion


    • C.

      Equipment error


    • D.

      Septic shock due to a urinary tract infection




Preferred response: B


Rationale


This patient’s presentation on admission to the PICU with widened pulse pressure with mild diastolic hypotension and prolonged capillary refill time despite the appearance of adequate perfusion is consistent with the peripheral vasodilation that can be seen in some patients on milrinone infusions, especially following a full loading dose. The response may be exaggerated in patients with mild acute kidney injury (such as this one) due to delayed drug clearance and those who are relatively volume depleted as was this patient. In the scenario presented here, renal hypoperfusion during cardiopulmonary bypass compounded by the brief cardiopulmonary arrest led to renal insufficiency. Approximately 85% of milrinone is cleared by the kidney. This property of the drug, along with its relatively long elimination half-life may result in accumulation of the medication, the consequences of which may persist for several hours. The need for dose adjustment should be evaluated for all medications whose effects may be intensified or prolonged by delayed renal clearance (e.g., certain sedatives and neuromuscular blocking agents).



  • 3.

    Once the child’s hemodynamic status improves, which of the following is most likely to prevent recurrence of the hypotension?



    • A.

      Decreasing the epinephrine to 0.05 μg/kg/min


    • B.

      Decreasing the milrinone to 0.25 μg/kg/min


    • C.

      Increasing the intravenous fluids to twice maintenance


    • D.

      Increasing the nitric oxide to 20 ppm


    • E.

      Increasing the dose of neuromuscular blocking agent




Preferred response: B


Rationale


Continuing the milrinone infusion is the best action, albeit at a lower rate, due to the patient’s history of congestive heart failure and pulmonary hypertension, along with the change in loading conditions following closure of the ventricular septum and recent cardiopulmonary arrest. In addition, there is evidence for improved outcomes in infants with postoperative cardiac dysfunction with administration of milrinone in the early postoperative period.


Increasing the intravenous fluids to twice maintenance in this patient with kidney injury will likely do little to prevent the hypotension and instead worsen postoperative anasarca, while also increasing the workload on the heart. Likewise, increasing the nitric oxide, decreasing the epinephrine infusion rate, or increasing the neuromuscular blockade will not prevent recurrence of the hypotension.


Following initial fluid administration and stabilization of hemodynamics, gentle up -titration of the epinephrine infusion may then be indicated to achieve optimal perfusion and to limit the number of fluid boluses.



  • 4.

    A 4-month-old male with Down syndrome is admitted to the PICU for postoperative care following patch repair of a large ventricular septal defect (VSD). He was managed preoperatively for congestive heart failure with furosemide and enalapril. His vital signs on admission to the PICU are significant for a temperature of 38.2°C; heart rate, 170 beats per minute; blood pressure, 86/38 mm Hg via a right radial arterial catheter (mean arterial pressure, 54 mm Hg); and oxygen saturation of 89% by pulse oximetry (with a probe on his great toe). On exam, his extremities are mottled and cool distally, with a capillary refill of 2.5 seconds. A bedside arterial blood gas suggests adequate oxygenation and ventilation. The arterial lactate is 6 mmol/L. You observe only a marginal change in his perfusion after three fluid boluses of 10 mL/kg. The central venous pressure is now 14 mm Hg. What is the most appropriate next step in his management?



    • A.

      Administer an intravenous (IV) milrinone bolus of 50 µg/kg over 15 minutes followed by a continuous infusion of milrinone at 0.5 µg/kg/minute.


    • B.

      Continue to bolus with fluids, but change to albumin for more effective intravascular volume repletion.


    • C.

      Start epinephrine at 0.5 µg/kg/minute, titrating to achieve a goal mean arterial pressure of 65 mm Hg.


    • D.

      Start dopamine at 15 µg/kg/minute, and overdrive pace to ensure atrioventricular synchrony.




Preferred response: A


Rationale


This patient is exhibiting signs of postoperative, decompensated cardiogenic shock, with tachycardia, cool and mottled extremities, prolonged capillary refill, and an elevated lactate, all of which indicate inadequate cardiac output. The decreased saturation via pulse oximetry, while the arterial blood gas demonstrates adequate oxygenation, is a function of distal vasoconstriction, which impacts the ability of the pulse oximeter probe to provide an accurate reading. Fluid resuscitation was attempted to augment preload to optimize this patient’s ventricular filling (to take advantage of the Starling relationship), but it did not restore perfusion because there is significant myocardial dysfunction. Therefore the best intervention is to start milrinone, which provides both inotropy and afterload reduction, without significantly increasing myocardial oxygen demand. If a patient’s blood pressure and hemodynamics allow, a loading dose is given before initiating the infusion.


A catecholamine such as epinephrine, as a sole agent and initiated at the dose provided in the scenario, would not be the best option for several reasons. First, the patient is already tachycardic, and epinephrine at 0.5 µg/kg/minute may increase the heart rate further and increase myocardial oxygen demand in a heart whose function is already impaired. Escalating this patient’s heart rate would also decrease the diastolic interval for ventricular filling, thus compromising cardiac output even further.


In addition, patients who undergo repair of large ventricular septal defects are at increased risk for developing junctional ectopic tachycardia (JET), a tachyarrhythmia unique to the postoperative state in children, in which the loss of atrioventricular synchrony compromises cardiac output. Patients in whom this rhythm is poorly tolerated may require treatment with antiarrhythmics and active cooling if conservative measures such as avoiding fever and minimizing catecholamine-induced increases in adrenergic tone fail.


Response B would not be the most appropriate next step because the central venous pressure is already elevated due to ventricular dysfunction, and rather than augmenting cardiac output, additional fluid boluses would likely produce pulmonary edema.


High-dose dopamine would not be the best choice for similar reasons as those stated for epinephrine. In addition, at a dose of 15 µg/kg/minute, dopamine exhibits strong α-adrenergic (vasoconstrictive) properties, which would increase afterload and compromise distal perfusion even further. The consequences of increasing heart rate with overdrive pacing were detailed earlier.



  • 5.

    Norepinephrine ___________ the heart rate. This action is mediated by ____________. Therefore ____________ would be an effective agent for reversing this response.



    • A.

      Increases, the hypothalamus, isoproterenol


    • B.

      Decreases, a reflex response, atropine


    • C.

      Decreases, a vagal response, isoproterenol


    • D.

      Has no effect on, the sinus node, ventricular pacing




Preferred response: B


Rationale


Norepinephrine exhibits strong α-adrenergic properties at doses used to reverse vasodilatory (hyperdynamic) shock. The resulting increase in vascular tone triggers a compensatory vagally mediated reflex slowing of the heart, which atropine will reverse via a muscarinic receptor blockade. Neither the hypothalamus nor the sinus node has been implicated.



  • 6.

    You have decided to add an epinephrine infusion to the management of an 11-year-old girl with myocarditis. Your goal is to improve cardiac contractility via epinephrine’s effect on β receptors. A medical student on call with you asks why epinephrine is more selective for β receptors than norepinephrine. How do you respond to this question?



    • A.

      Because of an increased number of hydroxyl groups on the phenylethylamine core


    • B.

      Because of a decrease in size of the substituent on the amino group


    • C.

      Because of an increase in size of the substituent on the amino group


    • D.

      Because of a lack of hydroxyl group at position 4 on the phenylethylamine core




Preferred response: C


Rationale


Increasing the size of the substituent on the amino group enhances β selectivity, while decreasing the size favors α selectivity. Hydroxyl groups are the same in terms of number and location for all catecholamines.



  • 7.

    Catecholamines and phosphodiesterase III (PDE III) inhibitors share a common mechanism in that both groups of agents do which of the following?



    • A.

      Decrease intracellular calcium in the vascular smooth muscle


    • B.

      Decrease the activity of protein kinase A


    • C.

      Increase the activity of adenylate cyclase


    • D.

      Increase the levels of cyclic adenosine monophosphate (cAMP) in the cardiac myocyte




Preferred response: D


Rationale


Catecholamines vary in terms of their effect on intracellular calcium in the vascular smooth muscle. PDE III inhibitors increase activity of adenylate cyclase. Both groups of drugs increase levels of the second messenger, cAMP.



  • 8.

    You are taking care of a 6-month-old boy in the immediate postoperative period of a ventricular septal defect repair. The infant is normotensive but has decreased perfusion. You wish to add an inotropic agent. Which statement best describes an inotropic agent?



    • A.

      Inotropic agents primarily increase blood pressure.


    • B.

      Inotropic agents primarily increase systemic vascular resistance.


    • C.

      Inotropic agents primarily increase stroke work.


    • D.

      Inotropic agents primarily decrease myocardial oxygen consumption.




Preferred response: C


Rationale


Inotropic agents increase stroke work at a given preload and afterload; they do not necessarily increase blood pressure. Vasopressor agents increase blood pressure by increasing systemic vascular resistance. Inotropic agents may increase or decrease myocardial oxygen consumption, depending on the clinical scenario.



  • 9.

    What is the result of an increase in cAMP in vascular smooth muscle?



    • A.

      Inhibition of uptake of calcium by sarcoplasmic reticulum with resultant vasoconstriction


    • B.

      Inhibition of uptake of calcium by sarcoplasmic reticulum with resultant vasodilation


    • C.

      Uptake of calcium by sarcoplasmic reticulum with resultant vasoconstriction


    • D.

      Uptake of calcium by sarcoplasmic reticulum with resultant vasodilation




Preferred response: D


Rationale


In vascular smooth muscle, both β 1 and β 2 receptors are present, although β 2 receptors predominate. The β 2 receptor is coupled to G s ; therefore activation of β 2 receptors promotes formation of cAMP. The resulting activation of cAMP-dependent protein kinase in vascular smooth muscle, however, stimulates pumps that remove calcium from the cytosol and also promotes calcium uptake by the sarcoplasmic reticulum. As cytosolic calcium concentration decreases, smooth muscle relaxes and the blood vessel dilates. Adrenergic receptors also have been demonstrated on the endothelium and are capable of producing relaxation of the vessel. The exact mechanism involved, including the role of nitric oxide and the subtype of β receptors involved, remain under investigation.



  • 10.

    What is the result of an increase in cAMP in myocyte?



    • A.

      Closing of voltage-gated calcium channels with a decrease in inotropy


    • B.

      Opening of voltage-gated calcium channels with an increase in inotropy


    • C.

      Uptake of calcium by the sarcoplasmic reticulum with a resultant decrease in the force of the next contraction


    • D.

      Uptake of calcium by the sarcoplasmic reticulum with a decrease in lusitropy




Preferred response: B


Rationale


Adrenergic receptors are typically coupled to one of three types of G proteins: G s , G i , or G q . G s proteins produce an increase in adenylate cyclase activity, whereas G i proteins promote a decrease in adenylate cyclase activity. G q protein receptors stimulate phospholipase C to generate diacylglycerol and inositol 1,4,5-triphosphate (IP 3 ).


Myocardial β 1 -adrenergic receptors are associated with G s . When an agonist agent engages this receptor type, the result is enhanced activity of adenylate cyclase and a rise in the concentration of cAMP, which activates protein kinase A (PKA). PKA in turn phosphorylates voltage-dependent calcium channels, increasing the fraction of channels that can be open and the probability that these channels are open, producing an increase in intracellular calcium concentration. Calcium then binds to troponin C, allowing for actin-myosin cross-bridge formation and sarcomere contraction. Also, PKA phosphorylates phospholamban, relieving the disinhibitory effect of the unphosphorylated form on calcium channels in the sarcoplasmic reticulum. The accumulation of calcium by the sarcoplasmic reticulum is thus enhanced, increasing the rate of sarcomere relaxation (lusitropy) and subsequently increasing the amount of calcium available for the next contraction. This leads to both enhanced contractility and active diastolic relaxation.



  • 11.

    Activation of V 1 receptors by arginine vasopressin (AVP) results in which of the following?



    • A.

      Stimulation of G q protein and activation of phospholipase C


    • B.

      Stimulation of G s protein and an increase in cAMP


    • C.

      Stimulation of G s protein and a decrease in cAMP


    • D.

      Stimulation of G i protein and an increase in cGMP




Preferred response: A


Rationale


Vasopressin receptors belong to the family of G protein–coupled receptors. V 1 receptors are coupled to G q , and V 2 receptors are coupled to G s . When vasopressin binds to the V 1 receptor, phospholipase C (PLC) is activated with the eventual production of inositol 1,4,5-triphosphate (IP 3 ) and 1,2 diacylglycerol (1,2 DG). These molecules increase the release of calcium from the endoplasmic reticulum and increase the entry of calcium through gated channels. The increase in intracellular calcium leads to an increase in the activity of myosin light chain kinase. This kinase acts on myosin to increase the number of actin-myosin cross-bridges, enhancing contraction of the myocyte. Of note, vasopressin has been shown to produce vasoconstriction in the skin, skeletal muscle, and fat while producing vasodilation in the renal, pulmonary, and cerebral vasculature. This effect may be mediated though nitric oxide or may be a function of the isoform of adenyl cyclase with which the receptor is coupled. AVP also has been shown to increase the pressor effects of catecholamines, although in two different vascular smooth muscle cell lines, AVP had opposing effects on isoproterenol-induced activation of adenyl cyclase.



  • 12.

    Which of the following drugs has the least effect on increasing myocardial oxygen consumption?



    • A.

      Dopamine


    • B.

      Dobutamine


    • C.

      Epinephrine


    • D.

      Milrinone




Preferred response: D


Rationale


With the exception of the bipyridines (amrinone, milrinone), all inotropes increase myocardial oxygen consumption because they increase myocardial work.



  • 13.

    Which of the following drugs worsens intrapulmonary shunt in acute respiratory distress syndrome?



    • A.

      Atropine


    • B.

      Digoxin


    • C.

      Dopamine


    • D.

      Milrinone




Preferred response: C


Rationale


Dopamine depresses the ventilatory response to hypoxemia and hypercarbia by as much as 60%. Dopamine (and other β agonists) decreases Pao 2 by interfering with hypoxic vasoconstriction. In one study, dopamine increased intrapulmonary shunting in patients with acute respiratory distress syndrome from 27% to 40%.



  • 14.

    Which of the following drugs may cause a decrease in heart rate?



    • A.

      Dopamine


    • B.

      Epinephrine


    • C.

      Isoproterenol


    • D.

      Norepinephrine




Preferred response: D


Rationale


Norepinephrine produces increases in systemic vascular resistance (SVR), arterial blood pressure, and urine flow. It is most valuable in the context of tachycardia because infusion of the drug does not produce significant elevation of heart rate and may even lower heart rate through reflex mechanisms. In a study of adults with abdominal sepsis, norepinephrine infusion was associated with increases in systemic blood pressure and SVR. Stroke volume increased as the heart rate declined. The cardiac index did not change, although creatinine clearance increased substantially. Norepinephrine has been shown to improve right ventricular performance in adults with hyperdynamic septic shock.



  • 15.

    Which of the following statements is true regarding the origin of these pharmacologic agents?



    • A.

      Arginine vasopressin is synthesized in the posterior pituitary.


    • B.

      Dobutamine is synthesized in the adrenal medulla from norepinephrine.


    • C.

      Dopamine is the immediate precursor of norepinephrine in the adrenal medulla.


    • D.

      Norepinephrine is synthesized in the adrenal medulla from epinephrine.




Preferred response: C


Rationale


Dopamine is a central neurotransmitter. It also is found in the sympathetic nerve terminals and in the adrenal medulla, where it is the immediate precursor of norepinephrine. Dopamine is hydroxylated at the β-carbon to produce norepinephrine, the principal neurotransmitter of the sympathetic nervous system. Arginine vasopressin is a nonapeptide hormone synthesized in the supraoptic and paraventricular nuclei of the hypothalamus. Dobutamine is a synthetic catecholamine. Isoproterenol is the synthetic N-isopropyl derivative of norepinephrine.



  • 16.

    An epinephrine infusion may result in which of the following adverse effects?



    • A.

      Hypercalcemia


    • B.

      Hyperkalemia


    • C.

      Hypocalcemia


    • D.

      Hypokalemia




Preferred response: D


Rationale


Hypokalemia is produced by an epinephrine infusion because of stimulation of β 2 -adrenergic receptors, which are linked to sodium-potassium-adenosine triphosphatase (ATPase) located in skeletal muscle. Infusion of 0.1 µg/kg/min lowered serum potassium by 0.8 mEq/L. Hyperglycemia results from β-adrenergic–mediated suppression of insulin release.



  • 17.

    Which of the following adverse effects is the most likely result of an isoproterenol infusion?



    • A.

      Bradycardia


    • B.

      Bronchoconstriction


    • C.

      Decrease in myocardial oxygen consumption


    • D.

      Decrease in serum theophylline concentrations




Preferred response: D


Rationale


Isoproterenol enhances cardiac contractility and cardiac rate. Peripheral vasodilation produces a decrease in systemic vascular resistance (SVR), augmenting the direct chronotropic action of the drug. Significant tachycardia ensues. Systolic blood pressure increases, while mean and diastolic pressures decrease. If normal prior to infusion of isoproterenol, mesenteric and renal perfusions decrease; however, if the subject was in shock, then the increase in cardiac output associated with isoproterenol administration may result in an increase in blood flow to these tissues. Isoproterenol increases myocardial demand for oxygen and decreases supply by reducing diastolic coronary filling. If intravascular fluid of the patient is depleted, hypotension may complicate initiation of isoproterenol infusion.


Pulmonary bronchial and vascular bed β 2 -adrenergic receptors produce bronchodilation and pulmonary vasodilation, respectively.


Adverse effects associated with isoproterenol include fear, anxiety, restlessness, insomnia, and blurred vision. Other effects may include headache, dizziness, tinnitus, sweating, flushing, pallor, tremor, nausea, vomiting, and asthenia. Cardiovascular effects may include ventricular tachycardia and other ventricular dysrhythmias that may be life threatening. Isoproterenol may cause hypertension and also can cause severe hypotension.


Isoproterenol decreases serum theophylline concentrations during concomitant therapy of status asthmaticus, and thus it may be necessary to increase theophylline dosage when isoproterenol therapy is initiated and reduce theophylline dosage when isoproterenol is discontinued.



  • 18.

    Which of the following is the major method of metabolism/elimination of milrinone?



    • A.

      Hepatic glucuronidation


    • B.

      Metabolism by catechol O -methyltransferase (COMT)


    • C.

      Metabolism by N -acetyltransferase


    • D.

      Renal excretion




Preferred response: D


Rationale


Milrinone is approximately 70% bound to plasma proteins, with approximately 85% renal elimination. Hepatic glucuronidation accounts for a minor elimination pathway. Both renal dysfunction and congestive heart failure affect the elimination profile of milrinone, extending the elimination half-life to approximately 2 hours. Amrinone is metabolized by N -acetyltransferase. Epinephrine is metabolized by COMT to metanephrine in the liver and kidneys or deaminated via the action of MAO. Dopamine and dobutamine are metabolized by COMT and MAO.



  • 19.

    What is the mechanism of action for digitalis?



    • A.

      Closure of voltage-gated calcium channels


    • B.

      Closure of K channels


    • C.

      Inhibition of sodium/potassium (Na/K)-ATPase


    • D.

      Opening of voltage-gated calcium channels




Preferred response: C


Rationale


Glycosides bind to and inhibit Na/K-ATPase. Binding of digoxin to ATPase is affected by serum potassium. Hyperkalemia depresses digoxin binding, whereas hypokalemia has the opposite effect, accounting in part for potentiation of digoxin-induced dysrhythmias during hypokalemia. Inhibition of ATPase produces an increase in intracellular calcium and enhances the inotropic state of the myocardium.



  • 20.

    Which of the following abnormalities potentiates digitalis toxicity?



    • A.

      Hypercalcemia


    • B.

      Hyperkalemia


    • C.

      Hypocalcemia


    • D.

      Hypokalemia




Preferred response: D


Rationale


Digitalis toxicity is made more likely by factors that increase myocardial irritability, such as myocarditis, ischemia, hypoxemia, or catecholamine support. Hypokalemia and alkalosis also potentiate digoxin-induced dysrhythmias. Treatment of digoxin toxicity involves supportive treatment and correction of electrolyte disturbances. Specific pharmacologic support (via atropine, lidocaine, phenytoin, or magnesium sulfate) may be necessary (although frequently unsuccessful), and in life-threatening circumstances, treatment with digoxin-specific Fab antibody fragments is indicated.


Chapter 32 : Cardiopulmonary interaction




  • 1.

    A child is receiving positive pressure ventilation for the acute respiratory distress syndrome. The child develops a large pleural effusion. For a given airway pressure, which of the following statements is true following the development of the effusion:



    • A.

      End expiratory lung volume is increased


    • B.

      Pleural pressure is increased


    • C.

      The transpulmonary pressure is increased


    • D.

      Tidal volume is unaffected




Preferred response: B


Rationale


For a given airway pressure, as chest wall elastance increases, pleural pressure rises and as a result the transpulmonary pressure decreases throughout the respiratory cycle, decreasing tidal volume and end expiratory lung volume.



  • 2.

    High airway pressure used for the management of hypoxic respiratory failure due to the acute respiratory distress syndrome may cause right ventricular stroke volume to decrease. Which of the following indicates the mechanism for decreased right heart output in this setting:



    • A.

      Enlarged right ventricle with a flattened interventricular septum throughout the cardiac cycle


    • B.

      Increasing arterial to end tidal CO 2 gradient


    • C.

      Lactic acidosis associated with hypotension


    • D.

      Systolic pressure variation or pulse pressure variation




Preferred response: A


Rationale


High airway pressure may adversely affect right ventricular (RV) stroke volume and output due to a decrease in RV preload, afterload, or a combination of both. Lactic acidosis and hypotension are consistent with a shock state but do not indicate the mechanisms responsible. An increase in the arterial to end tidal CO 2 gradient indicates an increase in wasted ventilation, which may be due to a decrease in pulmonary perfusion. However, if so, it does not indicate the mechanism. Systolic pressure variation is due to a decrease in RV stroke volume, which may result from altered RV loading conditions. An enlarged RV in association with elevated RV afterload (systolic flattening of the ventricular septum) would be consistent with positive pressure ventilation (PPV) increasing RV afterload and precipitating cor pulmonale. If the predominant effect of PPV was on pleural pressure and a decrease in systemic venous return, the RV would have decreased volumes throughout the cardiac cycle, and the ventricular septum would demonstrate normal position and orientation throughout the cardiac cycle.



  • 3.

    Which of the following does not affect the mean systemic pressure (Pms):



    • A.

      Venomotor tone


    • B.

      Systemic arterial blood pressure


    • C.

      Intravascular volume


    • D.

      Neurohormonal activity




Preferred response: B


Rationale


The determinants of the Pms are intravascular volume and vascular capacitance (compliance); an alteration in intravascular volume or venomotor tone (affecting vascular capacitance) will alter Pms. Similarly, altered neurohormonal activity will indirectly impact these factors as well. Blood pressure does not impact Pms; cardiac output is responsible for filling the systemic venous reservoirs. Blood pressure is not responsible for driving blood back to the heart.



  • 4.

    Why would positive pressure ventilation be expected to increase left ventricular stroke volume and cardiac output in patients with left ventricular systolic dysfunction?



    • A.

      In expiration, positive pressure ventilation propels forward coronary blood flow.


    • B.

      In patients with systolic ventricular dysfunction, the proportion of lung units with zone III conditions (pulmonary venous pressure > alveolar pressure) exceeds that of the normal circulation.


    • C.

      Positive airway pressure reduces left ventricular afterload.


    • D.

      Positive airway pressure raises transmural left atrial pressure.




Preferred response: C


Rationale


Positive pressure ventilation raises juxtacardiac pressure. This reduces the left ventricular wall tension needed to generate systolic pressure within the left ventricle sufficient to open the aortic valve. In effect, positive juxtacardiac pressure reduces afterload to the left heart. Positive pressure ventilation also reduces demand for cardiac output, but this does not, itself, increase cardiac output. Positive airway pressure would not be expected to propel forward coronary blood flow. There would be an increase in zone III lung units when pulmonary venous pressure is elevated, and this might blunt the rise in pulmonary vascular resistance attributable to alveolar capillary compression. Overall, a rise in juxtacardiac pressure would be expected to reduce transmural atrial pressure.



  • 5.

    Which of the following mechanisms mediates the positive pressure ventilation (PPV)-induced rise in right ventricle (RV) afterload?



    • A.

      A compensatory increase in the mean systemic pressure (Pms)


    • B.

      An increase in the proportion of lung units under zone III conditions (Pv > Ps)


    • C.

      Compression of alveolar vessels


    • D.

      Hypoxic pulmonary vasoconstriction




Preferred response: C


Rationale


Most of the alveolar surface is laced with a network of alveolar capillaries coursing within the alveolar septum. These sheets of epithelium and vessels lie between adjacent alveoli, so they are subject to the airway pressure that functionally surrounds them. Positive airway pressure compresses alveolar capillaries when alveolar pressure exceeds pulmonary capillary pressure, reducing the driving pressure that propels blood from pulmonary artery to pulmonary vein. There are also corner vessels that traverse the intersection of alveolar septae. These vessels are stretched and stented open at lung volumes above functional residual capacity, whether the lung is stretched by positive pressure or by spontaneous inspiration. This stenting of corner vessels reduces pulmonary vascular resistance. Positive airway pressure has little direct effect on mean systemic pressure but may, over a period of time, cause it to rise. A rise in Pms would increase venous return but would not increase right ventricular afterload. Hypoxic pulmonary vasoconstriction is a potent mechanism to alter the distribution of blood flow through the lungs. It does raise right ventricular afterload, but alveolar hypoxia should be reduced by positive pressure ventilation, not accentuated. As the lung is distended by positive airway pressure, the proportion of lung subject to zone III conditions is reduced, not increased. This reduction would raise right ventricular afterload.



  • 6.

    Pulsus paradoxus is an exaggeration of the normal version of which of the following?



    • A.

      Fall in systolic arterial pressure during spontaneous inspiration


    • B.

      Fall in right ventricular filling during spontaneous inspiration


    • C.

      Rise in aortic ejection velocity during spontaneous inspiration


    • D.

      Rise in right ventricular stroke volume during spontaneous inspiration




Preferred response: A


Rationale


Pulsus paradoxus is an exaggeration of the fall in systolic arterial pressure that normally occurs during spontaneous inspiration. It is a classical finding in pericardial tamponade but also occurs when pleural pressure drops precipitously during strained inspiration (e.g., in persons with croup, asthma, or pneumonia). In tamponade, end-diastolic cardiac volume is essentially fixed. Inspiration favors filling of the right side of the heart over filling of the left side of the heart, shifting the interventricular septum to the left and reducing left ventricular distention. This phenomenon decreases contractility by the Starling mechanism. Inspiration also increases left ventricular afterload by reducing juxtacardiac pressure. Restricting total cardiac volume accentuates these normal effects. Respiratory distress can accentuate these effects by the same mechanisms.


Pulsus paradoxus during tamponade was defined before the advent of modern invasive monitoring. It was the appearance and disappearance of the Korotkoff sound over the respiratory cycle that defined the phenomenon. As the pressure in the cuff fell, the first appearance of the Korotkoff sound signaled the expiratory arterial pressure. As the mercury continued to fall, the sound went from intermittent to regular and more frequent when the inspiratory systolic pressure was reached, and it could be heard with every beat of the heart. Although there should be an associated rise in right ventricular stroke volume, that finding is not the defining event in tamponade.


Chapter 33 : Disorders of cardiac rhythm




  • 1.

    A 4-month-old child presents to the intensive care unit following operative repair of tetralogy of Fallot in complete AV block with an atrial rate of 120 bpm and a wide complex ventricular escape rate of 50 bpm. Temporary epicardial pacing wires were placed on the right atrium and right ventricle during surgery, with acceptable sensing and pacing characteristics. What would be the best immediate management step?



    • A.

      Temporary AAI pacing


    • B.

      Temporary VOO pacing


    • C.

      Temporary DDD pacing


    • D.

      Placement of a transvenous dual chamber permanent pacemaker




Preferred response: C


Rationale


With acceptable atrial and ventricular sensing and pacing, dual chamber pacing is the preferred pacing mode. AAI pacing would be inappropriate in the setting of AV block. Temporary VOO pacing is associated with an increased risk of inducing ventricular tachycardia or fibrillation. Placement of a permanent pacemaker system would be inappropriate on postoperative day 1, given that AV nodal conduction may recover.



  • 2.

    What is the most common mechanism for supraventricular tachycardia (SVT) in infants and children?



    • A.

      AV node reentry tachycardia


    • B.

      Orthodromic reciprocating tachycardia (antegrade over accessory pathway, retrograde over atrioventricular [AV] node)


    • C.

      Orthodromic reciprocating tachycardia (antegrade over AV node, retrograde over accessory pathway)


    • D.

      Wolff-Parkinson-White syndrome




Preferred response: C


Rationale


Orthodromic reciprocating tachycardia utilizes an accessory pathway as the retrograde limb, but Wolff-Parkinson-White syndrome may or may not be present in sinus rhythm, making response D incorrect.



  • 3.

    Which statement is most true regarding adenosine?



    • A.

      Adenosine should never be administered in the presence of a wide QRS tachycardia.


    • B.

      Adenosine terminates most tachycardias by blocking conduction in accessory pathways.


    • C.

      Adenosine specifically blocks conduction in the AV node with little effect on blood pressure or sinus rate.


    • D.

      Prompt access to defibrillation should be readily available when administering adenosine.




Preferred response: D


Rationale


Adenosine can cause profound hemodynamic compromise or even ventricular fibrillation, requiring access to immediate defibrillation, especially if it is being administered to patients with wide QRS tachycardia or known Wolff-Parkinson-White syndrome.



  • 4.

    What is the most appropriate acute therapy for incessant supraventricular tachycardias (SVTs) such as persistent junctional reciprocating tachycardia in an infant?



    • A.

      Intravenous verapamil


    • B.

      Procainamide infusion


    • C.

      Repeated doses of adenosine until sinus rhythm is sustained


    • D.

      Vagal maneuvers such as ice to face




Preferred response: B


Rationale


The challenge in persons with incessant SVTs is the tendency for the SVTs to reinitiate. Measures directed at termination are of lesser utility. Repeated adenosine doses or repeated DC cardioversion may aggravate incessant behavior, and intravenous calcium channel blockers should not be administered to infants.



  • 5.

    Regarding ventricular tachycardias (VTs), which of the following represents the most correct statement?



    • A.

      Calcium channel blockers should never be administered for VT.


    • B.

      Intravenously administered amiodarone is superior to intravenously administered lidocaine for treatment of sustained VT.


    • C.

      The most appropriate acute therapy for polymorphic VT is intravenous magnesium.


    • D.

      Unlike in adults, VTs in children are rarely life threatening.




Preferred response: B


Rationale


Certain idiopathic forms of VT are seen in healthy young patients; these forms of VT may respond to calcium channel blockers acutely and are amenable to catheter ablation. However, all ventricular tachycardias should be approached initially as potentially life threatening, and for polymorphic VT (including torsades de pointes) or pulseless monomorphic VT, immediate cardioversion should precede intravenous drug therapy.


Chapter 34 : Shock states




  • 1.

    You are called to the emergency department to see a 17-year-old male who presents as the driver in a high speed motor vehicle crash. He was restrained, vehicle intrusion was greater than 18 inches, and he was unresponsive at the scene. He was orotracheally intubated by EMS and presents with the following vital signs: heart rate, 165 beats/min; respiratory rate, 20/min; blood pressure, 75/35 mm Hg. He has sternal bruising, and his abdomen is distended and firm. In preparation for presentation to the operating room, your priorities are to:



    • A.

      Initiate your institution’s massive transfusion policy with the goal of administering colloid fluids as soon as possible without delay in presentation to the operating room.


    • B.

      Obtain an arterial blood gas and then adjust mechanical ventilator settings


    • C.

      Obtain arterial and central venous access and then obtain a complete blood cell count, coagulation function tests and transaminases before clearing the patient to go to the operating room.


    • D.

      Treat hypotension by administering 20 mL/kg NS intravenously




Preferred response: A


Rationale


This patient presents with a mechanism and clinical evidence suggestive of blunt thoracic and abdominal trauma. Initiating massive transfusion protocols for patients presenting in shock with suspected hemorrhage and reducing barriers to presentation to the operating room for damage control procedures are important. Room temperature normal saline can exacerbate the “triad of death” in trauma patients by worsening hypothermia, diluting coagulation, and contributing to hyperchloremic acidosis.



  • 2.

    You are called to the emergency room to help evaluate two 3-year-old patients in separate rooms. The child in room A presents with a 3-day history of vomiting, diarrhea, and reduced eating and drinking. You find the child in room A to be awake, alert, but fussy; the child has dry mucous membranes; heart rate, 146 beats per minute; blood pressure, 73/52 mm Hg; warm extremities; and capillary refill of 3 seconds. The child in room B presents with a similar history of 3 days of vomiting, diarrhea, and reduced intake. The child in room B is sleepy but able to be aroused. She becomes agitated when aroused and has dry mucous membranes; heart rate, 144 beats per minute; blood pressure, 74/46 mm Hg; cool extremities, and capillary refill of 3 seconds. Your assessment is that one of these children is hypovolemic, and the other is in compensated hypovolemic shock. Therefore what is your recommendation?



    • A.

      Both child A and B should be treated with oral rehydration over the next 24 to 48 hours per WHO guidelines.


    • B.

      Both child A and B should be treated with 20 mL/kg of crystalloid delivered intravenously in ≤15 min.


    • C.

      Child A should be treated with 20 mL/kg of crystalloid delivered intravenously in ≤15 min, and child B should be treated with oral rehydration over the next 24 to 48 hours per WHO guidelines.


    • D.

      Child A should be treated with oral rehydration over the next 24 to 48 hours per WHO guidelines, and child B should be treated with 20 mL/kg of crystalloid delivered intravenously in ≤15 min.




Preferred response: D


Rationale


Child A has hypovolemia due to gastroenteritis. Child A has normal compensatory responses without evidence of end-organ dysfunction. Child B has an abnormal neurologic exam and a widening pulse pressure. Child B is in compensated hypovolemic shock. Compensated shock should be initially treated with rapid volume administration, whereas patients with hypovolemia can be safely treated with oral rehydration per WHO recommendations.



  • 3.

    A 5-year-old child with acute lymphoblastic leukemia and neutropenia presents to the pediatric intensive care unit with evidence of septic shock. Data suggest that children who receive <40 mL/kg of intravenous fluids and are still in shock at the end of the first hour have worse outcomes than those who receive >40 mL/kg and are still in shock at the end of the first hour. Therefore what is your goal?



    • A.

      Obtain a Scvo 2 from the indwelling central catheter, place an arterial line and obtain a serum lactate, and then titrate fluid resuscitation until you see the Scvo 2 and lactate improve to normal ranges.


    • B.

      Treat with 20 mL/kg of crystalloid in ≤15 min, and then reevaluate for evidence of fluid response.


    • C.

      Treat with 60 mL/kg of crystalloid fluid resuscitation within the first hour.


    • D.

      Treat with 60 mL/kg of crystalloid and colloid as indicated within the first hour.




Preferred response: B


Rationale


Although data suggest that early, rapid fluid resuscitation improves mortality, there are accumulating data that suggest that fluid overload worsens mortality. Therefore the goal in fluid resuscitation is to give just what the patient requires. If the patient shows evidence of sustained response to fluid administration and resolution of shock, no further fluid boluses should be administered. If the patient shows refractory shock, fluid resuscitation should be continued for up to a total of 60 mL/kg in the first hour. After the first hour, if there is evidence of sustained shock, careful evaluation should be used to determine the indication for vasoactive/inotropic support with or without further fluid resuscitation.



  • 4.

    You are working in the CICU and you receive a phone call from an outside provider in an emergency department caring for a 17-day-old newborn who weighs 3.2 kg and presented in shock. The patient is a full-term infant born to a G2P2 female who did not receive prenatal care. The patient was born at home and delivered by a midwife. Delivery was uncomplicated. The mother reports poor feeding for the last 24 hours, and this morning the newborn was found to be less responsive. In the ED, the patient presents with the following vital signs: temperature, 36.7°C; heart rate, 185 beats per minute; blood pressure, 52/28 mm Hg; respiratory rate, 74 per minute. The patient appears gray and is unresponsive. What do you recommend?



    • A.

      Ceftriaxone, 20 mL/kg intravenous bolus of crystalloid, and transfer to the PICU


    • B.

      Orotracheal intubation, ceftriaxone, 20 mL/kg intravenous bolus of crystalloid, and transfer to the PICU


    • C.

      Orotracheal intubation, ampicillin and gentamicin, 10 mL/kg intravenous bolus of crystalloid, continuous infusion of prostaglandin E 1 , and transfer to the PICU


    • D.

      Orotracheal intubation, ampicillin and gentamicin, 10 mL/kg intravenous bolus of crystalloid, echocardiogram, and transfer to the PICU




Preferred response: C


Rationale


Newborns (<28 days) presenting in shock should be suspected for sepsis and a ductal-dependent congenital heart disease. Therefore fluid resuscitation should be provided, but it should start with a lower volume and then be reevaluated for responsiveness. Ceftriaxone may be okay for newborns, but risks in the newborn period make ampicillin and gentamicin a better empiric choice. Empirically starting a continuous infusion of prostaglandin E 1 (PGE 1 ) at 0.05 to 0.1 μg/kg/min may be lifesaving and should be started prior to obtaining the echocardiogram. PGE 1 may cause apnea, but in an unresponsive infant presenting in shock, early intubation is indicated.



  • 5.

    You are caring for a 5-year-old female with acute viral myocarditis. She is sedated, mechanically ventilated, and treated with a continuous infusion of epinephrine at 1 μg/kg/min. Her vital signs are as follows: temperature, 36.8°C; heart rate, 140 beats per min; blood pressure, 75/45 mm Hg; central venous pressure (CVP), 12 cm H 2 O. Her serum lactate level is rising. Echocardiogram shows that her right atrium is adequately filled, systolic ventricular function is depressed bilaterally with a left ventricular shortening fraction of 18% by M-mode. The septum is midline.



The next best step in the management of this patient is to:



  • A.

    Call your extracorporeal life support team because she is failing medical management of heart failure


  • B.

    Deepen her sedation and start to cool her to reduce metabolic demand


  • C.

    Treat with a pulmonary vasodilator because you are concerned that her CVP suggests right ventricular failure


  • D.

    Treat with a vasodilator because you are concerned that her cardiac output is reduced to high systemic vascular resistance.



Preferred response: A


Rationale


Vasodilators may be indicated in a hypertensive patient with evidence of shock and low systolic function. However, this patient is hypotensive, suggesting that peripheral vasodilation may not be indicated. Although the patient’s CVP is greater than 10 cm H 2 O and the echo suggests low right ventricular systolic function, the septum remains midline, which suggests that reduction of pulmonary vascular resistance will not substantially increase cardiac output. Although reduction of metabolic demand is a reasonable next step, deepening sedation carries the risk of worsening myocardial function. Consideration of extracorporeal support is a reasonable next step in this patient with evidence of adequate right ventricular preload, low systolic function, and refractory shock despite high inotropic therapy.


Chapter 35 : Pediatric cardiopulmonary bypass




  • 1.

    When blood comes into contact with the cardiopulmonary bypass circuitry, these foreign surfaces will not do which of the following?



    • A.

      Activate inflammatory response


    • B.

      Cause hemolysis


    • C.

      Disrupt hemostasis


    • D.

      Shift the oxyhemoglobin curve leftward




Preferred response: D


Rationale


It is well documented that nonendothelial blood contact activates the systemic inflammatory response and causes hemolysis and coagulopathy. Oxyhemoglobin shifts are caused by changes in blood pH, temperature, pco 2 , and 2,3-DPG.



  • 2.

    Which of the following is an effective strategy to protect the immature myocardium during ischemic arrest?



    • A.

      Increase the cardiac membrane resting potential.


    • B.

      Keep the myocardium warm.


    • C.

      Perfuse the coronaries with a hypotonic solution.


    • D.

      Prevent intracellular calcium accumulation.




Preferred response: D


Rationale


Calcium shifting at the cellular membrane is an energy-dependent process, and minimizing intracellular calcium accumulation will help to prevent ATP depletion. Decreasing the cardiac membrane resting potential to achieve diastolic arrest is the goal of depolarizing cardioplegia. Delivering a cold cardioplegia solution will minimize metabolic demands and prolong the myocardial tolerance to ischemia. An isotonic or hypertonic cardioplegia solution is desired to reduce intracellular water accumulation and edema.



  • 3.

    During the aortic cross clamp, which of the following perfusion techniques would be the most effective in reducing collateral blood flow to the heart and improving operative visibility?



    • A.

      Administer more heparin and raise the activated coagulation time (ACT)


    • B.

      Decrease arterial flow and patient temperature


    • C.

      Increase the mean arterial pressure and cardiac index


    • D.

      Transfuse an isotonic crystalloid solution and reduce viscosity




Preferred response: B


Rationale


Patients with pulmonary blood flow restrictions (e.g., tetralogy of Fallot, pulmonary atresia) can develop major aortopulmonary collateral arteries, and these collaterals can flood the heart during the aortic cross-clamp period if cardiopulmonary bypass flow is maintained. This excessive blood return not only obscures the surgical site but may also warm the cold arrested myocardium or wash out cardioplegia from the coronary arteries. Hypothermia and perfusion flow rate reduction can attenuate this collateral flow while maintaining adequate oxygenation delivery to the patient.



  • 4.

    Which of the following factors does not increase the incidence of acute kidney injury (AKI) during cardiopulmonary bypass?



    • A.

      Deep hypothermic circulatory arrest (DHCA)


    • B.

      Hypotension


    • C.

      Pulsatile perfusion flow


    • D.

      Younger age




Preferred response: C


Rationale


The kidneys perceive nonpulsatile flow or a decrease in arterial flow as hypovolemia, and the resultant neurohormonal cascade is thought to trigger the AKI complex. Since most perioperative risk factors such as younger age and the incidence of higher surgical complexity are nonmodifiable, therapeutic strategies have focused on optimally managing perfusion flow rate, arterial pressure, and hematocrit.


Chapter 36 : Critical care after surgery for congenital cardiac disease




  • 1.

    A 10-month-old girl (8 kg) is recovering from surgery to repair a ventricular septal defect. She has been taking her usual diet of breast milk and age-appropriate food since postoperative day 2. On postoperative day 3, while extubated and still requiring oxygen via nasal cannula, a chest radiograph showed a moderately sized right pleural effusion. A pigtail catheter was placed in the right pleural space and yielded 150 ml of white effluent containing 2000 WBCs (95% lymphocytes) and elevated triglycerides (410 mg/dL). The pleural catheter drains another 100 ml over the following day and follow up chest radiograph obtained on postoperative day 4 shows a small residual effusion.




    • The MOST appropriate next step in the management of this patient is to:


    • A.

      Begin a diet with medium-chain triglycerides (MCT) as the source of fat.


    • B.

      Begin an intravenous infusion of octreotide.


    • C.

      Continue current management and follow chest tube output.


    • D.

      Discontinue enteral nutrition and begin total parenteral nutrition.


    • E.

      Perform a lymphangiogram in preparation for lymphatic embolization or thoracic duct ligation.




Preferred response: A


Rationale


This patient developed chylothorax following cardiac surgery. With drainage of 12.5 mL/kg over 24 hours, this chylothorax would be classified in the “low volume” category. The initial management of low volume chylothoraces is centered on a diet low in long-chain triglycerides, where the source of fat is in the form of medium-chain triglycerides (MCT). This is often sufficient to resolve the chylothorax within 7 days. If persistent, additional treatments may be started sequentially; these include enteral fasting with total parenteral nutrition, octreotide, and thoracic duct embolization or surgical ligation.



  • 2.

    A 5-day-old child born with d-transposition of the great arteries is recovering in the cardiac ICU following an arterial switch operation. Approximately twelve hours after successful separation from cardiopulmonary bypass, the patient is noted to have frequent premature ventricular contractions (PVCs). She is receiving mechanical ventilation and an infusion of epinephrine (0.05 μg/kg/min). The vital signs are heart rate, 170 beats per minute; respiratory rate, 28 per minute; blood pressure. 55/30 mm Hg (mean 41 mm Hg), central venous pressure, 14 mm Hg; left atrial pressure, 17 mm Hg. Electrolytes are normal, but an arterial blood gas shows a mild metabolic acidosis (7.34/38/123/-3/99%) and a lactate of 3.2 mmol/L.




    • The MOST appropriate next step in the management of this patient is to:


    • A.

      Administer a 10 mL/kg intravenous bolus of 5% albumin


    • B.

      Obtain a STAT chest radiograph


    • C.

      Obtain a STAT echocardiogram


    • D.

      Start a furosemide infusion at 0.1 mg/kg/h


    • E.

      Start a vasopressin infusion at 0.3 mU/kg/min




Preferred response: C


Rationale


This patient is tachycardic, hypotensive, has a disproportionate elevation of the left atrial pressure, an elevated blood lactate, and new onset of frequent ventricular ectopy. Although it would be tempting to ascribe these findings to the vagaries of a post-cardiopulmonary bypass low cardiac output state, they are much more likely to be the manifestation of poor coronary blood flow. Therefore, it is imperative that a surgical issue, such as a coronary reimplantation kink or tamponade be ruled out. A STAT echocardiogram should be obtained to evaluate function, rule out tamponade, and ascertain adequate coronary blood flow. If the coronary buttons cannot be well visualized by echocardiogram, emergent cardiac catheterization should be performed. A fluid bolus would not be indicated in this patent with an already elevated left atrial pressure. Although a furosemide infusion is often initiated in the postoperative period, it would not address the primary concern of myocardial ischemia. Vasopressin would cause an increase in systemic vascular resistance, but it would not be a fruitful therapy if the patient indeed has a kink of the left coronary button. A chest radiograph would have been useful to rule out a new pulmonary process or pleural effusions, but would be low yield in this situation.



  • 3.

    A 6-day-old child is returned to the cardiac intensive care unit after a stage I palliation (Norwood procedure) for hypoplastic left heart syndrome. He remains endotracheally intubated and has an open sternum. Heart rate is 180 beats/min and blood pressure is 55/28 mm Hg while on epinephrine (0.05 µg/kg/min) and milrinone (0.5 µg/kg/min). He is being ventilated with a tidal volume of 8 mL/kg, PEEP of 5 cm H 2 O, rate of 26 breaths/min, and F io 2 of 0.4. An arterial blood gas analysis shows severe metabolic acidosis (pH 7.16, Paco 2 40 mm Hg, Pao 2 78 mm Hg, Sao 2 94%, base deficit, 13). What is the next best step to manage this condition?



    • A.

      Decrease the F io 2 to 0.21.


    • B.

      Decrease the milrinone dose to 0.3 µg/kg/min.


    • C.

      Increase the epinephrine dose to 0.08 µg/kg/min.


    • D.

      Increase the respiratory rate to 30 breaths/min.




Preferred response: A


Rationale


The patient has classic signs of an unbalanced circulation following the Norwood procedure, with excess pulmonary blood flow and decreased systemic perfusion (high Qp:Qs). This is a serious emergency and requires prompt action. Decreasing the F io 2 to room air can be accomplished rapidly and will help balance the Qp:Qs. In some cases, carbon dioxide may need to be added to the inspired gas or subatmospheric F io 2 to help reduce excessive pulmonary blood flow.


Decreasing the dose of milrinone would not be a reasonable strategy for this patient, who can in fact benefit from additional afterload reduction to increase systemic blood flow and help balance the Qp:Qs. For the same reason, increasing the dose of epinephrine could be detrimental as it will likely increase the systemic vascular resistance and increase pulmonary blood flow. Increasing the respiratory rate to 30 breaths/min would lower the Paco 2 , and although this will help compensate the acidosis, it will also lower pulmonary vascular resistance and adversely contribute to the already increased pulmonary blood flow.



  • 4.

    A 6-month-old child with history of tricuspid atresia returns to the cardiac intensive care unit intubated following a bidirectional Glenn anastomosis. She is hemodynamically stable and in sinus rhythm but has significant hypoxemia (Sao 2 of 65%) despite being offered a F io 2 of 1.0 through the ventilator. The hypoxemia persists despite adequate sedation and intravascular volume expansion with packed red blood cells to increase the hematocrit to 40%. Of the following, which intervention is least likely to meaningfully improve the arterial oxygen saturation in this patient?



    • A.

      Controlled hypoventilation (permissive hypercapnia)


    • B.

      Head elevation (30 degrees)


    • C.

      Inhaled nitric oxide


    • D.

      Intravenous milrinone and epinephrine




Preferred response: C


Rationale


Following the bidirectional Glenn anastomosis, arterial oxygen saturation should be in the 80% to 85% range; however, stabilization to this level can take a number of days. Persistent hypoxemia (Sao 2 <70%) can be secondary to a low cardiac output state (low Svo 2 ), low pulmonary blood flow, or lung disease. Treatment is directed at improving contractility, increasing superior vena cava venous return, reducing afterload, and ensuring the patient has a normal rhythm and hematocrit. Increased pulmonary vascular resistance is an uncommon cause, and inhaled nitric oxide (NO) is not generally beneficial in these patients. This finding is not surprising because pulmonary artery (PA) pressure and resistance and vascular tone are not high enough following this surgery to see a demonstrable benefit from NO. Controlled hypoventilation targeting hypercapnia promotes increased cerebral blood flow and, consequently, increased blood flow through the superior vena cava (SVC)-PA anastomosis. Persistent profound hypoxemia should be investigated in the catheterization laboratory to evaluate hemodynamics, look for residual anatomic defects limiting pulmonary flow, such as SVC or PA stenosis or a restrictive atrial septal defect (ASD), and coil any significant venous decompressing collaterals (e.g., azygous vein), if present.



  • 5.

    Following a Fontan operation for palliation of the hypoplastic left heart syndrome, which of the following postoperative strategies is commonly used?



    • A.

      Early extubation whenever the patient is able to assume spontaneous breathing


    • B.

      Forced diuresis to lower the central venous pressure


    • C.

      High doses of dopamine and epinephrine to increase systemic vascular resistance and blood pressure


    • D.

      Mechanical ventilation with high levels of positive end-expiratory pressure to optimize lung inflation and increase pulmonary blood flow




Preferred response: A


Rationale


Following a Fontan operation, liberation from positive pressure ventilation should be accomplished as soon as the patient is able to assume spontaneous breathing, provided there is no significant lung disease or atelectasis. For this reason, prolonged sedation and paralysis generally are not indicated. High doses of vasoactive drugs should be avoided because they increase the afterload to the systemic right ventricle. Lowering the central venous pressure is not indicated because pulmonary blood flow is largely dependent on systemic venous return. A high level of positive end-expiratory pressure should be avoided in these patients because it increases intrathoracic pressure, reduces venous return, and consequently reduces pulmonary blood flow.



  • 6.

    Which of the following is true for a patient with tetralogy of Fallot?



    • A.

      A residual ventricular septal defect (VSD) promotes left-to-right shunt and augments pulmonary blood flow.


    • B.

      Epinephrine is the drug of choice in the postoperative period because it increases cardiac output and contractility of the poorly compliant right ventricle.


    • C.

      Patients usually require high right atrial pressure postoperatively because of the hypertrophic and poorly compliant right ventricle.


    • D.

      The presence of an atrial level communication is undesirable because it can lead to hypoxemia and decreased preload to the left ventricle.




Preferred response: C


Rationale


The poorly compliant right ventricle requires high right-sided filling pressures (right atrial pressure). Use of vasodilators should be avoided during a hypercyanotic spell, because lowering systemic vascular resistance will increase the right-to-left shunt and worsen hypoxemia and acidosis. An atrial level communication is highly desirable in patients with significant right ventricular diastolic dysfunction, because it allows for a right-to-left atrial level shunt that ensures adequate left ventricular preload (although it causes arterial oxygen desaturation). Milrinone is the drug of choice in the postoperative period because of its lusitropic effects. A residual ventricular septal defect (VSD) is deleterious, particularly in the setting of a persistent right ventricular outflow tract obstruction.


Chapter 37 : Cardiac transplantation




  • 1.

    A 5-year-old patient with dilated cardiomyopathy presents to the cardiac ICU with severe decompensated congestive heart failure. Which one of the following therapeutic options would be the most successful bridge to transplant?



    • A.

      Enalapril


    • B.

      Extracorporeal membrane oxygenation (ECMO)


    • C.

      Furosemide


    • D.

      Mechanical ventilation


    • E.

      Ventricular assist device




Preferred response: E


Rationale


While all options could be used to treat congestive heart failure, use of a ventricular assist device has been shown to improve waitlist mortality and improve posttransplant outcomes. Mechanical ventilator support and ECMO are risk factors for increased waitlist and posttransplant mortality.



  • 2.

    Which of the following would be the strongest contraindication to isolated heart transplantation in a neonate?



    • A.

      Common pulmonary vein atresia


    • B.

      Dextrocardia


    • C.

      Discontinuous pulmonary arteries


    • D.

      Double aortic arch


    • E.

      Total anomalous pulmonary venous return




Preferred response: A


Rationale


Isolated heart transplantation in the presence of pulmonary vein atresia would be technically difficult to perform, especially in a neonate. All other conditions presented could be overcome with good surgical planning.



  • 3.

    In a 2-month-old infant with hypoplastic left heart syndrome who has undergone a Stage I reconstruction, the strongest indication for heart transplantation would be which of the following:



    • A.

      Aortic arch obstruction with right ventricular dysfunction


    • B.

      Marked cyanosis


    • C.

      Protein losing enteropathy


    • D.

      Restrictive atrial septal defect


    • E.

      Severe tricuspid valve regurgitation




Preferred response: E


Rationale


Significant tricuspid valve regurgitation is a risk factor for failure of single ventricle palliation with poor outcomes. Aortic arch obstruction and restrictive atrial septal defect require intervention. Should that be unsuccessful, then transplantation could be considered. Protein losing enteropathy is generally not a complication of a stage I reconstruction, and cyanosis alone is not an indication for transplantation.



  • 4.

    You admit a 12-year-old from the operating room following heart transplantation. The nurse is concerned about an episode of hypotension and asks which value on the monitor most likely indicates the presence of ventricular diastolic dysfunction. You answer that it is which of the following:



    • A.

      Cardiac index 3.3 L/min/m 2


    • B.

      CVP 22 mm Hg


    • C.

      Heart rate 115 bpm


    • D.

      NIRS 65%


    • E.

      PA pressure 32/15 mm Hg




Preferred response: B


Rationale


Diastolic dysfunction of the right ventricle is common after heart transplantation. Central venous pressure (CVP) represents the end-diastolic pressure of the right ventricle in the setting of normal tricuspid valve function. Therefore, elevation of the CVP postcardiac surgery indicates the presence of ventricular diastolic dysfunction. The other parameters listed have values that are normal or near normal following heart transplantation.



  • 5.

    The major cause of death in adolescents after heart transplant is which of the following?



    • A.

      Dysrhythmia


    • B.

      Noncompliance


    • C.

      Renal failure


    • D.

      Thromboembolism




Preferred response: B


Rationale


Currently the major cause of death in the adolescent heart transplant recipient is noncompliance with the medical regimen.



  • 6.

    The preferred anticoagulant for patients in the intensive care unit who have myocardial dysfunction and are awaiting a heart transplant is which of the following?



    • A.

      Aspirin


    • B.

      Coumadin


    • C.

      Heparin


    • D.

      Lovenox




Preferred response: C


Rationale


All patients waiting for heart transplantation should be managed with systemic anticoagulation. Heparin is preferred, but warfarin is acceptable in a stable patient who is receiving inotropic support. We add a word of caution regarding the use of low-molecular-weight heparin for prophylaxis: Enoxaparin cannot be easily reversed in a patient who must go to the operating room emergently because a donor heart has been identified. Cardiovascular surgeons prefer using heparin for prophylactic anticoagulation.



  • 7.

    First-line medication for management of hypotension in the potential heart donor is which of the following?



    • A.

      Dobutamine


    • B.

      Dopamine


    • C.

      Epinephrine


    • D.

      Vasopressin




Preferred response: D


Rationale


A catecholamine surge causing unnatural circulatory physiology that rapidly evolves is associated with brain death, making management of the donor difficult. This intense sympathomimetic outflow initially causes vasoconstriction resulting in tachycardia, hypertension, and increased myocardial oxygen demand. The result can be a direct injury to the myocardium in the potentially transplantable heart. Myocardial structural damage that includes myocytolysis, contraction band necrosis, subendocardial hemorrhage, edema formation, and interstitial mononuclear infiltration is seen. This initial sympathetic outflow is followed by a loss of sympathetic tone, resulting in marked vasodilation and hypotension. The hypotension and cardiovascular collapse are related to decreased systemic vascular resistance rather than primary myocardial dysfunction. Vasopressin is now the first-line blood pressure support medication because it treats diabetes insipidus in addition to supporting blood pressure. Vasopressin infusion of less than 2.5 units/hour usually is sufficient to increase mean arterial blood pressure and not cause end-organ injury.



  • 8.

    Cardiac output in the heart immediately after transplantation is impaired primarily by which of the following?



    • A.

      Arch obstruction


    • B.

      Diastolic dysfunction


    • C.

      Mitral valve insufficiency


    • D.

      Tricuspid valve insufficiency




Preferred response: B


Rationale


The major changes in the physiology of the transplanted heart are related to autonomic denervation, including diastolic dysfunction and an exaggerated response to exogenously administered catecholamines. The transplanted heart also must adapt to a new environment related to the recipient’s lung function and elevated pulmonary vascular resistance (PVR). Hemodynamics of the transplanted heart reflect a significant shift to the left of the pressure/volume curve. Diastolic dysfunction can be demonstrated from the early transplant period. Diastolic dysfunction is a significant impairment to early allograft function, limiting cardiac output. Diastolic dysfunction emphasizes the importance of heart rate and early sinus node function. The capability of temporary pacing in the early perioperative period is mandatory.



  • 9.

    Increased perioperative mortality for heart transplantation is associated with which of the following?



    • A.

      Mitral valve insufficiency


    • B.

      Tricuspid valve insufficiency


    • C.

      Pulmonary arterial pressure <15 mm Hg


    • D.

      Pulmonary vascular resistance (PVR) >6 Wood units




Preferred response: D


Rationale


High PVR in the recipient increases perioperative morbidity and mortality and can affect late survival. All potential heart recipients undergo cardiac catheterization prior to heart transplantation to document the anatomy of systemic and pulmonary venous connections, determine pulmonary artery size and distribution, and calculate PVR. The upper limit of PVR associated with successful orthotopic heart transplantation is not known. Criteria developed from the adult heart transplant experience indicate that a PVR greater than 6 Wood units or a transpulmonary gradient (pulmonary artery mean pressure — left atrial mean pressure) greater than 15 mm Hg is associated with increased perioperative mortality. The transpulmonary gradient is the most useful number for estimating PVR because measurement of cardiac output in the catheterization laboratory can be flawed. In children the PVR index, which is determined by dividing transpulmonary gradient by cardiac index, is more useful because children come in all sizes. A PVR index less than 6 index units is associated with low perioperative mortality.



  • 10.

    Hyperglycemia following heart transplantation is most likely related to which of the following?



    • A.

      Antithymocyte globulin


    • B.

      Basiliximab


    • C.

      Furosemide


    • D.

      Tacrolimus




Preferred response: D


Rationale


Hyperglycemia is common after heart transplantation with tacrolimus-based immune suppression. The combination of decreased insulin production from islet cells caused by tacrolimus and decreased peripheral utilization related to high-dose corticosteroids results in nonketotic hyperglycemia. Insulin is initially mandatory in management but often can be discontinued if the tacrolimus dose is reduced and the corticosteroid portion of maintenance immune suppression is discontinued.



  • 11.

    Diastolic dysfunction in a patient after heart transplantation most commonly will manifest as which of the following changes following a normal saline solution fluid challenge?



    • A.

      An increase in right atrial pressure by 2 mm Hg


    • B.

      An increase in left atrial pressure by 2 mm Hg


    • C.

      No change in right atrial pressure


    • D.

      No change in stroke volume




Preferred response: B


Rationale


Hypotension and cardiovascular collapse in patients after heart transplantation are related to decrease in systemic vascular resistance rather than primary myocardial dysfunction. Large fluid volumes and high-dose inotropic agents at α-adrenergic dosing range are administered, causing volume overload and vasoconstriction that can injure all donor organs. Hearts that are supported on high-dose inotropic agents likely will exhibit myocardial injury. A risk factor that predicts donor heart failure is a history of high-dose dopamine, dobutamine greater than 20 mg/kg/min, and epinephrine greater than 0.1 mg/kg/min.



  • 12.

    A 5-year old child with dilated cardiomyopathy in your pediatric intensive care unit is waiting for an orthotopic heart transplant. The child’s weight is 20 kg. She is receiving intermittent noninvasive continuous positive airway pressure. Her mixed venous oxygen saturation is 45%, and her left ventricular ejection fraction (LVEF) is less than 20%. Inotropic support includes a milrinone drip of 0.75 mg/kg/min and an epinephrine drip of 0.02 mg/kg/min. A donor organ becomes available within 1 hour of flying time from your medical center. The donor is a teenager who became brain dead after a motor vehicle accident 3 days ago. The donor weight is 40 kg. Resuscitation at the accident site was required with unknown down time. Vasopressin and epinephrine were initiated and given for the first 24 hours but subsequently weaned to low-dose dopamine. Current evaluation of the donor heart shows an LVEF of 60% with trivial mitral valve regurgitation. Would you recover this donor heart for your patient?



    • A.

      No; the uncertain down time and high-dose inotropic support make the donor unacceptable.


    • B.

      No; the donor is too large for my patient.


    • C.

      No; the distance from the center makes it impossible for ischemic time to be less than 4 hours.


    • D.

      Possibly; the donor is potentially acceptable. I would review all of the information from the donor site.




Preferred response: D


Rationale


Donor heart availability is limited, and the number of patients who die while waiting for a heart transplant is significant. Your recipient is critically ill, and the need for circulatory support is imminent (via extracorporeal membrane oxygenation or a ventricular support device), so all potential donor hearts should be critically evaluated. An estimate of donor “down time” is important but often inaccurate. Inotropic resuscitation of the donor is common. The use of high-dose inotropic support (i.e., epinephrine and norepinephrine) for a prolonged period of time (>24 hours) reflects significant donor heart ischemia, and in such cases the donor heart should not be used for transplantation. A donor/recipient weight of greater than 2:1 is common. We use the donor/recipient aortic root size rather than weight to estimate the size mismatch. We rarely use an undersized donor (i.e., weight less than 20% of the recipient) for transplantation. Flight time from the transplant center is relative. Ideally a total ischemic time of less than 4 hours is ideal (i.e., the time from aortic cross-clamp at the donor site to release of the aortic cross-clamp on the recipient). It is important to remember that a perfect heart to transplant is never available. We encourage direct verbal communication with the donor referral center.



  • 13.

    A 15-year old patient with tricuspid atresia has had palliative surgery (atrial to pulmonary connection) (Fontan). A myopathic ventricular dysfunction and protein-losing enteropathy have developed. He has just had an orthotopic transplant. You are discussing potential immune suppression regimens. Pretransplant panel reactive antibody is 10% for class 1 human leukocyte antigens. Which of the following is of major concern for renal dysfunction in the early perioperative course after heart transplantation?



    • A.

      Antithymocyte globulin (ATG)


    • B.

      Basiliximab


    • C.

      Methylprednisolone


    • D.

      Mycophenolate


    • E.

      Tacrolimus




Preferred response: E


Rationale


Immunosuppression protocols are similar from center to center with generally only small variations. Initial immune suppression protocols typically include high-dose corticosteroids, induction with IL-2 receptor blockade (e.g., basiliximab) or antithymocyte globulin (ATG), followed within approximately 48 hours by introduction of a calcineurin inhibitor (CNI) such as cyclosporine or tacrolimus.


Induction protocols using IL-2 receptor antagonists or ATG are effective in delaying the time to first allograft rejection episode and the time needed to initiate CNI medications, which is especially useful when there is significant renal dysfunction. Induction therapy also reduces the risk of death due to rejection, although it does not appear to have a long-term survival benefit, except possibly in those with a PRA >50% or diagnosis of congenital heart disease. Induction therapy may also be useful in steroid avoidance protocols. Some centers do not use induction therapy under particular circumstances due to concerns for risk of infection or viral reactivation, although that has not been borne out in recent studies.


Corticosteroids have been part of standard protocols since the early days of solid organ transplantation. High-dose methylprednisolone (5 to 10 mg/kg) is administered at the time of aortic cross-clamp removal and continued in tapering doses over the first several days after surgery. Corticosteroids have immunosuppressive properties and benefit the allograft because of membrane-stabilizing and antioxidant effects on the graft. Steroid-sparing/steroid-avoidance protocols exist for other solid organ transplants and are in development for heart transplantation.


More controversial is the timing of the introduction of the CNIs cyclosporine and tacrolimus. A major complication in the early perioperative course after heart transplantation is renal dysfunction; in the past, CNIs were major contributors.


Acute kidney injury is a major complication following orthotopic heart transplantation. It is often multifactorial in etiology, as the premorbid risk factors of heart transplant recipients cannot necessarily be controlled. One must monitor and control use of calcineurin-inhibitor immune suppression agents, especially when renal dysfunction is present or expected. Therapeutic strategies include delaying the initiation of cyclosporine and tacrolimus by using antithymocyte globulin or IL-2 receptor blockade for induction of immune suppression. The other option is to use a modified oral/nasogastric protocol for tacrolimus administration. This protocol targets tacrolimus levels to below 6 ng/mL in the first 3 days after transplantation, followed by a rapid increase in dosing and target level over the next 4 days. It is important to avoid early IV administration of these agents because they invariably lead to renal afferent arteriolar vasoconstriction and oliguria. If renal dysfunction is complicating the posttransplant course, it is still difficult to withdraw CNIs completely, but lowering the target level to less than 6 ng/L and substituting higher doses of mycophenolate mofetil and adding sirolimus are reasonable options.



  • 14.

    An 8-year old girl with restrictive cardiomyopathy is being evaluated for transplant. A hemodynamic study is performed as part of the pretransplant evaluation. Hemodynamics are as follows: blood pressure, 110/80 mm Hg; pulmonary artery, 62/40 mm Hg, mean 50 mm Hg; pulmonary artery wedge, 35 mm Hg; left ventricular end-diastolic pressure, 35 mm Hg; and cardiac index, 2.5 L/min/m 2 . What is your decision regarding this patient’s suitability for orthotopic heart transplant?



    • A.

      Before making a decision, hemodynamics should be repeated with a fraction of inspired oxygen of 1.0 and nitric oxide.


    • B.

      Listing the patient for a heart transplant should be delayed until medical management of pulmonary hypertension can be initiated.


    • C.

      The patient is a candidate for heart and lung transplant only.


    • D.

      The patient is a candidate for orthotopic heart transplant with a recognized increased risk factor of elevated pulmonary vascular resistance (PVR).




Preferred response: D


Rationale


The evaluation of pulmonary vascular resistance (PVR) in the potential heart transplant recipient is a critical part of the evaluation. The right ventricle of the donor heart will acutely fail if it is exposed to excessive afterload from pulmonary vascular disease of the recipient. A pretransplant hemodynamic assessment with conditions favoring pulmonary vasodilation using oxygen, nitric oxide, and or nitroprusside is necessary to determine if the recipient is an orthotopic heart transplant candidate alone or will need to be referred for a heart and lung transplant. A transpulmonary gradient (pulmonary artery mean pressure–left atrial pressure) of less than 15 mm Hg or an estimated PVR index of less than 4 to 6 units/m 2 is acceptable level of PVR for orthotopic heart transplantation.


Chapter 38 : Physiologic foundations of cardiopulmonary resuscitation




  • 1.

    The odds of death are increased after extracorporeal cardiopulmonary resuscitation (E-CPR) with the following risk factors?



    • A.

      Age


    • B.

      Preexisting renal insufficiency


    • C.

      Shorter CPR times prior to ECMO


    • D.

      Venovenous (VV) ECMO Cannulation




Preferred response: B


Rationale


Data from the Extracorporeal Life Support Organization and AHA “Get with the Guidelines-Resuscitation” registries to determine risk factors related to unfavorable outcomes with E-CPR among 593 children. In this study they found that odds of death were increased with a noncardiac diagnosis and preexisting renal insufficiency, and that for each additional 5 minutes of CPR prior to ECMO initiation, the odds risk of death increased by 1.04. Age was not found to increase the odds of death. Longer CPR times (rather than shorter times) were found to have increased risk of death. E-CPR refers to veno-arterial rather than venovenous cannulation.



  • 2.

    Cerebral blood flow during CPR is increased with the following medication?



    • A.

      Dexmedetomidine


    • B.

      Metoprolol


    • C.

      Milrinone


    • D.

      Phenylephrine




Preferred response: D


Rationale


Cerebral blood flow during CPR depends on peripheral vasoconstriction that can be enhanced using an α-adrenergic agonist or arginine vasopressin receptor agonist (V1 receptor). This action produces selective vasoconstriction of noncerebral peripheral vessels to areas of the head and scalp without causing cerebral vasoconstriction. As with myocardial blood flow, pure α-agonist agents are as effective as epinephrine in generating and sustaining cerebral blood flow during CPR in adult animal models and in infant models. A β-blocking agent such as metoprolol (β 1 adrenergic receptor) would have a negative effect on cardiac output and no direct effect on systemic vascular resistance. A selective α 2 adrenoreceptor agonist such as dexmedetomidine would activate G-proteins via α 2a adrenoreceptors in the brainstem and inhibit norepinephrine release, causing no effect on the peripheral vasculature. Finally, use of a selective phosphodiesterase inhibitor such as milrinone would induce inhibition of cardiac and vascular tissue, resulting in vasodilation.



  • 3.

    What is the most common presenting rhythm in a pediatric patient in cardiac arrest?



    • A.

      Asystole


    • B.

      Bradycardia


    • C.

      Torsades de pointes


    • D.

      Ventricular fibrillation




Preferred response: A


Rationale


Asystole is the most common presenting rhythm in a pediatric patient who presents in cardiac arrest, noted in 25–70% of victims. Bradycardia and pulseless electrical activity (PEA) are other common rhythms, while ventricular rhythms are infrequent. Systemic disturbances, such as hypoxia, acidosis, sepsis, and hypovolemia, often precede the arrest and lead to the asystole rhythm.



  • 4.

    A 16-year-old male with no previous medical history had a witnessed collapse on the basketball court, and the automated external defibrillator (AED) recommended defibrillation, which was given. The patient arrives to your emergency room in cardiac arrest, having received several rounds of CPR with defibrillation attempts. Which antiarrhythmic would NOT be appropriate?



    • A.

      Amiodarone


    • B.

      Lidocaine


    • C.

      Procainamide


    • D.

      Sotalol




Preferred response: D


Rationale


Sotalol is not an appropriate antiarrhythmic choice for a ventricular arrhythmia in an acute cardiac arrest. According to the 2015 AHA guidelines, amiodarone and lidocaine are appropriate first-line agents, though neither has been shown to have a survival benefit over the other. Amiodarone given intravenously may depress myocardial function and lead to hemodynamic collapse, which may not be a concern in a patient undergoing active cardiopulmonary resuscitation. The half-life of amiodarone is on the magnitude of days, so it will take several doses to appropriately load a patient. It is contraindicated in patients with torsades de pointes as it increases the QTc and would exacerbate the arrhythmia. Lidocaine has a shorter half-life (5–10 minutes) though it may depress myocardial function at high plasma concentrations. Procainamide may also be used to treat ventricular arrhythmias, though like amiodarone it also prolongs the QTc.



  • 5.

    What is the major pharmacologic effect of epinephrine during cardiopulmonary resuscitation?



    • A.

      It raises aortic systolic pressure.


    • B.

      It raises aortic diastolic pressure.


    • C.

      It raises the heart rate via its β-agonist actions.


    • D.

      It improves cardiac compliance by relaxing myocardium during diastole.




Preferred response: B


Rationale


At resuscitative doses of epinephrine, the α-adrenergic receptor effect predominates, increasing systemic vascular resistance in addition to increasing cardiac output. The α-adrenergic–mediated vasoconstriction of epinephrine increases aortic diastolic pressure and thus coronary perfusion pressure, a critical determinant of successful resuscitation.


At nonresuscitative doses, epinephrine has potent β 1 -adrenergic receptor activity and moderate β 2 – and α 1 -adrenergic receptor effects. Clinically, low doses of epinephrine increase cardiac output because of the β 1 -adrenergic receptor inotropic and chronotropic effects, whereas the α-adrenergic receptor–induced vasoconstriction is often offset by the β 2 -adrenergic receptor vasodilation. The result is an increased cardiac output, with decreased systemic vascular resistance and variable effects on the mean arterial pressure.



  • 6.

    Compared with monophasic defibrillators, which of the following is true about biphasic defibrillators?



    • A.

      They are too expensive and have an unacceptable fault rate.


    • B.

      They have been shown to burn too much myocardium in smaller children.


    • C.

      They require less energy output to be effective.


    • D.

      They should be used at one-fourth the starting dose in children.




Preferred response: C


Rationale


Defibrillators can deliver energy in a variety of waveforms that are broadly characterized as monophasic or biphasic. Biphasic waveforms defibrillate more effectively and at lower energies than do monophasic waveforms. The value of higher energy biphasic shocks was demonstrated in the BIPHASIC trial.



  • 7.

    A 16-year-old female with a history of chronic medications, including metoclopramide, ondansetron, and erythromycin, presents conscious with the tracing shown in Figure 136.15 , below. Which of the following antiarrhythmic agents would be most appropriate in the acute setting?



    • A.

      Adenosine


    • B.

      Amiodarone


    • C.

      Lidocaine


    • D.

      Procainamide




    • Fig. 136.15



Preferred response: C


Rationale


Due to her medication profile, this patient is at risk for acquired long QT syndrome (LQTS); this tracing represents torsades de pointes (TdP). TdP is a form of polymorphic ventricular tachycardia that occurs in the setting of either acquired or congenital LQTS, where it appears as though the QRS is twisting around an isoelectric axis. Delay in treatment can cause the rhythm to degenerate into ventricular fibrillation. The first-line treatment for TdP is IV magnesium, which is effective in both treatment and prevention of TdP. It is effective even in the setting of a normal serum magnesium level. Of the antiarrhythmics presented, lidocaine would be the most appropriate choice in the acute management of TdP, as it shortens the duration of the action potential. Both amiodarone and procainamide prolong the QT interval, which worsens the pathophysiology of this arrhythmia. Adenosine is used primarily to diagnose and treat supraventricular tachycardias. It would not treat TdP. In the setting of an unstable patient, ventricular defibrillation would be the first-line therapy.


Chapter 39 : Performance of cardiopulmonary resuscitation in infants and children




  • 1.

    A 6-year-old child with a history of asthma is brought to the emergency department by paramedics with cardiopulmonary resuscitation (CPR) in process. She developed severe respiratory distress and was intubated during the ambulance ride to the hospital. Appropriate position of the endotracheal tube was documented, including exhaled end-tidal carbon dioxide (ETCO 2 ). About 2 minutes prior to her arrival, she became progressively hypoxemic and bradycardic and cardiopulmonary resuscitation (CPR) was started. Upon arrival to the resuscitation bay, she is transferred to the stretcher with ongoing CPR, ETCO 2 monitoring is confirmed, and a CPR quality recording defibrillator is placed. Which of the following is the correct compression-to-ventilation ratio and frequency of chest compressions in this situation?



    • A.

      15 compressions: 2 ventilations, at least 100 compressions/minute


    • B.

      30 compressions: 2 ventilations, no more than 120 compressions/minute


    • C.

      Continuous chest compressions at a rate of 100 to 120 compressions/minute with 10 breaths per minute (asynchronous ventilation)


    • D.

      100–120 compressions/minute without ventilations (“hands-only” CPR)




Preferred response: C


Rationale


When a child has an invasive airway in place during CPR, guidelines recommend the provision of continuous chest compressions at a rate of 100 to 120 per minute with asynchronous ventilations. To avoid excessive ventilation, ventilations should be provided at a rate of 1 breath every 6 seconds (10 breaths per minute).



  • 2.

    During ongoing resuscitation of the child above, an arterial line is placed in the femoral artery. Transduction during active CPR demonstrates a blood pressure of 75/22 mm Hg. Among these markers of CPR quality, which one has been associated with improved survival to hospital discharge with favorable neurological outcome after pediatric cardiac arrest?



    • A.

      Chest compression rate 100–120 per minute


    • B.

      Depth of compression >50 mm


    • C.

      Diastolic blood pressure ≥25 mm Hg in infants and ≥30 mm Hg in older children


    • D.

      ETCO 2 during CPR ≥20 mm Hg




Preferred response: C


Rationale


In a recent large registry study of the Collaborative Pediatric Critical Care Research Network (CPCCRN), among children with an arterial line in place at the time of the arrest, a diastolic blood pressure ≥25 mm Hg in infants and ≥30 mm Hg in older children was associated with improved survival to discharge and survival to discharge with favorable neurological outcome compared to lower blood pressures.



  • 3.

    Following successful resuscitation from cardiac arrest, which of the following represent an optimal set of goals during the post-arrest period?



    • A.

      Blood pressure >5th percentile for age; Paco 2 35–50 mm Hg; normal Pao 2 ; avoidance of fever


    • B.

      Core body temperature 32–34°C; Paco 2 <30 mm Hg; blood pressure >5th percentile for age


    • C.

      Core body temperature 35–37°C; Paco 2 <30 mm Hg; blood pressure >5th percentile for age


    • D.

      Serum glucose 60–100 mg/dL; blood pressure >5th percentile for age; normal Paco 2




Preferred response: A


Rationale


Goals for the provision of post cardiac arrest care include:




  • Avoidance of hypotension (provision of isotonic fluids, vasopressors, and inotropic agents to maintain blood pressure >5th percentile for age).



  • Avoidance of fever with continuous core body temperature monitoring and targeted temperature management to maintain normothermia. Therapeutic hypothermia can be considered following out-of-hospital cardiac arrest (OHCA).



  • Avoidance of significant hypocapnia/hypercapnia or hypoxemia/hyperoxia.



  • Monitoring for seizures.





  • 4.

    A 14-year-old girl was found unresponsive and CPR is being provided for pulseless electrical activity cardiac arrest. During a pulse and rhythm check, there is no pulse palpable and the monitor and defibrillator display a disorganized rhythm that appears to be ventricular fibrillation (VF). You direct the team to resume chest compressions. What is your next step in management?



    • A.

      Amiodarone 5 mg/kg intravenously


    • B.

      Continued CPR according to the PEA algorithm, as this was the original rhythm


    • C.

      Defibrillation for VF of 2 J/kg


    • D.

      Synchronized cardioversion for VF: 0.5–1 J/kg




Preferred response: C


Rationale


Though they encompass a smaller proportion of arrest rhythms relative to adults, ventricular fibrillation (VF) and pulseless ventricular tachycardia (pVT) do occur in children and require vigilance by clinicians to identify and appropriately treat. Moreover, a substantial number of children have VF or pVT as a subsequent cardiac arrest rhythm—that is, they initially have a nonshockable rhythm for which CPR is initiated and then develop VF/pVT. Therefore constant reassessment during resuscitation is paramount. In addition to ongoing high-quality CPR, the first-line treatment of VF or pVT is defibrillation, following which CPR should be immediately resumed. If two defibrillation attempts are unsuccessful, amiodarone or lidocaine should be administered, in addition to ongoing defibrillation attempts and CPR. Minimization of peri-defibrillation interruptions in chest compressions has been associated with improved outcomes.



  • 5.

    A healthy 12-year-old is playing baseball and is hit in the center of the chest by a pitch. He suddenly collapses and loses consciousness, unresponsive to stimulation, with an occasional gasp (agonal) breath. When paramedics arrive, the first ECG rhythm they are likely to see, is:



    • A.

      Asystole


    • B.

      Pulseless electrical activity


    • C.

      Sinus bradycardia


    • D.

      Ventricular fibrillation




Preferred response: D


Rationale


Following a sharp blow to the chest with sudden collapse, called “commotion cordis,” the most common cause of arrest is an R on T phenomena leading to ventricular fibrillation. If an AED (automated external defibrillator) had been applied prior to EMS arrival, a shock would have been advised.



  • 6.

    A 12-year-old with severe asthma has respiratory distress unresponsive to albuterol aerosols and oxygen. The child becomes confused, then unconscious and unresponsive. The Spo 2 was 90%, then 70% and now is not picking up. There is no pulse palpable and the arterial catheter tracing is flat (nonpulsatile) on the monitor. The child has an occasional gasp (agonal) breath. The ECG rhythm on the monitor is most likely:



    • A.

      Pulseless electrical activity


    • B.

      Sinus bradycardia


    • C.

      Ventricular fibrillation


    • D.

      Ventricular tachycardia




Preferred response: A


Rationale


This child has progressive respiratory failure due to lower airway obstruction (asthma) with severe arterial desaturation, and has progressed to a pulseless rhythm, with evidence of no perfusion and no pulse (by palpation or arterial line). This is most likely pulseless electrical activity. One should review the H’s and T’s for potential causes of PEA, with high suspicion for pneumothorax in a severe asthmatic.



  • 7.

    During cardiopulmonary resuscitation (CPR), which of the following bedside monitoring tools is most predictive of return of spontaneous circulation (ROSC)?



    • A.

      Exhaled CO 2 capnograph of >15 Torr


    • B.

      Femoral pulse palpable with chest compressions


    • C.

      Pacer spikes visible on electrocardiogram when transcutaneous pacing is initiated


    • D.

      Pulse oximeter waveform detected with chest compressions




Preferred response: A


Rationale


During the low-flow phase of CPR, achieving optimal cardiac output/coronary perfusion pressure is consistently associated with an improved chance of return of spontaneous circulation. Bedside capnography can be useful as a rough estimate of pulmonary blood flow. Achieving an optimal exhaled carbon dioxide concentration >15 Torr has been associated with improved short-term outcome in both animal and human studies.



  • 8.

    A previously healthy 7-year-old boy under evaluation for syncope suddenly collapses. His cardiac arrest rhythm looks like ventricular fibrillation (VF) on the monitor. If high-quality standard CPR was provided (with chest compressions and 100% fraction of inspired oxygen rescue breathing), an arterial blood gas (ABG) and venous blood gas (VBG) drawn just prior to defibrillation would most likely show:



    • A.

      ABG: pH 7.10, Pco 2 65, Po 2 300 VBG: pH 7.35, Pco 2 65, Po 2 40


    • B.

      ABG: pH 7.10, Pco 2 55, Po 2 50 VBG: pH 7.10, Pco 2 55, Po 2 50


    • C.

      ABG: pH 7.05, Pco 2 25, Po 2 20 VBG: pH 7.25, Pco 2 55, Po 2 35


    • D.

      ABG: pH 7.30, Pco 2 30, Po 2 300 VBG: pH 7.10, Pco 2 75, Po 2 30




Preferred response: D


Rationale


Provision of high-quality CPR (i.e., push hard, push fast but not too fast, allow full chest recoil, minimize interruptions, and don’t overventilate) can result in cardiac output that approaches 50% of normal. The other choices have either inappropriately low Po 2 levels given this scenario or a venous pH that is higher than the one recorded from the corresponding ABG.


Chapter 40 : Structure and development of the upper respiratory system




  • 1.

    A child with 22q11 deletion syndrome and congenital heart disease presents with a difficult intubation due to a laryngeal web. Which abnormality in the normal larynx embryogenesis caused this to occur?



    • A.

      Atresia as the most common result of failure to recanalize


    • B.

      Cartilages and muscles derived from third and fourth branchial arches


    • C.

      Derivation of epithelium from mesoderm of the laryngotracheal tube


    • D.

      Formation of epiglottis by mesenchyme proliferation of the fourth and sixth branchial arches


    • E.

      Temporary occlusion of larynx until the 10th week when recanalization occurs




Preferred response: E


Rationale


As epithelium proliferates rapidly, the temporary occlusion of the larynx ends by the 10th week when recanalization occurs. Atresia is the least common result of failure to recanalize. Laryngeal webs and/or stenosis may be common results of failure to recanalize. Branchial arches involved in embryogenesis of the larynx are as follows: cartilages and muscles are derived from the fourth and sixth branchial arches, whereas the third and fourth branchial arches form the epiglottis by mesenchyme proliferation. Derivation of epithelium is from endoderm of the laryngotracheal tube, not mesoderm.



  • 2.

    A newborn intubation maybe challenging due to the anterior superior location of the larynx compared to a toddler. What oropharyngeal changes with growth and development account for this?



    • A.

      Growth in the oropharynx is primarily in the anteroposterior direction.


    • B.

      Lateral walls of the pharynx consist of a pair of constrictor muscles innervated by cranial nerves V, IX, and X.


    • C.

      Oropharynx is the crossroads for the soft palate above and the hypopharynx below.


    • D.

      The oropharynx is prominent in young infants.


    • E.

      The oropharynx is first evident in children between ages 2 and 3 years.




Preferred response: E


Rationale


In young infants there is no defined oropharynx. The nasopharynx and hypopharynx are contiguous. Over the first 2–3 years, growth is primarily in the vertical direction such that a distinct oropharynx becomes evident, usually between 2 and 3 years of age. Three pairs of constrictor muscles are seen in the lateral walls of the pharynx, not just one pair. The innervation is via cranial nerves, V, IX, and X. The oropharynx forms the crossroads for the nasopharynx above and the hypopharynx below. The soft palate is the muscular extension of the bony hard palate and a critical structure for occlusion of the nasal cavity while eating and drinking, as well as retraction is important for speech production.



  • 3.

    What is narrowest part of the upper airway in infants?



    • A.

      Larynx


    • B.

      Nasal valve


    • C.

      Nasopharynx


    • D.

      Subglottis




Preferred response: B


Rationale


The structure of the upper airway differs in the infant, young child, and young adult. Preferential nasal breathing is present in neonates and persists until 6 months of age because of the high-riding larynx in the neck with the soft palate and vallecula in close anatomic approximation. The nasal tip—in particular, the nasal valve area—is the area of highest resistance in the upper airway of the infant.



  • 4.

    A pediatric patient requires intubation. Laryngoscopy shows an easy grade 1 view; however, the age-appropriate endotracheal tube is difficult to pass. Which is not a possible reason for this to happen?



    • A.

      Laryngeal web


    • B.

      Subglottic stenosis


    • C.

      Tracheal stenosis


    • D.

      Tracheomalacia




Preferred response: D


Rationale


Tracheomalacia would not obstruct passing of the endotracheal tube because it is weakness or external compression of the trachea. It may hinder ventilation but the endotracheal tube should be able to pass easily.



  • 5.

    What method can be used to reach a definitive diagnosis of a laryngeal cleft?



    • A.

      Fiberoptic endoscopic examination of swallowing (FEES)


    • B.

      Flexible nasopharyngoscopy


    • C.

      Operative endoscopy with palpation of the larynx


    • D.

      Videofluoroscopic swallow study (VFSS)




Preferred response: C


Rationale


To assess for a laryngeal cleft, palpation of the interarytenoid area is the gold standard.


Chapter 41 : Structure and development of the lower respiratory system




  • 1.

    A baby is born at 24 weeks’ gestation and has a difficult respiratory course in the NICU. She eventually gets a tracheostomy for chronic ventilation and is transferred to the PICU at 6 months of age. Her oxygen requirement has been decreasing, and she is discharged home on a ventilator. Which of the following is a correct statement regarding the development of this child’s lower respiratory system?



    • A.

      Alveolarization is complete in this child.


    • B.

      The acinus is the gas exchange area of the lung.


    • C.

      The pulmonary veins course with the airways and the bronchioles throughout the lung.


    • D.

      Type I alveolar epithelial cells produce and secrete surfactant.




Preferred response: B


Rationale


Surfactant is produced by type II cells, not type I cells. Gas exchange occurs in the acinus. Alveolarization appears to be nearly complete at about age 2 years and so should continue to occur in this child. The pulmonary veins do not normally course with the airways and bronchioles. When they do, this suggests alveolar-capillary dysplasia.



  • 2.

    The primary role of the lung is gas exchange. What structural feature of the lung supports this function?



    • A.

      Alveolar macrophages are scarce in the normal lung.


    • B.

      The connective tissue space, or interstitium of the lung at the alveolar level, has abundant lymphatics.


    • C.

      The internal surface area of the adult lung is 70 to 80 m 2 , of which 90% covers the pulmonary capillaries.


    • D.

      When fully matured the pulmonary artery and its thickness is approximately 90% that of the aorta.




Preferred response: C


Rationale


The internal surface area of the adult lung is 70 to 80 m 2 , of which 90% covers the pulmonary capillaries; thus the air-blood surface available for gas exchange is 60 to 70 m 2 . When fully matured, the pulmonary artery and its thickness is only about 60% that of the aorta. Alveolar macrophages are abundant and form an important arm of the defense mechanism of the lung. The connective tissue space, or interstitium of the lung at the alveolar level, does not have lymphatics, but it can accumulate fluid that can be absorbed into the lymphatic system, which ends usually at the respiratory bronchiolar level.



  • 3.

    Which of the following is the most predominant bronchial mucosal cell type?



    • A.

      Basal cells


    • B.

      Brush cells


    • C.

      Ciliated cells


    • D.

      Neuroendocrine cells




Preferred response: C


Rationale


The bronchial mucosa contains several epithelial cell types: ciliated, mucus producing (goblet cells), basal, brush, and neuroendocrine. Ciliated cells constitute more than 90% of the epithelial cell population in the conducting airways, but the proportion and number of cilia per cell decrease from the proximal to the distal airways. In addition to its ciliary beating movement, the ciliated columnar cells regulate the depth of the composition of the periciliary fluid and transport ions across the epithelium. The basal cell has a progenitor cell role and also functions to maintain adherence of columnar cells to the basement membrane. The brush cell, thought to have a role in fluid absorption or chemoreceptor function, is found rarely in the tracheobronchial and alveolar epithelia. Although mucous goblet cells secrete mucin, it is the submucosal glands that produce more than 90% of the mucus needed for mucociliary function.


Neuroendocrine cells can be solitary near the basal lamina between columnar cells or in collections called neuroepithelial bodies that occur near branch points of bronchi. A number of neural markers are expressed (e.g., 5-hydroxytryptamine, chromogranin A, neuron-specific enolase, and synaptophysin), and a number of hormones are produced (e.g., endothelin, calcitonin, and bombesin [gastrin-releasing peptide]). They are more abundant in the fetus and likely have a role in lung growth or maturation.



  • 4.

    It is not known with certainty when alveolar development is completed. However, based on our current knowledge, we believe which of the following statements?



    • A.

      All alveoli are present at birth.


    • B.

      All alveoli are present at 16 weeks’ gestation.


    • C.

      Alveoli continue to develop until 2 to 8 years of age.


    • D.

      Alveoli continue to develop into adulthood.




Preferred response: C


Rationale


At birth, primitive alveoli called saccules are evident, but approximately 50 million alveoli are already formed. The number of alveoli in a normal adult can vary from 300 to 500 million, and they have a diameter of 150 to 200 µm. The early work by Dunnell suggesting that new alveolar formation ceased at about age 8 years has been challenged by Thurlbeck, who has shown that alveolarization appears to be nearly complete at about age 2 years. Lung volume correlates with body size, but alveolar surface area correlates with metabolic activity; thus alveoli become more complex in shape during maturation and as increasing O 2 is required.


Chapter 42 : Physiology of the respiratory system




  • 1.

    Regarding respiratory physiology, which one of the following statements is least accurate?



    • A.

      Peripheral airway resistance in children <5 years is fourfold higher than in older children and adults.


    • B.

      Specific compliance is the same for adults and children, but specific conductance is higher in children.


    • C.

      With laminar flow, resistance to flow is proportional to viscosity.


    • D.

      With turbulent flow, resistance to flow is proportional to density.




Preferred response: B


Rationale


Compliance is a measure of the elastic nature of the chest wall and lungs. In children, due to decreased development of the structure of the lungs and developing calcification of the ribs, the compliance of both the lungs and chest wall is decreased.



  • 2.

    Which one of the following clinical conditions is not expected to be associated with sudden decline in end-tidal CO 2 as it relates to the alveolar gas equation?



    • A.

      Air embolism


    • B.

      Cardiac standstill


    • C.

      Hypoventilation


    • D.

      Obstruction of the endotracheal tube




Preferred response: C


Rationale


The volume of air entering the lungs each minute that actually participates in gas exchange is called the alveolar ventilation (A). It is therefore the difference between the total volume of air entering the lungs each minute (minute ventilation, E) and the volume of air entering the lungs that does not participate in gas exchange (dead space: D): A = E − D. Anything decreasing the amount of CO 2 seen by the end-tidal sensor will decrease the level displayed, but a sudden decrease represents a sudden change, whereas hypoventilation would demonstrate a gradual increase.



  • 3.

    Lack of oxygen equilibration due to diffusion limitation (alveolar-capillary block) can be evaluated by measuring which of the following?



    • A.

      Diffusing capacity of CO (carbon monoxide)


    • B.

      Diffusing capacity of CO 2 (carbon dioxide)


    • C.

      Distribution of an inhaled gas mixture containing a radioactive marker


    • D.

      FEV 1 /FVC when inhaling pure oxygen




Preferred response: A


Rationale


DCO is a good index of the diffusion capacity of oxygen (D o 2 ).



  • 4.

    Reduction of the pulmonary diffusing (D) capacity to one-fourth of its normal value would be expected to have what effect on systemic arterial oxygen and carbon dioxide partial pressures (compared to normal)?



    • A.

      Decrease Pao 2 and decrease Paco 2


    • B.

      Decrease Pao 2 but no change in Paco 2


    • C.

      Increase Pao 2 and decrease Paco 2


    • D.

      Increase Pao 2 and increase Paco 2




Preferred Response: B


Rationale


Pao 2 decreases when the diffusion capacity of oxygen (D o 2 ) decreases to less than one-third its normal value. But D o 2 is so high normally that even a decrease to one-fourth will still permit carbon dioxide to equilibrate in the time that blood passes through pulmonary capillaries.


Chapter 43 : Noninvasive respiratory monitoring and assessment of gas exchange




  • 1.

    Which of the following is least likely associated with a source of error when measuring oxygen saturation via pulse oximetry (Spo 2 )?



    • A.

      Increased proportion of the oxidized form of hemoglobin


    • B.

      Presence of fetal hemoglobin (HgbF)


    • C.

      Probe placement on a digit of a patient immediately status post–cold water submersion


    • D.

      Probe placement on the right second digit of a patient experiencing bilateral upper extremity tonic-clonic seizure activity




Preferred response: B


Rationale


Fetal hemoglobin and adult hemoglobin have nearly identical absorption. Studies have demonstrated that Spo 2 measurements by pulse oximetry are unaffected by the presence of HgbF in neonates. All other choices are associated with known sources of error for pulse oximetry readings. Specifically, patient movement (such as seizure activity in answer D) can be associated with poor signal and inaccurate pulse oximetry readings. Additionally, local perfusion and patient temperature may affect the probe’s ability to measure Spo 2 due to alterations in the local pulsatile blood flow. In patients with cold extremities and significant peripheral vasoconstriction (expected in a patient status post–cold water submersion in answer C), perfusion to the digits is likely significantly impaired and will affect the accuracy of pulse oximetry readings.



  • 2.

    Which of the following is true regarding near-infrared spectroscopy (NIRS)?



    • A.

      NIRS measures only light absorbed by hemoglobin.


    • B.

      NIRS measurement is not affected by skin temperature.


    • C.

      Values from NIRS may be helpful for trending a patient’s oxygenation and hemodynamic status.


    • D.

      When placed on the forehead, NIRS provides a measurement that is equivalent to the mixed venous saturation.




Preferred response: C


Rationale


Near-infrared spectroscopy (NIRS) is a noninvasive method utilized to measure the Hgb-oxygen saturation of a local region of interest. This is achieved through the use of multiple (2–4) wavelengths of near-infrared light directed via a cutaneous probe into the underlying tissue, which are absorbed by pigments such as myoglobin, hemoglobin, and cytochrome. Studies have reported varying degrees of accuracy for cerebral NIRS oximeters and have also found large variation in reading errors. Extracranial tissue changes due to superficial vasoconstriction secondary to vasoactive medication (e.g., norepinephrine, phenylephrine) or sympathetic response to pain or hypothermia can be additional sources of error with cerebral NIRS oximetry. Based on these findings, many authors recommend the use of cerebral oximetry as trend monitors and not absolute tissue oxygenation measures or injury threshold determinants.



  • 3.

    A standard pulse oximeter is able to detect:



    • A.

      Carboxyhemoglobin


    • B.

      Methemoglobin


    • C.

      Oxygenated hemoglobin only


    • D.

      Oxygenated and deoxygenated hemoglobin




Preferred response: D


Rationale


Pulse oximetry is based on the observation that the attenuation of light passing through blood-perfused tissue changes with pulsation of blood and that the alternating component of the light attenuation results from the composition of arterial blood. Hemoglobin has characteristic light-absorbing properties that change with oxygen binding. The deoxy form of hemoglobin (deoxyHb) has a single peak in the visible and near-infrared region. Oxyhemoglobin (oxyHb) has two peaks in the visible region but no significant peak in the near-infrared region. At any given wavelength, there is a difference in absorption between oxyHb and deoxyHb except where the spectra cross, at wavelengths called isosbestic wavelengths, where the absorption is the same for each state. At nonisosbestic wavelengths, the difference in absorption can be used to determine the fraction of oxyhemoglobin. Saturation of hemoglobin is defined as follows:


Hbsat=[OxyHb]/([OxyHb]+[DeoxyHb])
Hbsat=[OxyHb]/([OxyHb]+[DeoxyHb])


where Hb sat = fractional saturation of hemoglobin, [oxyHb] = concentration of oxyhemoglobin, and [deoxyHb] = concentration of deoxyhemoglobin. Hemoglobin percent saturation, as commonly reported, is determined by multiplying Hb sat by 100.



  • 4.

    Which of the following are the most reliable sites for monitoring rapidly changing core temperature?



    • A.

      Distal esophagus, tympanic membrane, pulmonary artery, and nasopharynx


    • B.

      Distal esophagus, pulmonary artery, bladder, and rectum


    • C.

      Skin, bladder, and rectum


    • D.

      Skin, bladder, and nasopharynx




Preferred response: A


Rationale


Commonly used core temperature monitoring sites include the distal esophagus, tympanic membrane, pulmonary artery, and nasopharynx. These sites detect core temperature changes rapidly, in contrast to urinary bladder or rectal measurements, which are good reflections of core temperature during steady-state conditions. Cutaneous temperature monitoring is the least reliable indicator of rapid core temperature changes. However, monitoring peripheral temperatures can be useful in defining core peripheral gradients in temperature and assist in tracking vasoconstriction and vasodilation. Oral probes are used as thermometers, and some have been attached to pacifiers. A thermometer that scans the temporal artery is also available. The ideal spot for continuous core temperature monitoring is a pulmonary artery catheter, but because of the invasive nature of this monitor, it would never be placed for temperature monitoring alone.


An esophageal temperature probe positioned in the lower third of the esophagus is a good alternative. In this position, the temperature sensor is immediately behind the left atrium and accurately tracks core temperature without significant time lag in the majority of situations. If a gastric tube with applied suction is present next to the temperature probe, it must be on the low intermittent setting or the temperature readings will be falsely lowered. Nasopharyngeal and tympanic membrane temperatures are good indicators of cerebral temperature but can be inaccurate as a result of sensor positioning. Axillary and peripheral skin probably are the most convenient sites for monitoring temperatures, but they also are the most inaccurate because of skin perfusion.



  • 5.

    You are leading resuscitation for a 2-year-old boy who suffers a bradycardic cardiac arrest requiring cardiopulmonary resuscitation (CPR). He receives approximately 2 minutes of high-quality CPR and then a code dose of epinephrine. After approximately 30 seconds you noticed a change in his end-tidal CO 2 waveform ( Figure 136.16 , below). What does this change in capnography most likely indicate?



    • A.

      Dislodgement of the endotracheal tube


    • B.

      Inadequate depth of chest compressions


    • C.

      Presence of lower airway obstruction


    • D.

      Restoration of pulmonary blood flow through return of spontaneous circulation (ROSC)




    • Fig. 136.16



Preferred response: D


Rationale


Capnography has been shown to be a useful tool in many clinical situations. Recent studies have demonstrated that use of end-tidal CO 2 monitoring during CPR can help improve the quality of CPR and provide insight into the patient’s physiology. Recent Advanced Cardiac Life Support (ACLS) and PALS guidelines recommend using capnography to monitor the effectiveness of chest compressions during CPR. A sudden decrease in PETCO 2 is seen with loss of pulmonary blood flow in cardiac arrest. Increasing PETCO 2 values generated during CPR are associated with chest compression depth and ventilation rate, and an acute rise in PETCO 2 exceeding 10 mm Hg is seen with return of spontaneous circulation. Patients with ROSC after CPR have statistically higher levels of PETCO 2 , suggesting better lung perfusion and cardiac output. Current guidelines suggest achieving a threshold of 10 to 15 mm Hg to ensure adequate delivery of chest compressions, although an average PETCO 2 level of 25 mm Hg was found in patients with ROSC in a recent systematic review and meta-analysis. In the above waveform, we see initial PETCO 2 values of approximately 15–20 mm Hg, suggesting adequate chest compression depth. We then see an acute rise in PETCO 2 up to levels of 40 mm Hg and above. This acute rise is characteristic of increased pulmonary blood flow and is suggestive of achieving ROSC.



  • 6.

    Which of the following clinical scenarios best explains this capnography waveform ( Figure 136.17 )?



    • A.

      Airway obstruction


    • B.

      Apnea


    • C.

      Esophageal intubation


    • D.

      Hypoventilation




    • Fig. 136.17



Preferred response: C


Rationale


Incorrect placement of the endotracheal tube in the esophagus results in initial detection of trace PETCO 2 , which is not sustained over time, and an uncharacteristic waveform that lacks a defined respiratory upstroke, plateau, or inspiratory downstroke. Hypoventilation may be indicated by a gradual rise of PETCO 2 , while airway obstruction is demonstrated by a change in the shape of the capnography waveform (absence of expiratory or alveolar plateau). Sudden loss of PETCO 2 with a waveform that transitions from normal to flat line is an indication of laryngospasm or apnea.


Chapter 44 : Overview of breathing failure




  • 1.

    Compared with adults, neonates are more predisposed to respiratory failure because:



    • A.

      Airway resistance is lower in neonates.


    • B.

      Neonates have a greater proportion of type 1 fibers in the diaphragm.


    • C.

      The neonatal diaphragm has a greater angle from the vertical than the adult.


    • D.

      The neonatal chest wall is less compliant than the adult.


    • E.

      The ribs are more vertical than the adult.




Preferred response: C


Rationale


Neonates are at a number of mechanical disadvantages for breathing when compared with older children and adults. Because the neonatal chest wall is more compliant than the adult, the ribs recoil more and are more horizontal. This in turn promotes the diaphragm to be less apposed to the chest wall and a greater angle of the diaphragm to the chest well. Both of these make the diaphragm less efficient. These effects all act together to promote restrictive lung disease in the normal neonate. Therefore, they are more tachypneic at baseline and susceptible to any further restrictive process.


In addition, neonates have smaller airways, which greatly increases resistance to airflow and predisposes to more severe obstructive airway disease (both inspiratory and expiratory). Airway resistance is inversely proportional to the fourth power of the radius of the airway. (R= where η is gas viscosity, L is airway length, and r is radius of the airways). Thus, small changes in the airway caliber due to edema or mucus can cause severe increase in the respiratory work a neonate must perform. Because the neonatal diaphragm is composed of a greater proportion of type 2 fibers, it tolerates prolonged loads less well and becomes fatigued more easily. Type 1 fibers, which are more prevalent in the adult and well-trained diaphragm, resist fatigue better than type 2.



  • 2.

    Of the following, which is a cause of central hypoventilation?



    • A.

      Botulism


    • B.

      Cerebral palsy


    • C.

      Metabolic acidosis


    • D.

      Phox2B mutation


    • E.

      Tetrodotoxin




Preferred response: D


Rationale


Central hypoventilation is caused by a decreased stimulus, or altered threshold, for respiratory rhythm generators in the central nervous system. Patients with the congenital central hypoventilation syndrome lack normal automatic control of breathing due to mutations in Phox2B, which is expressed in the retrotrapezoid nucleus of the medulla, one of the major chemosensitive regions controlling ventilation. Because chemosensitive regions in both the periphery and the brain sense pco 2 through local changes in pH, metabolic acidosis increases the drive to breathe, as opposed to decreases it.


Patients with central hypoventilation lack effort despite increased CO 2 tension. Disorders of the peripheral nervous system and muscles may also have inadequate respiratory effort. Tetrodotoxin, via inhibition of nerve impulses, suppresses breathing, though the central drive is intact. Similarly, botulism inhibits neurotransmission at the motor endplate and prevents muscle response to central stimuli.


Patients with cerebral palsy have a number of factors that predispose to respiratory failure. These include increased (and inappropriate) airway tone as well as scoliosis and other chest wall deformities that occur over time. However, these patients nearly always have adequate respiratory drive at baseline. A patient with cerebral palsy who lacks adequate respiratory drive should be evaluated for other causes (i.e., seizures, sepsis, drug overdose).



  • 3.

    What is the primary mechanism by which shock causes respiratory failure?



    • A.

      Cortical brain ischemia.


    • B.

      Inadequate muscle blood flow.


    • C.

      Metabolic acidosis.


    • D.

      Overstimulation of medullary respiratory centers.




Preferred response: B


Rationale


In shock, blood flow to respiratory muscles becomes inadequate for aerobic muscle work. Respiratory muscles first develop an oxygen debt and then lose their ability to contract. Metabolic acidosis stimulates chemoreceptors and brainstem respiratory sites, causing tachypnea and dyspnea, but not preventing muscle work. The high respiratory muscle workload of respiratory failure does increase muscle blood flow to the extent supportable by the circulation and is a strain on the heart, but it is not the cause of respiratory failure in shock. Cortical brain ischemia might impair volitional responses to respiratory distress, but it is the brainstem that governs automatic responses to respiratory stimuli. Overstimulation of medullary respiratory centers may drive tachypnea but is not the cause of respiratory muscle failure from shock.



  • 4.

    A child fell while rock climbing and suffered injury to his thoracic spinal cord at the level of T6. He has lower extremity flaccidity and a sensory level at the lower sternum and at the level of T6 posteriorly. What would be the greatest impact of this injury on breathing?



    • A.

      Dependence on accessory muscles of respiration


    • B.

      Diminished strength of cough


    • C.

      Loss of the cough reflex


    • D.

      Reduced inspiratory capacity




Preferred response: B


Rationale


The muscles of the abdomen are used for active expiration and are innervated by the lower thoracic spinal cord. They are important for coughing, sneezing, and for forced expiration, but play little role in passive expiration. The cough reflex involves abdominal muscles as well as deep intercostals, which are innervated by both upper and lower thoracic motor neurons. The reflex would be intact despite a T6 injury, but the cough itself would be weak. Inspiration is powered by the superficial and parasternal intercostal muscles and by the accessory muscles of respiration, so inspiratory capacity should be preserved. Dependence on accessory muscles for normal inspiratory power should not be necessary. Retractions reflect inspiratory effort, which should not be directly altered by this injury.



  • 5.

    Which of the following is a true statement about control of breathing?



    • A.

      Aortic and carotid chemoreceptors directly modulate respiratory drive.


    • B.

      Brainstem respiratory control is located in a single medullary nucleus.


    • C.

      Volitional and automatic control of breathing may be independently impaired.


    • D.

      Volitional control of breathing requires medullary modulation.




Preferred response: C


Rationale


Volitional control of respiration normally may be asserted in the awake state, but only up to a point. One cannot consciously hold the breath beyond a certain level of medullary stimulation. Supratentorial injury may impair volitional control of breathing but spare automatic control. On the other hand, medullary stroke may impair automatic control of breathing but leave volitional control intact.


Brainstem respiratory control is widely distributed in the pons and medulla and involves numerous other brainstem nuclei. All mechanoreceptor and chemoreceptor input that modulates respiratory drive must be processed in the brain before effector responses can occur. Aortic and carotid chemoreceptors modulate breathing only after integration in the medulla. Patients with loss of all cortical behavior may exhibit respiratory distress and use accessory muscles of breathing.



  • 6.

    Which of the following is a true statement about respiratory muscle exhaustion?



    • A.

      Myoglobin provides adequate oxygen reserves to protect against respiratory muscle exhaustion from hypoxemia.


    • B.

      Muscle training can prevent fatigue of respiratory muscles.


    • C.

      Rapid shallow breathing is a sign of respiratory muscle exhaustion.


    • D.

      Shock may precipitate respiratory muscle exhaustion and respiratory arrest.




Preferred response: D


Rationale


Shock can diminish oxygen supply to the respiratory muscles and precipitate muscle exhaustion. Respiratory arrest is a final common pathway to death in all forms of untreated shock.


Hypoxemia also can cause respiratory muscle exhaustion. Although myoglobin may provide a short-term buffer to transient oxygen deficits, it cannot prevent persistent hypoxia or blood flow limitation from progressing to respiratory muscle exhaustion. Krebs cycle production of adenosine triphosphate is very inefficient. Most Krebs cycle adenosine triphosphate is generated by oxidation of Krebs cycle reducing fragments. When there is insufficient oxygen to feed oxidative phosphorylation, reducing fragments build up and inhibit Krebs cycle enzymes. Anaerobic metabolism cannot prevent respiratory muscle exhaustion. Muscle training improves the ability of respiratory muscles to meet excessive demand, although only to a point. Rapid shallow breathing is a compensatory mechanism to deal with elevated work of breathing and occurs before respiratory muscles become exhausted.



  • 7.

    Which of the following values is the least likely to stimulate the peripheral or central chemoreceptors, resulting in an increase in respiratory rate?



    • A.

      Low hemoglobin


    • B.

      Paco 2


    • C.

      Pao 2


    • D.

      pH




Preferred response: A


Rationale


Hypoxemia is a powerful stimulus to ventilation mediated by sensory input originating in the carotid body chemoreceptor. Peripheral chemoreceptor activity (reflected in minute ventilation) increases slightly with decrements in Pao 2 below 500 mm Hg and rises steeply as Pao 2 falls below 50 mm Hg. Low oxygen tension, rather than low oxygen content, is the important ventilator stimulus. Little carotid body response results from profound anemia. Carotid chemoreceptor response also contributes to arousal from sleep during episodes of hypoxia. Hydrogen ion concentration and carbon dioxide tension independently activate chemoreceptors in the carotid body and in the brainstem. The simultaneous presence of hypoxia augments the hypercapnic ventilatory response.


Chapter 45 : Ventilation/perfusion inequality




  • 1.

    The primary abnormality of gas exchange in all patients with respiratory distress syndrome (ARDS) include



    • A.

      Decreased alveolar-end capillary diffusion capacity


    • B.

      Decreased ventilation/perfusion (V/Q) matching


    • C.

      Increased intrapulmonary shunt


    • D.

      Hypoventilation




Preferred response: C


Rationale


The primary gas exchange abnormality of ARDS is intrapulmonary shunt. Some, but not all, patients will have areas of low V/Q in addition to shunt. Consequently, increases in the fraction of inspired oxygen (F io 2 ) usually have little influence on arterial oxygenation. Patients with ARDS do not suffer from abnormalities in alveolar-end capillary diffusion capacity or hypoventilation unless there are other conditions complicating the ARDS. Patients with ARDS do experience elevated alveolar Pco 2 and hypercapnia, which may be made worse if excessive levels of positive-end expiratory pressure (PEEP) are employed. According to the alveolar gas equation, elevations in the alveolar partial pressure of carbon dioxide (Paco 2 ) will result in a fall in alveolar and arterial oxygenation. These decrements are small, responsive to increases in F io 2 , and not the primary cause of hypoxemia in patients with ARDS.



  • 2.

    The abnormalities of gas exchange and the site of airflow obstruction in patients with asthma are, respectively:



    • A.

      Atelectasis and distal airways


    • B.

      Decreased alveolar-end capillary diffusion capacity and distal airways


    • C.

      Decreased ventilation/perfusion (V/Q) matching and proximal airways


    • D.

      Hypoventilation and distal airways


    • E.

      Intrapulmonary shunt and proximal airways




Preferred response: C


Rationale


The primary gas exchange abnormality of asthma is V/Q mismatch which occurs in the distal airways due to edema and/or mucus production while airflow obstruction occurs due to bronchoconstriction in larger more proximal airways. Although the same pathology produces the two pathophysiologic mechanisms, no correlation exists between measurements of airway obstruction and gas exchange. Bronchodilators may augment flow to areas of low V/Q match, temporarily worsening hypoxemia.



  • 3.

    Which of the following is most accurate regarding West zones of the lung



    • A.

      In zone 2, perfusion is determined by the difference between alveolar pressures and pulmonary venous pressures.


    • B.

      In zone 3, the greatest pressure is the alveolar pressure.


    • C.

      Zone 1 approximates dead space areas of the lung.


    • D.

      Zone 1 conditions are commonly seen with increased pulmonary blood flow.




Preferred response: C


Rationale


The West zones describe the forces affecting pulmonary blood flow (PBF), which are determined by the gravitational influences on the lung. In zone 1, the alveolar pressure (PA) exceeds both the pulmonary arterial (PPA) and venous pressures (PV). In this zone there is little to no blood flow so it approximates dead space, but such conditions are rare except in cases of diminished PBF. In zone 2, PPA exceeds PA, which is greater than PV, and perfusion in this zone is determined by the pressure difference between alveolar and pulmonary venous pressures. In zone 3, the pressures descend from PPA to PV and finally PA.



  • 4.

    Shunt fraction can be calculated. Which of the following data would be required?



    • A.

      Arterial oxygen content, alveolar-arterial oxygen difference, cardiac output


    • B.

      Arterial oxygen content, venous oxygen content, assumption of oxygen content of blood undergoing gas exchange


    • C.

      Cardiac output, arterial oxygen saturation, and oxygen consumption


    • D.

      Dead space fraction, minute ventilation, and cardiac output




Preferred response: B


Rationale


The equation for calculating the shunt fraction is based on the Fick equation


Qs/Qt=CcO2CaO2CcO2CVO2
Qs/Qt=CcO2-CaO2CcO2-CVO2


where Q s and Q t represent shunt and total pulmonary blood flow, respectively, and C a O 2 , C v O 2 , and C c O 2 are the arterial, venous, and pulmonary capillary oxygen contents, respectively. The dead space fraction can be derived from measurement of mixed end-tidal CO 2 and arterial Pco 2 but is not needed for calculation of shunt fraction. Minute ventilation also is not needed for the calculation of the shunt fraction, but total cardiac output is part of the equation as seen above. A similar equation employing carbon dioxide contents in place of oxygen contents could be constructed.


Although cardiac output is derived in the calculation of shunt fraction, response C is incomplete as the equation also requires contents. Alveolar ventilation and barometric pressure are not required for the calculation. Response A has only one of the necessary variables for the equation. Establishment of a significant shunt fraction will inform the clinician that there will be decreased alveolar-arterial oxygen difference locally.



  • 5.

    Regarding the effect of intrapleural pressures and alveolar size, which of the following statements is accurate?



    • A.

      The intrapleural pressure at the apex is more negative than at the base, so the alveoli are larger at the apex.


    • B.

      The intrapleural pressure at the apex is more positive than at the base, so alveoli are smaller at the apex.


    • C.

      The intrapleural pressures do not vary between the apex of the lung and the base and therefore do not affect alveolar size.


    • D.

      The intrapleural pressure at the base is more positive than at the apex, so alveoli are larger at the base.




Preferred response: A


Rationale


The lung is a viscoelastic structure encased in the supporting chest wall with gravity imposing a globular shape on the lung. Pleural pressure is more negative at the apex of the lung compared with the base, increasing approximately 0.25 cm H 2 O per centimeter of vertical distance toward the lung base. Thus transpulmonary pressure is more marked at the apex so apical alveoli are large and at the upper end of the normal pressure-volume curve. They distend less for a given pressure change, that is, they are less compliant. In the spontaneously breathing upright human, maximal gas distribution occurs at the base and progressively diminishes toward the lung apex. This gradient also exists when inhalation occurs in the supine or lateral decubitus position, although to a lesser degree.



  • 6.

    A 7-year-old child with status asthmaticus is undergoing treatment in your pediatric intensive care unit with systemic corticosteroids, β 2 -agonists, ipratropium, and 0.60 fraction of inspired oxygen. She has moderate air entry, bilateral wheezes, no nasal flaring, and mild intercostal retractions. Her respiratory rate is 22 per minute. Her pulse oximetry saturations prior to and after initiation of therapy were 91% and 86%, respectively. Which of the following is the most likely explanation for this observed change in oxygen saturation?



    • A.

      Excessive fatigue with hypoventilation and resultant hypoxemia


    • B.

      Increase in airway secretion due to the institution of ipratropium


    • C.

      Increase in ventilation/perfusion mismatch due to β 2 -agonist


    • D.

      Mucus plugging of the airways due to institution of ipratropium




Preferred response: C


Rationale


In persons with asthma, high inspired oxygen concentrations may prevent hypoxic pulmonary vasoconstriction and place low alveolar ventilation (V a )/perfusion (Q) regions at risk for absorption atelectasis, and high doses of bronchodilators may enhance the perfusion of low V a /Q areas, exacerbating V a /Q mismatch. However, the beneficial effects of bronchodilators on airway resistance generally outweigh the worsening in V a /Q mismatch.


This child is not showing signs of excessive fatigue. There is no nasal flaring, and retractions are only mild. The air entry is moderate and wheezes are present, therefore response A is not correct. Ipratropium is an anticholinergic that causes a decrease in airway secretion (thus response B is incorrect), and there are no clinical signs that this child has a mucus plug in her airway (thus response D is incorrect).



  • 7.

    One of the proposed mechanisms for improvement in oxygenation in patients with acute respiratory distress syndrome (ARDS) placed in the prone position is:



    • A.

      Perfusion is greater in the nondependent areas in the prone position.


    • B.

      Perfusion is greater in the dependent areas in the prone position.


    • C.

      Ventilation is greater in the dependent areas in the prone position.


    • D.

      Ventilation is lower in the nondependent areas in the prone position.




Preferred response: A


Rationale


In ARDS and other lung injury models, nonaerated or poorly aerated portions of the lung are found mainly in the dependent areas. Perfusion is largely gravity-independent, especially in West zone 3 conditions. The majority of perfusion goes through dorsal lung regions, whether in the prone or supine position. Consequently, perfusion is greatest to the dependent lung in the supine position and to the nondependent lung in the prone position. Positive pressure, especially positive end-expiratory pressure, redistributes perfusion toward the dependent portion of the lungs by creating the condition of West zones 2 and 1. This redistribution may increase the vertical perfusion gradient in the supine position but may reduce it in the prone position. These various physiologic factors contribute to the increase in the uniformity of perfusion in the prone position.



  • 8.

    You are caring for a 4-month-old child with bronchiolitis who has developed respiratory failure. You instruct the medical student rotating on your service that bronchiolitis has features of both restrictive and obstructive lung diseases. Common pulmonary abnormalities that occur in both restrictive as well as obstructive lung diseases are:



    • A.

      Altered closing capacity especially with respect to functional residual capacity (FRC)


    • B.

      Increased intrapulmonary shunt fraction


    • C.

      Increased time constants


    • D.

      Increased zone 3 pulmonary blood flow conditions




Preferred response: A


Rationale


In almost all lung diseases alterations in closing capacity and functions residual capacity (FRC) occur resulting in altered gas exchange. Obstructive lung diseases (e.g., asthma) are characterized by increases in FRC and TLC, whereas in restrictive lung diseases (e.g., ARDS) both capacities are decreased. However, closing capacity changes as well and so in asthma there is increased V a /Q mismatch, whereas in ARDS there is increased intrapulmonary shunt. Many other lung diseases result in V a /Q mismatch or increased intrapulmonary shunt, and some of these diseases are discussed further in the chapter. Increased alveolar-capillary diffusion time is seen in diseases that result in abnormalities of the alveolar-capillary interface, such as pulmonary fibrosis, and is not a defining feature of either obstructive or restrictive lung diseases. The time constant of lung segments is defined by the equation


τ=R×C
τ=R×C


where the time constant (τ) is the product of resistance (R) and compliance (C) or R divided by elastance (E), the inverse of compliance. In general the time constant is increased in obstructive lung diseases and decreased in restrictive lung disease. However, lung diseases tend to be inhomogeneous and so some local areas may have time constants that differ markedly from the lung as a whole. This is especially true in bronchiolitis. Finally, zone 3 in the West model of pulmonary blood flow occurs when pulmonary arterial and venous pressures exceed pulmonary alveolar pressures. In obstructive lung disease, airways become hyperinflated, increasing alveolar pressure, and this will likely diminish the amount of zone 3 areas. Similarly, in restrictive diseases, there is loss of lung volume or pulmonary edema that increases the amount of zone 4 regions.



  • 9.

    A 7-month-old girl with severe pneumococcal pneumonia develops respiratory failure requiring intubation of her trachea and institution of mechanical ventilation. Shortly after intubation an arterial blood gas shows the following results: pH 7.32, paco 2 36 mm Hg, pao 2 57 mm Hg on and F io 2 of 0.6. Her pulse oximeter reads 88%. Explanation for differences in the abnormalities seen in the levels of carbon dioxide and oxygen include:



    • A.

      Carbon dioxide has greater diffusion in blood than oxygen does.


    • B.

      The difference is entirely explained by the alveolar gas equation.


    • C.

      The greater density of CO 2 results in better gas exchange in gravitationally dependent areas of the lung.


    • D.

      The hemoglobin-oxygen binding curve has limited influence on blood CO 2 levels.




Preferred response: D


Rationale


Carbon dioxide does have greater density than oxygen and does diffuse more readily than oxygen. However, neither of these features of carbon dioxide would explain the differences in levels of oxygen and carbon dioxide in this patient with pneumonia. In patients with diffusion abnormalities (e.g., pulmonary fibrosis), especially in states of increased cardiac output, pulmonary capillary blood will equilibrate with the CO 2 in alveolar gas, but equilibrium may not be reached with the oxygen in alveolar gas. Still, pneumonia is not distinguished by decreased diffusion capacity. In patients with gas intrapulmonary shunt or V a /Q mismatch abnormalities the carbon dioxide levels in affected pulmonary capillary units will be elevated, but often this can be corrected by increases in alveolar minute ventilation. In more normal areas the CO 2 in the pulmonary capillary blood is below normal and corrects the high CO 2 of these areas when the blood mixes in the large pulmonary veins, and this probably is the case for this infant. In contrast, since the majority of oxygen in the blood is carried by hemoglobin, areas of abnormal gas exchange are characterized by pulmonary capillary blood that is not fully saturated. This cannot be corrected by less diseased areas of the lung because the small amount of dissolved oxygen in areas that are more normal will not improve the Po 2 enough to bring the saturations of arterial blood to normal levels.



  • 10.

    The correct statement regarding alveolar size and compliance in different lung regions in a healthy child in the upright position is:



    • A.

      Apical alveoli are larger and therefore more compliant than alveoli at the base of the lungs.


    • B.

      Alveolar size is the same at the apex and the base of the lungs, so compliance is equal.


    • C.

      Alveoli at the base of the lungs are larger and therefore more compliant than apical alveoli.


    • D.

      Alveoli at the base of the lungs are smaller and therefore more compliant than apical alveoli.




Preferred response: D


Rationale


Apical alveoli are large and at the upper end of the normal pressure-volume curve. They distend less for a given pressure change, that is, they are less compliant. Alveoli at the base are smaller and more compliant than apical alveoli.



  • 11.

    What is the definition of time constant?



    • A.

      The time required to inflate 63% of final lung volume and is equal to the product of resistance and compliance


    • B.

      The time required to inflate 95% of final lung volume and is equal to the product of resistance and compliance


    • C.

      The time required to inflate 63% of final lung volume and is equal to resistance/compliance


    • D.

      The time required to deflate 95% of final lung volume and is equal to resistance/compliance




Preferred response: A


Rationale


The time constant (the product of resistance and compliance) is defined as the time required for inflation to 63% of final lung volume, inflation being indefinitely prolonged. Therefore a given lung unit with a slow time constant will fill more slowly than one with a fast time constant and also will empty more slowly. Should the time constants of different lung units vary, as frequently happens in pulmonary illness, gas distribution will be determined in part by the rate, duration, and frequency of inhalation.



  • 12.

    In an upright individual with normal lung perfusion, which West zone represents a region where P A > P PV > P PA , where P A is alveolar pressure, P PA is pulmonary artery pressure, and P PV is pulmonary venous pressure?



    • A.

      Zone I


    • B.

      Zone II


    • C.

      Zone III


    • D.

      This representation does not exist




Preferred response: D


Rationale


The three-zone model of pulmonary blood flow has been widely used to explain the heterogeneity of perfusion within the lung. Three variables comprise the components of this model: pulmonary arterial (P pa ), alveolar (P a ), and pulmonary venous (P v ) pressures. The degree of blood flow within the lung depends on the relative magnitudes of these pressures within that zone. Zone 1 (P a > P pa > P v ) has negligible blood flow, because the higher alveolar pressure is believed to compress collapsible capillaries. This region is one of minimal gas exchange and “wasted” ventilation. Zone 1 conditions are rare except in cases of diminished pulmonary blood flow (e.g., hypotension and cardiac failure) or increased P a encountered during positive pressure ventilation. Zone 2 consists of the mid portions of the lungs in which P pa > P a > P v , where flow rate is determined by the difference between pulmonary arterial and alveolar pressure. Venous pressure does not influence the flow rate. Blood flow progressively increases with descent through this zone, because P pa increases whereas P a remains relatively constant.


In the lowest zone of the lung described by West, zone 3, P pa > P v > P a , therefore the arteriovenous pressure gradient (P pa -P v ) determines flow rate. This gradient remains relatively constant descending through this zone, although because pleural pressures increase less, blood flow is greater in more dependent areas of zone 3. A zone 4 region in the most dependent areas of lung also has been described. In this region, transudated pulmonary interstitial fluid increases interstitial pressures, thereby reducing blood flow; this effect is exaggerated as lung volume diminishes from total lung capacity to residual volume.



  • 13.

    Which West zone represents a region where P pa > P a > P v , where P a is alveolar pressure, P pa is pulmonary artery pressure, and P pv is pulmonary venous pressure?



    • A.

      Zone I


    • B.

      Zone II


    • C.

      Zone III


    • D.

      This representation does not exist




Preferred response: B


Rationale


See the rationale for Question 12.


Chapter 46 : Mechanical dysfunction of the respiratory system




  • 1.

    A child with advanced cirrhosis is admitted to the ICU for management of acute gastrointestinal bleeding. The child has severe ascites and has become progressively more obtunded and hypoxemic. Which of the following mechanisms is most likely to result in a negative cardiovascular response to the institution of positive pressure ventilation in this patient?



    • A.

      A disproportionate increase in pleural pressure as lung volume increases


    • B.

      An increased need for sedation after endotracheal intubation


    • C.

      A negative effect of PEEP on left ventricular contractility


    • D.

      A reduction in lung compliance after the addition of PEEP




Preferred response: A


Rationale


Abdominal distention decreases chest wall compliance and, as a result, pleural pressure increases markedly as the lungs expand, decreasing venous return.



  • 2.

    Which of the following statements describes most accurately the relationship between the pressure displayed by the ventilator (measured at the endotracheal tube connector) and alveolar pressure during volume-controlled (constant flow) and pressure-controlled (decelerating inspiratory flow) mechanical ventilation?



    • A.

      The difference between ventilator pressure and alveolar pressure is determined by lung compliance in both modes.


    • B.

      The difference between ventilator pressure and alveolar pressure is not affected by air leaks around the endotracheal tube during pressure-controlled ventilation.


    • C.

      Ventilator pressure is higher than alveolar pressure at end-inspiration during volume-controlled ventilation.


    • D.

      Ventilator pressure is higher than alveolar pressure at end-inspiration during pressure-controlled ventilation.




Preferred response: D


Rationale


During volume-controlled ventilation, inspiratory flow continues throughout inspiration. Consequently, at the end of inspiration, there is a gradient of pressure between the endotracheal tube connector and the alveoli. During pressure-controlled ventilation, flow decreases and ultimately ceases during inspiration. Thus the two pressures tend to equilibrate at the end of inspiration.



  • 3.

    Which of the following conditions is most likely to increase the volume displacement of the diaphragm in a spontaneously breathing infant?



    • A.

      Abdominal distention


    • B.

      Croup


    • C.

      Pulmonary edema


    • D.

      Spinal cord section at the level of C8




Preferred response: D


Rationale


Intercostal muscle paralysis causes severe rib cage distortion during inspiration, adding to the volume that the diaphragm has to displace during a breath. All the other conditions tend to decrease tidal volume.



  • 4.

    Regarding transmural pressures of thorax (PTH), lung (PL), and chest wall (PW), which of the following is correct (PA = alveolar pressure, Pb = atmospheric pressure, and Ppl = pressure at pleural surface)?



    • A.

      PL = PA − Pb


    • B.

      PL = PA − Ppl


    • C.

      PTH = Ppl − Pb


    • D.

      PW = PA − Ppl




Preferred response: B


Rationale


Different pressures are needed to inflate the thorax, the lungs, and the chest wall. When the respiratory muscles are completely relaxed, the thorax, the lungs, and the chest wall are all held at their respective volumes by outward-acting pressure gradients across their walls. These pressure gradients or transmural pressures are defined by the following equations (see Fig. 136.18 ): (1) PTH = PA − Pb, (2) PL = PA − Ppl, and (3) PW = Ppl − Pb.



  • 5.

    Which term is used to describe the property responsible for the development of pressure-volume loops of the respiratory system during breathing?



    • A.

      Compliance


    • B.

      Elastance


    • C.

      Hysteresis


    • D.

      Resistance





• Fig. 136.18


Preferred response: C


Rationale


Analysis of the volume-pressure relationships of the thorax and its components becomes more complicated when pressure changes generated by gas flow and by the movement of the lung and chest wall tissue as the lungs inflate and deflate are considered. These pressure changes result from molecular interactions between the gas and the airway walls, within the gas stream itself, and among the components of the gas-liquid interface and the tissue.


These molecular interactions always result in a net loss or dissipation of energy from the respiratory system. The lost energy can no longer be used to perform work, and consequently dissipative pressure losses cause the volume-pressure relationships of the respiratory system to follow a different trajectory depending on the direction of the volume change. This property, known as hysteresis, is responsible for the development of loops when the volume-pressure relationships are plotted continuously during a breath. In this graphic representation, the dissipative pressures can be easily identified as the horizontal distance between the volume-pressure tracing and the corresponding point on the elastic volume-pressure relationship. The work done against these pressures can be quantified as the area enclosed by the loop (see Figure 136.18 , below).



  • 6.

    Regarding elastic properties of the lung and chest wall, which of the following statements is correct?



    • A.

      Elasticity is a dissipative property of the lungs and chest wall.


    • B.

      Outward elastic recoil of the lungs is counterbalanced by the inward elastic recoil of the chest wall.


    • C.

      The relaxation volume of the lungs in an adult is the residual volume.


    • D.

      The negative pressure generated between the lung tissue and chest wall contributes to venous return at normal lung volumes.




Preferred response: D


Rationale


Elasticity is typically a nondissipative process because the energy needed to produce elastic deformation during inspiration is accumulated in the tissues and then used to empty the lungs during expiration. In contrast, all resistive processes are dissipative: The energy liberated by the friction of the gas against the airway walls or by the molecular interactions within the tissue is transformed into heat and transported outside of the system by the blood or the expired gas.


Elastic pressures result from the tendency of the components of the lungs and chest wall to recover their original shape after undergoing deformation. By definition, elastic recoil drives the thorax and its individual components to adopt a volume, known as the relaxation volume, at which recoil itself is extinguished. The relaxation volumes of the lungs and the chest wall are the volumes that each of these components would adopt if all the mechanical constraints imposed by their mutual attachments and interactions were removed. The relaxation volume of the thorax, in contrast, is defined by the mechanical interaction of the lungs and the chest wall. It coincides with the point at which the opposing elastic recoils of these two components neutralize each other.


In the adult, the relaxation volume of the lungs is lower than the residual volume (the volume of gas contained in the lungs at the end of a forced expiration). The relaxation volume of the chest wall, by contrast, exceeds 50% of the vital capacity (the maximal volume of gas that can be inhaled from residual volume). This discrepancy in the relaxation volumes of the lungs and chest wall has three important consequences. First, it forces the relaxation volume of the thorax as a whole to occupy a position intermediately between the relaxation volumes of the lungs and the chest wall (at approximately 35% of vital capacity). Under most circumstances, this volume coincides with the functional residual capacity (FRC), which is the volume contained in the lungs at the end of a tidal expiration. Second, as the thorax starts to rise above its relaxation volume during inspiration, the outward recoil of the chest wall contributes to the expansion of the lungs, thereby reducing the work that the respiratory muscles need to perform during normal circumstances. Finally, at normal breathing volumes, the opposing actions of the lungs and chest wall recoils create a negative pressure at the boundaries of the lung tissue with the chest wall and the other intrathoracic structures. This negative pressure is an important contributor to the return of venous blood to the chest.


In an infant, the chest wall generates remarkably little outward recoil within the normal range of breathing volumes. Because the inward recoil of the lungs varies little with respect to lung size and age during development, the relaxation volume of the infant’s thorax is proportionally smaller than that of the adult. If, as occurs in the adult, the FRC coincided with this relaxation volume (15% of vital capacity compared with 35% in the adult), then the infant would be at a definite disadvantage in terms of alveolar stability and oxygenation. Newborns of most mammalian species have developed physiologic strategies to maintain their FRC above the relaxation volume of the thorax.



  • 7.

    Maintenance of alveolar stability and oxygenation in neonates depend on physiologic strategies to do which of the following?



    • A.

      Increase closing capacity


    • B.

      Increase residual volume


    • C.

      Increase vital capacity


    • D.

      Maintain the FRC above the relaxation volume of the thorax




Preferred response: D


Rationale


Newborns of most mammalian species have developed physiologic strategies to maintain their FRC above the relaxation volume of the thorax. These strategies are generally directed at interrupting expiratory flow before expiration is complete and include shortening of the expiratory time, contraction of adductor muscles of the glottis to retard exhalation, and persistence of the tonic activity of the inspiratory muscles during expiration.



  • 8.

    Compared with adults and older children, premature infants and neonates are able to tolerate high lung volumes during mechanical ventilation without cardiovascular compromise, possibly because of which of the following conditions?



    • A.

      Their lower airway resistance


    • B.

      Their higher cardiac contractility


    • C.

      Their higher cardiac output


    • D.

      Their higher chest wall compliance




Preferred response: D


Rationale


The pressure inside the pleural space is really determined by the elastic recoil of the chest wall and the volume of the thoracic contents. As long as lung volume is not forced above its normal range and chest wall compliance is unaltered by disease, pleural pressure (and thus the pressure around the major vessels and the heart) remains low, regardless of the airway pressures. Conversely, excessive lung distention (e.g., in asthma) is always associated with a high pleural pressure and, for that reason, is less well tolerated from a cardiovascular point of view. The dependence of pleural pressure on chest wall compliance explains why premature infants and newborns, who have very large chest wall compliance, have limited changes in this pressure during positive pressure ventilation, even if physiologic lung volumes are exceeded. Disease-induced reductions in chest wall compliance, on the other hand, always increase pleural pressure and reduce venous return to the heart. This is one reason why patients with abdominal distention typically have low cardiac output and why relief of the distention (e.g., by paracentesis in patients with ascites) reduces pleural pressure and increases cardiac output.



  • 9.

    Which one of the following areas is considered the zero reference during inspiration?



    • A.

      The alveoli


    • B.

      The extrathoracic airway


    • C.

      The intrapleural space


    • D.

      The mouth




Preferred response: D


Rationale


Airway transmural pressure varies during breathing. Its variations result from the fact that inspiration and expiration have very different effects on the pressures inside and outside the airways. The pressure inside all airways undergoes qualitatively similar changes during each phase of the breathing cycle. During inspiration, for instance, there is a gradient of increasingly negative pressures from the mouth, where pressure is atmospheric (or the zero reference), to the alveolar spaces, where the pressure must be negative (or subatmospheric) for gas to flow in. This negative pressure is of course driven by the actions of the respiratory muscles and transmitted to the lungs via the link between the chest wall and lungs at the pleural space. During expiration, alveolar pressure becomes positive and the gradient is inverted, with the pressures inside the airways being always positive but diminishing toward the mouth.



  • 10.

    What happens to the extrathoracic and intrathoracic portion of the airways during a normal respiratory cycle of inspiration and expiration with airflow?



    • A.

      During inspiration, the pressure surrounding the extrathoracic airway is more positive than the lumen, resulting in narrowing of the airway.


    • B.

      During inspiration, the pressure surrounding the extrathoracic airway is more negative than the lumen, resulting in narrowing of the airway.


    • C.

      During expiration, the pressure surrounding the intrathoracic airway is more negative than the lumen, resulting in narrowing of the airway.


    • D.

      During expiration, the pressure surrounding the intrathoracic airway is more negative than the lumen, resulting in dilation of the airway.




Preferred response: A


Rationale


During inspiration, there is a gradient of increasingly negative pressures from the mouth (zero reference), where the pressure is atmospheric, distally to the airway. The pressure at the alveolar spaces must be negative (or subatmospheric) for air to flow. The pressure inside the airways is always negative, regardless of intra- or extrathoracic location.


During expiration, for air to move from the alveolar spaces to the mouth, the pressures must be more positive distally and decrease toward the mouth. Normal breathing changes in airway caliber depend on the location of the airway (intrathoracic versus extrathoracic) and the phase of the respiratory cycle (inspiration versus expiration).


Extrathoracic airways include the pharynx, larynx, and extrathoracic portion of the trachea and are surrounded by neck tissue, maintaining constant atmospheric pressure around these areas. During inspiration, the pressures distal to the extrathoracic airway become more negative, causing the extrathoracic portions of the airway to narrow. The opposite holds true for the intrathoracic airway. During inspiration, the pressure surrounding their walls (intrapleural pressure) becomes more negative compared with the pressure inside the lumen, which causes these portions of the airway to dilate.


To be correct, responses B through D should read as follows:



  • B.

    During inspiration, the pressure surrounding the extrathoracic airway is more positive than the lumen resulting in narrowing of the airway.


  • C.

    During expiration, the pressure surrounding the extrathoracic airway is less positive than the lumen, resulting in dilation of the airway.


  • D.

    During expiration, the pressure surrounding the intrathoracic airway is more positive than the lumen, resulting in narrowing of the airway.





  • 11.

    What is the equal pressure point?



    • A.

      During inspiration, the point at which transmural pressure is positive


    • B.

      During inspiration, the point at which transmural pressure is negative


    • C.

      During expiration, the point at which the pleural pressure equals alveolar pressure


    • D.

      The pressure midway in the airway, where the pressure distal to the mouth equals the pressure proximal to the alveoli




Preferred response: C


Rationale


The narrowing of the intrathoracic airways during expiration is contingent on the existence of a pressure gradient from the alveoli to the mouth. Alveolar pressure (P a ) must always exceed pleural pressure (P pl ) by a magnitude equivalent to the elastic recoil of the lungs. As the gas progresses downstream during expiration, frictional pressure losses lower the pressure inside the airways. Eventually the cumulative pressure losses can be as large as the pulmonary elastic recoil, and the pressure inside the airways becomes equal to P pl . Beyond this equal pressure point, airway transmural pressure becomes negative (i.e., the pressure outside exceeds the pressure inside the airway) and acts to collapse the airway.



  • 12.

    Stridor in a child with croup occurs during which process?



    • A.

      Expiration due to a more positive pressure downstream from the obstruction


    • B.

      Expiration due to a more negative pressure downstream from the obstruction


    • C.

      Inspiration due to a more negative pressure downstream from the obstruction


    • D.

      Inspiration due to a more positive pressure downstream from the obstruction




Preferred response: C


Rationale


When airway obstruction is extrathoracic (e.g., with croup, glossoptosis, or tonsil or adenoid hypertrophy), the person must create a more negative pressure inside the airway segment downstream from the obstruction to overcome the increased resistance during inspiration. Therefore this segment of the airway tends to collapse, worsening the obstruction and producing a characteristic turbulent noise (inspiratory stridor) as gas accelerates through the narrowest point and induces vibrations in the airway mucosa, creating in the process a decrease in inside pressure that approximates the walls of the airway even further. The obstruction is relieved during expiration because the pressure inside the airway segment, now upstream from the obstruction, must become more positive with respect to atmospheric pressure to force gas flow through the obstruction (see Figure 136.19 , below).



  • 13.

    When does wheezing in a child with tracheobronchial compression occur?



    • A.

      During inspiration and expiration due to a more positive pressure downstream from the obstruction


    • B.

      During expiration due to a less positive pressure downstream from the obstruction


    • C.

      During expiration due to a more positive pressure downstream from the obstruction


    • D.

      During inspiration due to a more positive pressure downstream from the obstruction



Jun 26, 2021 | Posted by in CRITICAL CARE | Comments Off on Board review questions

Full access? Get Clinical Tree

Get Clinical Tree app for offline access